Download as pdf or txt
Download as pdf or txt
You are on page 1of 347

2019

JC1 H2 Math
1. Anderson Serangoon JC

2. Anglo Chinese JC

3. Catholic JC

4. Dunman High School

5. Eunoia JC

6. Hwa Chong Institution

7. Nanyang JC

8. National JC

9. Raffles Institution

10. River Valley High School

11. St. Andrew's JC

12. Tampines Meridian JC

13. Temasek JC

14. Victoria JC

15. Yishun Innova JC


ANDERSON SERANGOON
JUNIOR COLLEGE
MATHEMATICS 9758
4 Oct 2019
H2 Mathematics Paper (100 marks)
3 hours
Additional Material(s): List of Formulae (MF26)

CANDIDATE
NAME

CLASS /

READ THESE INSTRUCTIONS FIRST Question


Marks
number
Write your name and class in the boxes above.
1
Please write clearly and use capital letters.
Write in dark blue or black pen. HB pencil may be used for graphs 2
and diagrams only.
Do not use staples, paper clips, glue or correction fluid. 3

Answer all the questions and write your answers in this booklet. 4
Do not tear out any part of this booklet.
5
Give non-exact numerical answers correct to 3 significant figures, or
1 decimal place in the case of angles in degrees, unless a different 6
level of accuracy is specified in the question.
You are expected to use an approved graphing calculator. 7
Where unsupported answers from a graphing calculator are not
allowed in a question, you are required to present the mathematical 8
steps using mathematical notations and not calculator commands.
9
All work must be handed in at the end of the examination. If you
10
have used any additional paper, please insert them inside this
booklet. 11
The number of marks is given in brackets [ ] at the end of each
question or part question. 12

Total

This document consists of 22 printed pages and 2 blank pages.

[Turn Over
2

1 The graph of y = f(x) undergoes transformations in the following order:


I. Reflection in the x-axis
II. Translation in the positive x-direction by 4 units
1
III. Scaling parallel to the x-axis by a scale factor of
3
The equation of the resulting graph is e 2 y 7  3x .
Find the equation of the original graph in the form y f x . [3]

2 The curve of a cubic polynomial y = f(x) has y-intercept 5 and a turning point at
1,1 . Given that it passes through 2, 7 , find the equation of the curve. [4]
3

A curve C has equation ln y  3xy  x 2  4 0.


2
3

(i) Show that


dy 3 y  2x y .
[3]
dx 2ln y  3xy

(ii) It is given that C contains the point A with coordinates D ,1 where D  0 .


Find the value of D and hence the equation of the normal to C at the point A. [3]

[Turn Over
4

4 The diagram shows a sketch of the curve y = f(x). The curve cuts the x-axis at
3, 0 and 6, 0 . It has a stationary point at 0, 3 and asymptotes x 5 and
y 3.
y

y = f(x)
y=3

x
0

On two separate diagrams, sketch the graphs of the following equations. Show
clearly the equations of asymptotes and the coordinates of the points of intersection
with the axes, if any.
1
(a) y , [3]
f ( x)
5

(b) y f x . [3]

d
5 (a) Find sin 2 x  2 x cos 2 x . Hence, or otherwise, find ³x
2
cos 2 x dx. [4]
dx

[Turn Over
6

1
(b) If 0  a  1, find ³ (a  x) a  x dx , leaving your answer in terms of a. [3]
0

6 (a) It is given that


­° 2  9  ( x  3) 2 for 0  x  6,
f ( x) ®
°̄ 2 for 6 d x d 12.
and that f x f x  12 for all real values of x.
(i) Sketch the graph of y = f(x) for – 11 d x d 12. [3]
7
10
(ii) Find the exact value of ³ f ( x) dx . [2]
9

2x2  3
(b) The curve C has equation y .
x2  6
Using an algebraic method, find the set of values of y that C can take. [3]

[Turn Over
8

7 Referred to the origin O, a, b and c are non-zero and non-parallel vectors denoting
the position vectors of the points A, B and C respectively.
(i) Given that a u b 3a u c , show that b  3c Oa where O is a scalar. [2]

The point M is the mid-point of OC and the point N lies on OB produced such that
3ON = 5OB. The point P lies on MN such that MP: MN = 2 : 3.
10 1
(ii) Show that the position vector of P is b c. [1]
9 6
9

It is given that b is a unit vector, c 2 , b  3c 5 and the angle between b and

c is 45q .
o o
(iii) Find the exact length of projection of OP on OA . [4]

[Turn Over
10

8 (a) The curve C is defined by the equations


S
x 2cosec t , y 5cos t sin 3 t , for 0  t d
.
2
The region R is bounded by C, the x-axis and x = 4 in the first quadrant. Find
the exact area of the region R. [6]
11
(b) The diagram below shows the curves C1 and C2 with equations
2 x  1  3 and y 3x2 respectively. The region in the first quadrant
2
y
enclosed by the curves and the y-axis is denoted by S. Find the volume of the
solid generated when the region S is rotated through 2S radians about the
y-axis, giving your answer correct to 4 decimal places.
y

C1
(0, 5)
(1, 3)
S
C2
x
0
[3]

[Turn Over
12

9 (a) The function g is defined by


g : x ax  b , x  , x ! 0 ,
where a and b are positive real numbers.
Show that g 2 exists and hence determine the range of g 2 , leaving your answer
in terms of a and b. [3]

(b) Function h is defined by


x7
h:x , x  , x z 1.
x 1
(i) Find h 1 x and state the domain of h 1 . [3]
13
(ii) Find the exact values of c such that h 2018 c h 1 c . Explain your
answers clearly. [3]

(c) Function f is defined by


f :x 2 x 2  O x  5,
5, x  ,
where O is a non-zero constant.
1
(i) Give a reason why f does not exists. [1]

(ii) For the function f defined above, the range of f is [ 3, f ) . If the domain
1
of f is restricted to the set of all positive real numbers, f exists. Find
the value of O. [2]

[Turn Over
14

10 The point P has position vector 2i  5 j  k . The plane 31 has equation


r i  2k 10 .
(i) Find the coordinates of N, the foot of perpendicular from P to 31 . [3]

The point A with position vector 4i  D j  3k , where D  , lies on plane 31 .


The plane 3 2 has equation r i  3k 5.
(ii) Show that point A lies on the plane 3 2 . Hence, using an algebraic method,
find a vector equation of l, the line of intersection of 31 and 3 2 . Leave your
answer in terms of D . [3]
15
The plane 3 3 is the image of the plane 3 2 about the plane 31 . Given that point P
lies on plane 3 2 ,
(iii) find an equation of plane 3 3 in scalar product form. [3]

[Turn Over
16

The line l1 has equation x 4, y z  2.


(iv) Find the position vectors of the points on the line l1 such that the shortest
4 5
distance from those points to the plane 31 is units. [4]
5
17
THIS PAGE IS INTENTIONALLY LEFT BLANK

[Turn Over
18

11 (a)

a h

A decorative artifact, as shown in the diagram above, is in the shape of a cone with
radius r and height h with a sphere of fixed radius a inscribed in it.
S a 2h2
(i) Show that the volume of the cone, V, is given by V .
3 h  2a
1 2
[Volume of cone, V Sr h ] [3]
3
19
(ii) Use differentiation to find, in terms of a, the minimum value of V. Leave your
answer in exact form. [5]

[Turn Over
20

(b) In a triangle PQR, PQ 3 cm and PR 2 cm. If angle QPR is increasing at a


constant rate of 0.1 radians per second, find the rate of increase of the length
π
QR at the instant when angle QPR is radians. [4]
3
21
12 Ben and his wife, Jerrie, intend to save for their child’s education in the future.
(a) Jerrie decides to save $200 on 1st January 2020. On the first day of each
subsequent month she will save $50 more than in the previous month, so that
she saves $250 on 1st February 2020, $300 on 1st March 2020, and so on. Find
[4]
the earliest date for Jerrie to save at least $22000 in total.

[Turn Over
22

(b) Ben decides to put $400 into a bank account on 1st January 2020. On the first
day of each subsequent month from February 2020, he puts another $150 into
the account. The interest rate is 1% per month, such that on the last day of each
month, the amount in the account on that day is increased by 1%.
(i) Show that the value of Ben’s account on the last day of the nth month
(where January 2020 is the 1 st month, February 2020 is the 2 nd month, and
so on) is 15400 1.01  15150 .
n
[3]

(ii) Hence, calculate the earliest date for Ben to save at least $59900 in his
[3]
bank account.
23

(iii) Ben wants to save $88000 in his bank account by 2 nd January 2038 for his
child’s university education. Given that the initial amount deposited on 1 st
January 2020 remains at $400 and the interest rate is 1% per month, how
much does Ben need to put into his savings account every month, from
February 2020, instead, in order to have at least $88000 in his savings
account by 2nd January 2038? [3]

End of Paper

[Turn Over
24

BLANK PAGE
ANDERSON SERANGOON
JUNIOR COLLEGE
MATHEMATICS 9758
4 Oct 2019
H2 Mathematics Paper (100 marks)
3 hours
Additional Material(s): List of Formulae (MF26)

CANDIDATE
NAME

CLASS /

READ THESE INSTRUCTIONS FIRST Question


Marks
number
Write your name and class in the boxes above. 1
Please write clearly and use capital letters.
Write in dark blue or black pen. HB pencil may be used for 2
graphs and diagrams only.
Do not use staples, paper clips, glue or correction fluid. 3
Answer all the questions and write your answers in this 4
booklet.
Do not tear out any part of this booklet. 5
Give non-exact numerical answers correct to 3 significant
figures, or 1 decimal place in the case of angles in degrees, 6
unless a different level of accuracy is specified in the
question.
7
You are expected to use an approved graphing calculator.
Where unsupported answers from a graphing calculator are
8
eqqu
not allowed in a question, you are required to present the
emaatica
mathematical steps using mathematicaltiica
cal nnotations and not
9
calculator commands.
10
All work must bee hhanded
aan
nded
nde
ded in
de in att tth hhe en
the nd ooff tthe examination.
end
sed
eedd any
If you have used nyy additional
add
ddit
it iio
o nnaal paper,
ppaap please insert them
kleleet.
inside this booklet. t. 11
maark
The number of marks rks is
is given
giv in brackets [ ] at the end of
each question or part art question. 12

Total

This document consists of 22 printed pages and 2 blank pages.

[Turn Over
2

1 The graph of y f x undergoes transformations in the following order:


I. Reflection in the x-axis
II. Translation in the positive x-direction by 4 units
1
III. Scaling parallel to the x-axis by a scale factor of
3
The equation of the resulting graph is e 2 y 7  3 x .
Find the equation of the original graph in the form y f x . [3]

Solution
Method 1
x
Reverse III: Scaling parallel to the x-axis by a scale factor of 3 . (Replace x by )
3
Equation becomes: e 2 y 7  x
Reverse II: Translation in the negative x-direction by 4 units. (Replace x by x  4 )
Equation becomes: e 2 y 7  x  4
e2 y 3 x
Reverse I: Reflection in the x-axis (replace y by  y )
Equation becomes e2 y 3  x
1
Ÿ y f x  ln 3  x
2
Method 2
After the 3 transformations, y f x becomes y f 3 x  4
1
As the final curve is y  ln 7  3 x
2
1
So f 3 x  4 ln 7  3 x
2
z4
Let z 3 x  4 Ÿ x
3
1 ª § z  4 ·º 1
f z ln «7  3 ¨ ¸» ln 3  z
2 ¬ © 3 ¹¼ 2
1
y f x ln 3  x
2
Method 3
After the 3 transformations, y f x bbecomes y f 3 x  4
1
As the final curve is y ln 7  3 x
 ln
2
1
So f 3 x  4 llnn 7  3 x
2
1
f 3x  4 n ª  3 x  4  3º¼
ln
2 ¬
1
f x ln 3  x
2
2 The curve of a cubic polynomial y f x has y-intercept 5 and a turning point at 1,1 .
Given that it passes through 2, 7 , find the equation of the curve. [4]

Solution
Let the polynomial be f x ax 3  bx 2  cx  5 .
At 1,1 , a  b  c 4 1
At 2, 7 , 8a  4b  2c 2 Ÿ 4a  2b  c 1 2
f c x 3ax 2  2bx  c
When x 1 , 3a  2b  c 0 3
Using G.C,
a 1, b 2, c 7
Equation of curve is f x x3  2 x 2  7 x  5

A curve C has equation ln y  3xy  x 2  4 0.


2
3

(i) Show that


dy 3 y  2x y .
[3]
dx 2ln y  3xy
(ii) It is given that C contains point A with coordinates D ,1 where D  0 . Find the value
of D and hence the equation of the normal to C at the point A. [3]

Solution

[Turn Over
4

ln y
2
(i)  3xy  x 2  4 0.
Differentiate w.r.t. x:
§ 1 · dy § dy ·
2 ln y ¨ ¸  ¨ 3 x  3 y ¸  2 x 0
© y ¹ dx © dx ¹
§ 2ln y · dy
¨  3x ¸ 3 y  2x
© y ¹ dx
dy 3 y  2 x y
dx 2ln y  3xy
(ii) At A, y = 1. Substitute into equation of C:
3 x  x 2  4 0
( x  4)( x  1) 0
x 4 or  1
Since x  0, x 1 Ÿ D 1
dy 3  2(1) 5
dx 3(1) 3
3
Gradient of normal = 
5
Equation of normal to C at A is
3
y  1  ( x  1)
5
3 2
y  x
5 5

4 The diagram shows a sketch of the curve y f x . The curve cuts the x-axis at 3, 0
and 6, 0 . It has a stationary point at 0, 3 and asymptotes x 5 and y 3.

y = f(x)
y=3

x x
3 0

On separate diagrams,
iag
ag
gra
rams
ms, sk
sketch the graphs of
1
(i) y , [3]
f ( x)
(ii) y f x , [3]
indicating clearly, in each case, the equations of asymptotes and the coordinates of the
points of intersection with the axes, if any.

Solution
(i)

y 1
y
f ( x)

x x
0
x
§ 1·
¨ 0,  ¸
© 3¹

(ii)

d
5 (a) Find sin 2 x  2 x cos 2 x . Hence,
Hen or otherwise, find
He ³x
2
cos 2 x dx.
dx [4]
1
(b) d ³ (a  x) a  x d
If 0  a  1, find dxx , leaving your answers in terms of a.
0 [3]

Solution

[Turn Over
6

d
(a) sin 2 x  2 x cos 2 x
dx
= 2cos 2x – [2x (– 2sin 2x) + 2cos 2x]
= 2cos 2x + 4xsin 2x – 2cos 2
x 2 sin 2 x 1 1
 sin 2 x  x cos 2 x  C
2 4 2
1
(b) ³ (a  x) a  x
0
dx , 0  a  1
a 1
³ 0
(a  x)(a  x) dx  ³ (a  x)[(a  x)] dx
a
a 1
³ 0
a 2  x 2 dx  ³ a 2  x 2 dx
a
a 1
ª 2 x º ª 2 3
x3 º
«   
3 »¼ 0 «¬ 3 »¼ a
a x a x
¬
ª§ 3 a 3 · º ª § 2 1 · § 3 a 3 · º
«¨ a  ¸  0 »  « ¨ a  ¸  ¨ a  ¸ »
¬© 3 ¹ ¼ ¬© 3¹ © 3 ¹¼
4a 3 1
 a2 
3 3

6 (a) It is given that


­° 2  9  ( x  3) 2 for 0  x  6,
® f ( x)
°̄ 2 for 6 d x d 12.
and that f(x) = f(x + 12) for all real values of x.
(i) Sketch the graph of y = f(x) for – 11 d x d 12. [3]
10
(ii) Find the exact value of ³ f ( x) dx . [2]
9

(b) The curve C


Using an algebraic method, find the set of values of y that C can take.
[3]

Solution
(a)(i)
y
5

2
1
11 9
 6
6 3
3 0 3 6 9 10 12 15 18 21 24 x
10
(ii) ³ 9
f ( x) dx
dx = Ar
Areea under curve bounded by x-axis from x = 9 to x = 10
Area

§1 ·
3 ¨ S (3)2 ¸  19 2
©4 ¹
27
S  38
4
2x2  3
(b) y
x2  6
x2 y  6 y 2x2  3
y  2 x2  6 y  3 0
For the equation to have real solutions, discriminant t 0 , y z 2
0  4 y  2 6 y  3 t 0
y  2 2 y  1 t 0 , y z 2
 
0.5 2
­ 1 ½
hence, ® y  : y d  or y ! 2 ¾ .
¯ 2 ¿

7 Referred to the origin O, a, b and c are non-zero and non-parallel vectors denoting the
position vectors of the points A, B and C respectively.

(i) Given that a u b 3a u c , show that b  3c Oa where O is a scalar. [2]


The point M is the mid-point of OC and the point N lies on OB produced such that
3ON = 5OB. The point P lies on MN such that MP: MN = 2 : 3.
10 1
(ii) Show that the position vector of P is b c.
9 6 [1]
It is given that b is a unit vector, c 2 , b  3c 5 and the angle between b and c is
45q .
o o
(iii) Find the exact length of projection of OP on OA . [4]

Solution
(i)
a u b 3a u c
a u b  3a u c 0
a u b  3c 0
a is parallel to b – 3c , hence b  3c Oa .
(ii) By ratio theorem,
o o
o 2 ON  OM
OP
3
1§ 5 1 ·
¨ 2u b  c¸
3© 3 2 ¹

10 1 20b  3c
= b  c or
9 6 18

[Turn Over
8

OA 1 20b  3c ˜ a
(iii) length of projection OP ˜
OA 18 a

1 20b  3c ˜ O b  3c
18 O b  3c
1 20b ˜ b  57c ˜ b  9c ˜ c
18 b  3c
2 2
1 20 b  57c ˜ b  9 c
=
18 b  3c

1 20  18  57 c b cos 45
=
18 5
1 55
=
18 5
11
=
18

8 (a) The curve C is defined by the equations


S
x 2cosec t , y 5cos t sin 3 t , for 0  t d
.
2
The region R is bounded by C, the x – axis and x = 4 in the first quadrant. Find the
exact area of the region R. [6]
2 x  1  3 and
2
(b) The diagram below shows the curves C1 and C2 with equations y
y 3x2 respectively. The region in the first quadrant enclosed by the curves and the
y-axis is denoted by S. Find the volume of the solid generated when the region S is
rotated through 2S radians about the y-axis, giving your answer correct to 4 decimal
places.
y

(0, 5)
(0 5 C1

(1, 3)
S

C2
x
[3]

Solution
S
(a) x 2cosec t , y 5cos t sin 3 t , for 0  t d .
2
When x = 2, 2 2 cosec t.
cosec t 1
sin t 1
S
t
2
When x = 2, 4 2 cosec t.
cosec t 2
1
sin t
2
S
t
6
dx
2 cot t cosec t
dt
4
Area ³ 2
y dx
S

³S 5cos t sin t 2cot t cosec t dt


6 3

2
S
§ cos t ·§ 1 ·
10³S6 5cos t sin 3t ¨ ¸¨ ¸ dt
2 © sin t ¹© sin t ¹
S
10³S2 cos 2 t sin t dt
6
S
10³S2 cos2 t ( sin t ) dt
6
S
ª cos3 t º 2
10 « »
¬ 3 ¼S
6

10 ª § 3 · º
3

 «0  ¨¨ ¸ »
3 « © 2 ¸¹ »
¬ ¼
10 § 3 3 ·
¨ ¸
3 ¨© 8 ¸¹
5 3
4
(b)

[Turn Over
10

(0, 5) C1

(1, 3)
S

C2
x

Required Volume
2
3 y 5§ y 3 ·
S³ dy  S ³ ¨¨1  ¸ dy
0 3 3
© 2 ¸¹
5.759586 units3
5.7596 units3 (to 4 dp)

9 (a) The function g is defined by


g : x ax  b , x , x ! 0 ,
where a and b are positive real numbers.
Show that g 2 exists and hence determine the range of g 2 , leaving your answer in
terms of a and b.
[3]
(b) Function h is defined by
x7
h:x , x  , x z 1.
x 1
(i) Find h 1 x and state the domain of h 1 . [3]
(ii) Find the exact values of c such that h 2018
c h 1
c . Explain your answers
clearly. [3]
(c) Function f is defined by
f :x 2 x 2  O x  5,
5 x ,
where O is a non-zero constant.
nstanant.
t
(i) Give a reason whyy f do 1
does
es nnot exists. [1]
(ii) For the function
uncti
ttiioon
n f defined
deeffin b v the range of f is [ 3, f ) . If the domain of f is
i ed above,
abo
bo
triict
c ed
restricteded tto
o tth
he sse
the et off al
set positive real numbers, f 1 exists. Find the value of O.
alll po [2]

Solution
(a) For g 2 to exist, R g Ž Dg .
From the graph of g, R g (b, f)
Dg 0, f
Since b ! 0 , R g Ž Dg . Hence g 2 exists. y
g(x)=ax+b
Consider the graph of g.
By restricting the domain of g to the range of g,
R g (b, f)
(b, ab+b)
R g2 (ab  b, f)
x
b
x7
(bi) y
x 1
xy  y x7
x y  1 7 y
7 y
x
y 1
x7
h 1 x
x 1
Rh D h 1 \ ^1`
(ii)
h 2018 (c) h 1 (c)
hh 2018 (c) hh 1 (c)
hh -1hh -1...hh -1h(c) c h h 1
2019 times
h(c ) c
c7
c
c 1
c  7 c2  c
c 2  2c  7 0
2 r 4  4 7
c
2
c 1r 2 2

[Turn Over
12

(c)(i) f(x) 2x2  O x  5


§ O ·
2 ¨ x2  x¸  5
© 2 ¹
2
§ O· O 2
2¨ x  ¸  5 
© 4¹ 8
Method 1 (Counter-example)
2
§O· §O O· O 2
f(0) = 5, f ¨ ¸ 2¨  ¸  5  5
©2¹ ©2 4¹ 8
§O· O O
Since f 0 = f ¨ ¸ but 0 z , 0,  ,
©2¹ 2 2
Ÿ f is not 1-1
Ÿ inverse function of f does not exists.
Method 2 (Graphical)
y

y=5
5
x

Since the horizontal line y = 5 cuts the graph of f at 2 points, f is not 1-1.
Hence f -1 does not exists.
ª O2 ·
(ii) Range of f = «5  , f ¸ [3, f)
¬ 8 ¹
O2
5 3
8
O 2 82
O r8
Since f 1 exists when domain of f is restricted to the set of positive real numbers,
?O 8

10 or 2i  5 j  k . The plane 31 has equation r


The point P has position vecto
vector i  2k 10 .
(i) Find the coordinates
dinat
aattes
es off N,
N, the
tth
he fo ot ooff perpendicular from P to 31 .
foot [3]
The point A with
tth
h pposition
osiitt io
os eccttor 4i  D j  3k , where D  , lies on plane 31 . The plane
on vve
vector
3 2 has equation
ati on r
tiion i  3k 5.
(ii) Show that
att ppo int A llies on the plane 3 2 . Hence, using an algebraic method, find a
point
oin
int
ati of l, the line of intersection of 31 and 3 2 . Leave your answer in
vector equation
terms of D . [3]
The plane 3 3 is the image of the plane 3 2 about the plane 31 . Given that point P lies on
plane 3 2 ,
(iii) find an equation of plane 3 3 in scalar product form. [3]
The line l1 has equation x 4, y z  2.
(iv) Find the position vectors of the points on the line l1 such that the shortest distance
4 5
from those points to the plane 31 is units.
5 [4]

Solution
§ 2· § 1·
¨ ¸ ¨ ¸
(i) lPN : r ¨ 5 ¸  O ¨ 0 ¸ , O 
¨ ¸ ¨ ¸
©1¹ ©2¹
Since N lies on both lPN and 31 , then we have
§ 2  O · § 1 ·
¨ ¸ ¨ ¸
¨ 5 ¸ x ¨ 0 ¸ 10 for some O 
¨ ¸ ¨ ¸
© 1  2O ¹ © 2 ¹
Ÿ 2  O  2  4O 10
O 2
§0·
o ¨ ¸
Hence ON ¨ 5 ¸
¨ ¸
©5¹
So the coordinates of N are 0,5,5
§ 4 · § 1 ·
¨ ¸ ¨ ¸
(ii) Since ¨ D ¸ x ¨ 0 ¸ 4(1)  D (0)  3(3) 5
¨ ¸ ¨ ¸
© 3 ¹ © 3¹
Hence point A lies on 3 2
§ 1· § 1 · § 0 · §0·
¨ ¸ ¨ ¸ ¨ ¸ ¨ ¸
¨ 0 ¸u ¨ 0¸ ¨ 5¸ 5¨ 1 ¸
¨ ¸ ¨ ¸ ¨ ¸ ¨ ¸
© 2 ¹ © 3¹ ©0¹ ©0¹
§ 4 · §0·
¨ ¸ ¨ ¸
l : r ¨ D ¸  E ¨1¸, E 
¨ ¸ ¨ ¸
© 3¹ ©0¹

[Turn Over
14

(iii) Let P ' be the image of P about the plane 31


o o
o OP  OP '
ON
2
§0· § 2· § 2 ·
o ¨ ¸ ¨ ¸ ¨ ¸
OP ' 2 ¨ 5 ¸  ¨ 5 ¸ ¨ 5¸
¨5¸ ¨1¸ ¨9¸
© ¹ © ¹ © ¹
§  2 · § 4 · § 2 ·
¨ ¸ ¨ ¸ ¨ ¸
A vector parallel to 3 3 = ¨ 5 ¸  ¨ D ¸ ¨ 5  D ¸
¨ 9¸ ¨ 3¸ ¨ 6 ¸
© ¹ © ¹ © ¹
§ 2 · § 0 · § 6 ·
¨ ¸ ¨ ¸ ¨ ¸
¨5 D ¸u¨1¸ ¨ 0 ¸
¨ 6 ¸ ¨0¸ ¨ 2 ¸
© ¹ © ¹ © ¹
§  3 · §  4 · § 3 ·
¨ ¸ ¨ ¸ ¨ ¸
Equation of 3 3 : r ¨ 0 ¸ ¨ D ¸ ¨ 0 ¸ 15
¨1¸ ¨ 3¸ ¨1¸
© ¹ © ¹ © ¹
§ 4 · §0·
¨ ¸ ¨ ¸
(iv) l1 : r ¨ 0 ¸  J ¨ 1 ¸ , J 
¨ 2 ¸ ¨1¸
© ¹ © ¹
Let the point B on the plane 31 be 10, 0, 0 .
ª§ 4 · § 10 · º § 1·
«¨ ¸ ¨ ¸» ¨ ¸
«¨ J ¸  ¨ 0 ¸ » x ¨ 0 ¸
«¬¨© J  2 ¸¹ ¨© 0 ¸¹ »¼ ¨© 2 ¸¹ 4 5
Ÿ for some J 
5 5
§ 6 · § 1·
¨ ¸ ¨ ¸
¨ J ¸x¨ 0 ¸
¨J  2¸ ¨ 2 ¸
© ¹ © ¹ 4 5
Ÿ
5 5
Ÿ 2J  10 4
Ÿ 2J  10 r4
Ÿ J 7 or 3
Method 2
§ 4 · § 1·
¨ ¸ ¨ ¸
¨ J ¸ x ¨ 0 ¸  10
¨J  2¸ ¨ 2 ¸
© ¹ © ¹ 4 5
Ÿ for some J 
5 5
Ÿ 2J  10 4
Ÿ 2J  10 r4
Ÿ J 7 or 3
§ 4 · § 4 ·
¨ ¸ ¨ ¸
So the position vectors are ¨ 7 ¸ or ¨ 3 ¸
¨ 5 ¸ ¨ 1 ¸
© ¹ © ¹

11 (a)

a h

A decorative artifact,
if as shown in the diagram above, is in the shape of a cone with radius
r and height h with a sphere of fixed radius a inscribed in it.
S a 2h2
(i) Show that the volume of the cone, V, is given by V .
3 h  2a [3]
[Turn Over
16

1 2
[Volume of cone, V Sr h ]
3
(ii) Use differentiation to find, in terms of a, the minimum value of V. Leave your answer
in exact form. [5]
(b) In a triangle PQR, PQ 3 cm and PR 2 cm. If angle QPR is increasing at a
constant rate of 0.1 radians per second, find the rate of increase of the length
π
QR at the instant when angle QPR is radians.
3 [4]

Solution
(a)

h a

r
ha a h r
(i) or
r h h  a
2 2 r 2 a
 a2
(h  a )(r ) a r 2  h2
Ÿ (h  a) 2 (r ) 2 a 2 (r 2  h 2 )
r 2 (h 2  2ah  a 2  a 2 ) a 2h2
a 2 h2 a2h
r2
h 2  2ah h  2a
1 2 1 § a2h ·
V Sr h S¨ ¸h
3 3 © h  2a ¹
S a 2 h2
V
3(h  2a)
dV S a 2 ª (h  2a )(2h)  h 2 º
(ii)
dh 3 «¬ ( h  2a ) 2 »
¼
S a 2 ª h 2  4ah º S a 2 h ª h  4a º
3 «¬ (h  2a) 2 »¼ 3 « ( h  2a ) 2 »
¬ ¼
dV
When V is minimum, 0
dh
h 0 (rej since h>0), h 4a
S a 2 ª (h  2a) (2h  4a)  2 h  4ah h  2a º
2 2
d 2V
« »
dh 2 3 « ( h  2a ) 4 »
¬ ¼
S a 2 ª« 2 h  2a ª¬(h  2a)  h  4ah º¼ º»
2 2

3 « ( h  2a ) 4 »
¬ ¼
8S a 4

3(h  2a)3
d 2V 8S a 4 Sa
Since h 4a , (h  2a)3 > 0, ? 2 !0
dh 3(h  2a) 3
3
?V is minimum at h 4a
S a 2 h2 16S a 4 8S a3
V
3(h  2a) 3(2a) 3
(b) Let QR x cm and ‘QPR T radians.
Using cosine rule,
x 2 32  22  2(3)(2) cos T
x 2 13  12 cos T
dx dT dx
2x 12sin T or 2x 12sin T
dt dt dT
π
When T , x 7
3
dx § 3· dx dx d T
2 7 12 ¨¨ ¸¸ (0.1) or u
dt © 2 ¹ d t d T dt
dx 0.3 3
| 0.196 cm/s
dt 7

12 Ben and his wife, Jerrie, rrie


iiee, in
iintend
teend
nd tto
o sa
sav
save
ve ffor
o their child’s education in the future.
or
st
(a) Jerrie decides
des tto o ssave
aav
vvee $2
$$200
200
0 onon 1 JJanuary
a 2020. On the first day of each subsequent
month sh he wi
she w ill
willll ssave
aavve $5 $$50
50 momore tthan in the previous month, so that she saves $250 on
1st February
uaarry
y 22020,
020
02 0,, $$300
30 00 oon 1st March 2020, and so on. Find the earliest date for Jerrie
[4]
to save att lleast
st $$22000
east
ea
east 220
22 00 in total.
(b) Ben decidess to put $400 into a bank account on 1st January 2020. On the first day of
each subsequent month from February 2020, he puts another $150 into the account.
The interest rate is 1% per month, such that on the last day of each month, the amount
in the account on that day is increased by 1%.

[Turn Over
18

(i) Show that the value of Ben’s account on the last day of the nth month (where
January 2020 is the 1st month, February 2020 is the 2nd month, and so on) is
15400 1.01  15150 .
n

[3]
(ii) Hence, calculate the earliest date for Ben to save at least $59900 in his bank
[3]
account.
(iii) Ben wants to save $88000 in his bank account by 2nd January 2038 for his
child’s university education. Given that the initial amount deposited on 1 st
January 2020 remains at $400 and the interest rate is 1% per month, how much
does Ben need to put into his savings account every month, from February 2020,
instead, in order to have at least $88000 in his savings account by 2 nd January
2038? [3]

Suggested Solutions
(a)

Month Amount invested in that month ($)


1 200
2 250
3 300
… …

Total amount at the end of the nth month


n
= ª¬ 2 200  n  1 50 º¼
2
= 200n  25n n  1
= 25n 2  175n
25n 2  175n t 22000
n 2  7 n  880 t 0
Consider n 2  7 n  880 0
Using GC, n 33.37 or 26.37

n d 33.37 or n t 26.37
(rejected since n t 1 )
Least n = 27
Hence, 1 March 2022 is the earliest
earli
lies
liestt date
es da for Jerrie to save at least $22000.
(b)(i)

Month Amount,
Am
A mou
ount
nt, in
in dollars
dol
olla
larrs
la Amount, in dollars
((Beginning
(BBeg
egin
in n ng ooff Mo
niing Month)
M (End of Month)
1 44000
40 400 1.01
2 4400 1.01  150 400 1.01  150 1.01
2

400 1.01  150 1.01  150 400 1.01  150 1.01  150 1.01
2 3 2
3
… … …
400 1.01 400 1.01
n 1 n

n
150 1.01 150 1.01  ...  150 1.01
n2 n1
 ...  150
th
Total amount at the end of n month
= 400 1.01  150 1.01  ...  150 1.01
n n1

= 400 1.01  150 ª1.01  1.012  ...  1.01 º


n n1
¬ ¼
ª1.01 1.01n 1  1 º
= 400 1.01  150 « »
n

«¬ 1.01  1 »¼
= 400 1.01  15000 1.01n  1.01
n

= 15400 1.01  15150


n

(b)(ii)
15400 1.01  15150 t 59900
n

1501
1.01 t
n

308
§ 1501 ·
ln ¨ ¸
308 ¹
nt ©
ln1.01
n t 159.17
When n 159,
Amount at the end of 159th month = 15400 1.01
159
 15150 | $ 59773.69
So, by the end of March 2033, Ben would have $59773.69 . Hence, on 1 April 2033, Ben
would have $59773.69  $150 = $59923.69 ! $59900
(first exceed $59900)
(b)(iii)
Let x be the monthly amount that Ben needs to deposit from February 2020 onwards.
From (bi),
Total amount at the start of the 217th month (by 2 January 2038)
ª1 1.01216  1 º
= 400 1.01  x «
216
»
«¬ 1.01  1 »¼
= 400 1.01  100 x 1.01216  1
216

400 1.01  100 x 1.01216  1 t 8


216
8000
88000
x t 111.59
o pu
d tto
eeds
Hence, Ben needs in aatt le
ut in
pput astt $1
as
lleast $111.59 per month, from February 2020 onwards.
$111

[Turn Over
ANGLO-CHINESE JUNIOR COLLEGE
JC1 PROMOTIONAL EXAMINATION
/100
Higher 2

CANDIDATE
NAME

TUTORIAL/ INDEX
FORM CLASS NUMBER

MATHEMATICS 9758/01
Paper 1 7 October 2019

3 hours

Candidates answer on the Question Paper.


Additional Materials: List of Formulae (MF26)

READ THESE INSTRUCTIONS FIRST


Question Marks
Write your index number, class and name on all the work you hand in. 1 /3
Write in dark blue or black pen.
2 /5
You may use an HB pencil for any diagrams or graphs.
Do not use staples, paper clips, glue or correction fluid. 3 /5

Answer all the questions. 4 /5


Write your answers in the spaces provided in the question paper. 5 /6
Give non-exact numerical answers correct to 3 significant figures, or 1
decimal place in the case of angles in degrees, unless a different level 6 /10
of accuracy is specified in the question.
The use of an approved graphing calculator is expected, where 7 /12
appropriate.
Unsupported answers from a graphing calculator are allowed unless a 8 /9
question specifically states otherwise. 9 /12
Where unsupported answers from a graphing calculator are not allowed
in a question, you are required to present the mathematical steps using 10 /9
mathematical notations and not calculator commands.
You are reminded of the need for clear presentation in your answers. 11 /12

The number of marks is given in brackets [ ] at the end of each question 12 /12
or part question.
The total number of marks for this paper is 100.

_________________________________________________________________________________
This document consists of 28 printed pages.

[Turn over
2

1 A banker is helping a client to invest $100,000 in stocks. He shortlisted three stocks; high
risk stock H, moderate risk stock M and low risk stock L with a projected rate of return of
12%, 8% and 4% per annum respectively.
He aimed to help his client achieve an overall projected rate of return of 10% per annum
from the $100,000 investment with a portfolio such that the amount invested in stock L is
equal to the total sum of 20% of the amount invested in stock H and 10% of the amount
invested in stock M. Assuming that all the money were invested, determine how much did
the banker invest in each of the three stocks. [3]

2 Without using a calculator, solve


6
(i) 1 t x, [3]
x
6
(ii) 1 t x. [2]
x

dm m2  m  1
3 Find the general solution of the differential equation . [5]
dv m 1

4 The sum, S n , of the first n terms of a sequence u1 , u2 , u3 , ... is given by


Sn kn 2  (5  k )n ,
where k is an unknown constant.

(i) Find u n in terms of n and k. [2]


(ii) Hence show that the sequence is an arithmetic progression. [1]

The rth term, vr , of another sequence v1 , v2 , v3 , ... , is given by vr e ur .

(iii) Show that this sequence is geometric and determine the values of k for which the
sum to infinity exists. [2]

ANGLO-CHINESE JUNIOR COLLEGE 2019 H2 MATHEMATICS 9758/01


3

5 (i) State a sequence of transformations that will transform the curve with equation
3 3
y x onto the curve with equation y 2 x  5  m , where m is a positive

constant. [3]
Functions f and h are defined by
3
f :x 2 x 5  m, x ,

h:x ln x  1 , x ,x 1,

where m is a positive constant.


(ii) Explain why the composite function hf exists. [1]
(iii) Find the domain and range of hf. [2]

y
6 (a)

The diagram shows the graph of y f( x) with asymptotes y  x  1 and x 1 . The


coordinates of the axial intercepts are  k ,0 ,
k ,0 and 0,  k , where k is a
positive constant.
On separate diagrams, sketch the graph of
(i) y f '( x) , [3]
1
(ii) y . [3]
f( x)

(b) Curve C has equation x  y  2 x  y


2 2
24 . Find the equation of a tangent to

C such that this tangent crosses the positive x- and positive y-axes and makes an
angle of 45 to the horizontal. [4]

ANGLO-CHINESE JUNIOR COLLEGE 2019 H2 MATHEMATICS 9758/01 [Turn over


4

§S ·
7 (a) Given that T is sufficiently small, show that tan ¨  T ¸ | 3  4T . [3]
© 3 ¹

(b) It is given that y sec2 x .


d2 y
(i) Show that 6 y2  4 y . [3]
dx 2
(ii) By further differentiation of this result, find the first three non-zero terms in the
Maclaurin series for sec 2 x . [4]
(iii) Hence find the first two non-zero terms in the Maclaurin series for tan 2 2x . [2]

S

8 (i) Show that ³02 e
2 x
cos 2 x dx 1  eS ¼º . [4]

S
(ii) The region bounded by the curve y e x cos x , y 1 and x is rotated through
2
2π radians about the x-axis. Using the result in (i), find the exact volume of the solid
formed. [5]

9 y

The diagram shows a curve C with parametric equations


x h cosT , y k sin T ,
where h and k are positive constants and π d T d 0 .

(i) Using the parametric equations of curve C, find the coordinates of the points where
C crosses the axes. [2]
(ii) The region enclosed by C and the x-axis is denoted by A. Find the area of A in terms
of h, k and π . [6]
(iii) Find the Cartesian equation of C, by expressing y in terms of x. [2]

(iv) A curve D has equation y  m  x 2 . State the conditions relating m, h and k such
that curve D is enclosed within region A. [2]

ANGLO-CHINESE JUNIOR COLLEGE 2019 H2 MATHEMATICS 9758/01


5

r r 1 r
§1· §1· §1·
10 (i) It is given that f (r ) r ¨ ¸ . Show that f (r )  f (r  1) ¨ ¸ r¨ ¸ . [1]
©2¹ ©2¹ ©2¹
(ii) Hence use the method of differences to show
r n 1 n
n
§1· §1· §1·
¦ r¨ ¸
r 1 ©2¹
2¨ ¸
©2¹
 n¨ ¸ .
©2¹
[4]

r
n
§1·
The function g, with domain the set of positive integers, is given by g n ¦ r¨ ¸ .
r 1 ©2¹

(iii) If n1 and n2 are two positive integers such that n2 ! n1 , by considering

§ 251 ·
g n2  g n1 , explain clearly why g has an inverse and find g 1 ¨ ¸. [4]
© 128 ¹

11 A research team would like to examine the growth of a certain bacteria in a controlled
environment. Beginning with a sample amounting to A0 of this bacteria, the

researchers noted down the amount of bacteria found in this environment at the end of
each day, such that A1 represents the amount present at the end of the first day and A2

for the second day.


(i) It was found that the amount of bacteria present at the end of k th day, denoted by
Ak , relies on the amount present at the end of the previous day, with the relationship

Ak Ak 1  360k 2 .

Use this relation to find expressions for A1 , A2 and A3 , leaving each in terms of

A0 . Hence, or otherwise, show that the amount of bacteria present at the end of n th

day, An is given by An A0  an 3  bn 2  cn , where a, b and c are constants to be


N
1
determined. [You may use the result ¦r
r 1
2

6
N N  1 2 N  1 .] [5]

ANGLO-CHINESE JUNIOR COLLEGE 2019 H2 MATHEMATICS 9758/01 [Turn over


6

The research team later discovered an antibody to curb the growth of the bacteria. With
the introduction of the antibody, the amount of bacteria present (measured in cells per
2n
ml) in the new controlled environment, P (n) is given by P (n) ¦ (450  nr ) , where
r 1

n is a positive integer that denotes the number of days from which the antibody is
administered.
(ii) (a) Find the number of days after the antibody is administered before it
effectively reduces the amount of bacteria present. [3]
(b) Calculate the total number of days required for the bacteria to be
completely wiped out in the controlled environment. [2]
Based on the chemical composition of the antibody, the team formulated a synthetic
medication. During the clinical trial where the synthetic medication is used, the amount
of bacteria present (measured in cells per ml) in the experimental controlled

environment, Q(n) can be modelled by Q(n) 1617  20 n  7 , where n is a


2

positive integer that denotes the number of days from which the medication is
administered.
(iii) Assuming the initial amount of bacteria present in both controlled environments
are the same, comment with justification whether the antibody or the synthetic
medication is more effective in reducing the amount of bacteria present. [2]

ANGLO-CHINESE JUNIOR COLLEGE 2019 H2 MATHEMATICS 9758/01


7

12 (i) A cameraman is filming a race at a family carnival. A runner runs along a straight
track at a constant speed of p m/s. X represents the location of the runner who starts
running from the starting point S.
The cameraman positions himself at a point C which is 10 m from the starting point
S. CS is perpendicular to the track as shown in the diagram below. θ is the angle
through which the cameraman has to rotate his camera to film the runner who starts
running from S. Find the rate of change of θ, in terms of p, at the instant when the
runner is 10 m from the S. X [4]
S

10
θ

(ii) Another cameraman who is also positioned at C, wishes to film two other runners,
A and B. Both runners start from S at the same time and run along the straight track.
The first runner A runs at a constant speed of 2q m/s and the second runner B runs
at a constant speed of q m/s. The angle through which the cameraman has to rotate
his camera to film the runners are and where SCB and BCA as
shown in the diagram below
below.
S B A

10
α μ

By finding an expression for tan D  P , t seconds after the race has started,
show that

§ qt · § qt ·
P tan 1 ¨ ¸  tan 1 ¨ ¸ . [2]
©5¹ © 10 ¹

Hence, use differentiation, to find the maximum value of P as the two runners run and
show that the value obtained is a maximum. [6]

ANGLO-CHINESE JUNIOR COLLEGE 2019 H2 MATHEMATICS 9758/01 [Turn over


8

BLANK PAGE

ANGLO-CHINESE JUNIOR COLLEGE 2019 H2 MATHEMATICS 9758/01


2019 ACJC H2 Math Promo P1 Marking Scheme
Qn Solutions
1 Let the amount invested in high risk stock, moderate risk stock and low
risk stock be H, L and M respectively.
H  M  L 100 000    (1)
0.2 H  0.1M  L 0    (2)
0.12 H  0.08M  0.04 L 10 000    (3)
Using G.C,

H $65 000, M $20 000, L $15 000.

2 (i)
6
1 tx , xz0
x
x6
tx
x
x6
xt0
x
 x2  x  6
t0
x
 x  3 x  2 t 0
x
 x  3 x  2 x t 0
x d 2 or 0  x d 3

(ii)
Replace x with x ,
x d 2 or 0 x d3
(Reject as x >0 ) x d3
 3 d x d 3, x z 0.
3 dm m2  m  1
dv m 1
m 1
³ m 2  m  1 dx ³ 1 d m
1 1
2m  1 
2 2 dx
2
m  m 1 ³ ³ 1 dm
1 2m  1 1 1
2
2 m  m 1 ³
dx  2
2 m  m 1 ³ ³ 1 dm
1 2m  1 1 1

2 m  m 1
dx 
2 § 3· § 2

³2 ³ 1 dm
¨ ¸ ¨ m  ¸
© 4¹ © 2¹
§ · § 1·
¨ ¸ ¨ m ¸
1
2

ln m 2  m  1  ¨ ¸ ¨
2
© ¹ ¨ 3

§ 1 · 1 ¸ 1 ¨
¸
tan
¨ 3

¸
mC
¨ ¸ ¨ ¸
© 4¹ © 4 ¹
§ 1 · 1 § 2m  1 ·
1
2

ln m 2  m  1  ¨
© 3¹
¸ tan ¨
© 3 ¹
¸ m  C.

4 (i)
Sn kn 2  (5  k )n
k n  1  (5  k ) n  1
2
Sn 1
kn 2  2kn  k  5n  5  kn  k
kn 2  3kn  2k  5n  5

un Sn  Sn 1
kn 2  5n  kn  kn 2  3kn  5n  5  2k
2kn  2k  5

(ii)
un 1 2k n  1  2k  5
un  un 1 2kn  2k  5  2kkn  2k  2k  5 2k
Since un  un1 2k w which
whhiicch is
is a constant,
con
onst
stan
st an
nt,
t hence the sequence is an
arithmetic progression.
rogr
grressssiion
g on.
n.

(iii)
Given vr e ur ,
vr 1 e ur 1
vr e ur
vr 1 e ur 1
e ur ur 1
e 2k
vr
Since e 2 k which is a constant, this sequence is geometric.
vr 1

For sum to infinity to exist,


e 2k  1
e 2k  1 .
2k ln e  ln1
k  0.

5 (i)
3
y x
p Translation of 5 units in the positive x-direction.
3
y x 5
p Scaling by a factor of 2 parallel to the y-axis.
3
y 2 x 5
p Translation of m units in the positive y-direction.
3
y 2 x 5  m
(ii)
R f [m, f), D h 1, f
Since m ! 0 hence R f Ž Dh .
? hf exists.

(iii)
Dhf Df
To find the range of hf , we use the R f as the restricted domain of h
nge from the graph of y h( x) .
and read off the corresponding range
R hf [ln m 1 , ).

6 (a)
(i)
x=1
y
x
(ii) y

(b)
x  y  2 x  y
2 2
24    (1)
§ dy · § dy ·
2 x  y ¨1  ¸  4 x  y ¨1  ¸ 0
© dx ¹ © dx ¹

x  y ¨§1  ¸·  2 x  y ¨§1  ¸· 0    (2)


dy dy
© dx ¹ © dx ¹
Since the tangent crosses the positive x- and positive y-axes and makes
dy
an angle of 45 to the horizontal,  tan 45 1 .
dx
dy
Sub. 1 into eqn. (2),
dx
x  y 1  1  2 x  y 1  1 0
x y    (3)
Sub. (3) into (1),
x  x  2 x  x
2 2
24
2x
2
24
x r 6
Since x ! 0, ? x 6 and y 6.
Hence equation of tangent:
y 6 1 x  6
y  x  2 6.
7 (a)
S
tan
tta
an an T
 ttan
§S · 3
tan ¨  T ¸
©3 ¹ S
1  tan
t tan T
3
3  tan T
1  3 tan T

1
| 3  T 1  3T
|
3  T 1  1  3T  ...
| 3  3T  T 3  4T .
(b)(i)
y sec2 x
dy
2 sec x sec x tan x
dx
2sec2 x tan x
2 y tan x
d y 2
§ § dy · ·
2 ¨ y ˜ sec 2 x  2 tan x ¨ ¸¸
dx 2 © © dx ¹ ¹
2 y 2  2 tan x 2 y tan x
2 y 2  4 y tan 2 x
2 y 2  4 y sec 2 x  1
2 y 2  4 y y  1
6 y 2  4 y .(Shown)

(ii)
d3 y § dy · dy
6¨ 2y ¸  4
dx3 © dx ¹ dx
dy dy
12 y  4
dx dx
4
d y § d y dy dy ·
2
d2 y
12 ¨ y 2  ˜ ¸  4 2
dx 4 © dx dx dx ¹ dx
2
d2 y § dy · d2 y
12 y  12 ¨ ¸  4
dx 2 © dx ¹ dx 2
dy d2 y d3 y d4 y
When x 0 , y 1, 0, 2, 0, 16
dx dx 2 dx3 dx 4
2 16
sec 2 x 1  x 2  x 4  ...
2! 4!
2
| 1  x2  x4
3
(iii)
2
sec 2 2 x | 1  2 x  2 x
2 4

3
32
1  4x
4 x2  x4
3
Since tan 2 2 x sec 2 2 x  1
32 4
tan 2 2 x | 1  4 x 2  x 1
3
32 4
4x2  x
3
8 (i) Method 1:
S
2
2 x
³e cos 2 x dx
0
S S
2
ª § 1 2 x · º 2 § 1 2 x ·
« cos 2 x ¨  2 e ¸»  ³ ¨  e ¸ 2sin 2 x dx
¬ © ¹¼0 0 © 2 ¹
S
2
ª § 1 S · § 1 0 ·º § 1 2 x ·
« cos S ¨  2 e ¸  cos 0 ¨  e ¸ » ³ ¨  e ¸ 2sin 2 x dx

¬ © ¹ © 2 ¹¼ 0 © 2 ¹
S
2
ª 1 S 1 º
e 2 x sin 2 x dx    (1)
2 »¼ ³0
«¬ 2 e  

S
2
Consider ³ e 2 x sin 2 x dx
0
S S
2
ª § 1 2 x · º 2 § 1 2 x ·
« sin 2 x ¨  e ¸»  ³ ¨  e ¸ 2 cos 2 x dx
¬ © 2 ¹¼0 0 © 2 ¹
S
2
ª § 1 S · § 1 0 ·º
« sin S ¨  2 e 2 x
¸  sin 0 ¨  e ¸ » ³ e cos 2 x dx

¬ © ¹ © 2 ¹¼ 0
S
2
0  ³ e 2 x cos 2 x dx    (2)
0

Sub. (2) into (1).


S S
2 2
ª 1 S 1 º
³ e cos 2 x dx
2 x
«¬ 2 e  » 
2¼ 0 ³ e2 x cos 2 x dx
0
S
2
1 S 1
2 ³ e 2 x coss 2 x dx e 
0
2 2
S
2
1 ª S º
³e
2 x
cos 2 x dx e  1¼ (Shown)
0

(i) Method 2:

S
2
2 x
³e cos 2 x dx
0
S S
8 2
ª 2 x § 1 ·º 2
«
¬

e ¨
© 2

sin 2 x ¸»
¹

¼0 0
³
§1
© 2
·
¨ sin 2 x ¸ 2e
¹
2 x

dx
S
2
ª S § 1 ·º
«
¬

e ¨
© 2
sin S
·
¸
¹
 e 0 §1
¨
© 2
sin
0
¹¼ 0 ©
§1
2
·
¸ » ³ ¨ sin 2 x ¸ 2e

¹
2 x
dx
S
2
0  ³ e 2 x sin 2 x dx    (1)
0
S
2
Consider ³ e 2 x sin 2 x dx
0
S S
2
ª 2 x § 1 ·º 2

« e
¬
§ 1 ·
¨  cos 2 x ¸ »  ³ ¨  cos 2 x ¸ 2e
© 2 ¹¼0 0 © 2 ¹
2 x
dx
S
2
ª S § 1 ·º
«
¬

e ¨ 
© 2
cos S
·
¸
¹
 e 0 § 1
¨ 
© 2
cos 0

¹¼ 0
2 x
¸ » ³ e cos 2 x dx
S
2
ª 1 S º 2 x
«¬ 2 e  1»¼  ³ e cos 2 x dx    (2)
0

Sub. (2) into (1).


S ª S º
2 « ª 1  S º 2 2 x »
³e
2 x
cos 2 x dx 0  « « e  1»  ³ e cos 2 x dx »
0 «¬ 2 ¼ 0 »
¬« ¼»
S
2
1 S 1
2 ³ e 2 x cos 2 x dx e 
0
2 2
S
2
2 x 1 ª S º
³e cos 2 x dx

e  1¼ ((Shown)
Sh
how
o
y
0

(ii) y=1

x
S

§S ·
2

Required Volume S 12 ¨ ¸  S ³ y 2 dx
©2¹ 0
S
S2 2

2
 S ³ e  x cos x dx
8 2 0
S
2
S 2
 S ³ e2 x cos 2 x dx
2 0
S
S2 2
§ 1  cos 2 x ·
 S ³ e2 x ¨ ¸ dx
2 0 © 2 ¹
S
S2 S 2

2

2 ³e
2 x

 e 2 x cos 2 x dx
0
S S
2
S S 2
2 x S 2
2 x
2³ 2³
 e dx  e cos 2 x dx
2 0 0
S S
S2 S ª e2 x º 2 S 2
e 2 x cos 2 x dx
2 ¬ 2 ¼ 0 2 ³0
 « » 
2
S
2 S
S S ª 2 x º 2 S 2 2 x
2

4 ¬e ¼  ³ e cos 2 x dx
0 2 0
S
S2 S S 2
 ª¬eS  1º¼  ³ e 2 x cos 2 x dx
2 4 20
S2 S ª S º S § 1 ª ·
 ¬ e  1¼  ¨ ¬1  e S º¼ ¸
2 4 2©4 ¹
S2 S S S S
 e S    e S
2 4 4 8 8
S2 S 3S
 e S 
2 8 8

4S  e S  3º¼ units3 .

9 (i)
Finding x-intercepts:
Using y k sin T , when y 0,
k sin T 0
T 0, S (sincee  π d T d 0)
0)
WhenT 0 0, x h ccos
oss 0 h
o
WhenT S , x cos S
co
h cos h
rce
cept
ceptss ar
Hence x -intercepts are h, 0 and h, 0 .
Finding y-intercepts:
Using x h cosT , when x 0,
h cos T 0
S
T  (since  π d T d 0)
2
S § S·
WhenT  k sin ¨  ¸
,y k
2 © 2¹
Hence y -intercept is 0,  k .

(ii)
x h cos T S
dx
When x 0, cos T 0 ŸT π d T d 0
2
 h sin T
dT When x h, cos T 1Ÿ T 0 π d T d 0
h
A 2³ y dx
0
0
2³S k sin T  h sin T dT
2
0
2kh ³ S sin T sin T dT
2

sin T dT
0
2kh ³ S
2

2
0 § 1  cos 2T ·
2kh ³S ¨ ¸ dT
2 © 2 ¹
0
kh ³S 1  cos 2T dT
2
0
ª sin 2T º
=kh «T 
¬ 2 »¼ S
2

ª§ sin 0 · § S sin S · º khS


kh «¨ 0  ¸¨  ¸» units 2 .
«¬© 2 ¹ © 2 2 ¹ »¼ 2

(iii)
x h cos T y k sin T
x y
cos T    (1) in T
ssin    (2)
h k
(1)2  (2)2 :
x2 y 2
cos 2 T  sin
n2 T
in 
h2 k 2
x2 y 2
1 
h2 k 2
x2 y 2
 1
h2 k 2
§ x2 ·
y2 k 2 ¨¨1  2 ¸¸
© h ¹
§ x2 ·
y r k 2 ¨¨1  2 ¸¸
© h ¹
x2
y k 1  (since k >0 and from diagram y d 0)
h2
(iv)
y  m  x2

m
2
y 2  x2

Since y  0 , curve D is a semicircle with centre (0,0) and radius m,


lying below the x-axis.

If h ! k , mdk.
If h  k , m dh.

10 (i)
r r 1
§1· §1·
f (r )  f (r  1) r ¨ ¸  r  1 ¨ ¸
©2¹ ©2¹
r r 1 r 1
§1· §1· §1·
r¨ ¸ r¨ ¸ ¨ ¸
©2¹ ©2¹ ©2¹
r r 1
§1· §1·
¨ ¸ > r  2r @  ¨ ¸
©2¹ ©2¹
r 1 r
§1· §1·
¨ ¸ r¨ ¸ (Shown)
©2¹ ©2¹
(ii)
r 1 r
§1· §1·
f (r )  f (r  1) ¨ ¸ r¨ ¸
©2¹ ©2¹
n ª§ 1 · r 1
n
§1· º
r

¦ >f (r )  f (r  1))@ ¦ «¨ ¸  r ¨ ¸ »
r 1 ¬© 2 ¹
r 1 « © 2 ¹ »¼
n
Consider ¦ >f (r )  f (r  1)@
r 1
f (1)  f(0) 

f (2)  f(1) 
f (3)  f(2)  ...
f (2)  f(1) 
f (n  1)  f(n - 2) 
f (n)  f(n -1)
f (n)  f(0)
n
§1·
n¨ ¸  0
©2¹
n ª
§1· º
r 1 r
n
§1·
¦ >f (r )  f (r  1)@ ¦ «¨ ¸  r ¨ ¸ »
r 1 «© 2 ¹
r 1¬ © 2 ¹ ¼»

§1·
n n ª§ 1 · r 1 º n ª § 1 · r º
n¨ ¸  0
©2¹
¦ «¨ ¸ »  ¦ « r ¨ ¸ »
¬© 2 ¹ »¼ r 1 «¬ © 2 ¹ »¼
r 1 «

n
§1·
r n ª§ 1 · r 1 º §1·
n

¦ r¨ ¸
r 1 ©2¹
¦ «¨ ¸ »  n ¨ ¸
«© 2 ¹ ¼»
r 1¬ ©2¹
ª 1 1  0.5n º §1·
n

« »  n¨ ¸
«¬ 1  0.5 »¼ ©2¹
n
§1·
2 1  0.5  n ¨ ¸
n

©2¹
n 1 n
§1· §1·
2¨ ¸  n ¨ ¸ (Shown)
©2¹ ©2¹
(iii)

r r
§1· §1·
n2 n1
g n2  g n1 ¦ r¨ ¸ ¦r¨ ¸
r 1 ©2¹ r 1 ©2¹

Since n2 ! n1 ,
r r r
§1· §1· §1·
n2 1 n n2
g n2  g n1 ¦ r ¨ ¸ ¦r¨ ¸
r 1 ©2¹

r 1 ©2¹
¦ r¨ ¸ ! 0
r n1 1 © 2 ¹

? g n2 ! g n1 .
Hence g iss a one
one to
to one function as g n2 ! g n1 .
one function
? g 1 exists.
s.
r
n
§1·
g n ¦ r¨ ¸
r 1 ©2¹
n 1 n
§1· §1·
g n 2¨ ¸  n¨ ¸
©2¹ ©2¹
§ §1·
n 1
§1· ·
n

n g 1 ¨ 2  ¨ ¸  n ¨ ¸ ¸
¨ ©2¹ © 2 ¹ ¸¹
©

§ 251 ·
To find g 1 ¨ ¸,
© 128 ¹
n 1 n
§1· §1· 251
2 ¨ ¸  n¨ ¸
©2¹ ©2¹ 128
Using GC,
n 8.

11 (i)
Using Ak Ak 1  360k 2 ,
A0  360 1
2
A1 A0  360
A1  360 2 A0  360 1  360 2
2 2 2
A2 A0  1800
A2  360 3 A0  360 1  360 2  360 3
2 2 2 2
A3 A0  5040

A0  360 1  360 2  360 3  ...  360 n


2 2 2 2
An

A0  360 ª 1  2  3  ...  n º
2 2 2 2
¬ ¼
ª n
º
A0  360 « ¦ r 2 »
¬r 1 ¼
ªn º
A0  360 « n  1 2n  1 »
¬6 ¼
A0  60n ª¬ 2n 2  3n  1º¼
A0  120n3  180n 2  60n.
a 120, b 180, c 60.

11 (ii)(a)
2n
P ( n) ¦ (450
(450  nr )
(4
r 1
2n 2n

¦ (450)
r 1
0)  n ¦ ( r )
0)
r 1

450 2n  n ª« 1  2n º»
2n
¬2 ¼
900n  n 2  2n3 .
P(n) 900n  n2  2n3
Using G.C to find maximum point of graph y P(n) ,
n 12.08 (since n>0)
Number of days the amount of bacteria present reduces is 13.
Key into G.C,
Alternative Method (Using G.C):
2X
n 11, P(11) 7117
Y1 ¦ 450-XA and press
n 12, P(12) 7200 A 1
nd
n 12.5, P(12) 7187.5 [2 ][graph] to look at table
generated.
n 13, P(13) 7137
From G.C, it can be observed that the bacteria start to reduce between
the 12th and 13th day.
Since n is a positive integer, the number of days the amount of bacteria
present reduces is 13.

(ii)(b)
P ( n) 0
900n  n 2  2n3 0
Using G.C to solve 900n  n 2  2n3 0 ,
n 20.96 (since n>0)
Number of days the amount of bacteria to be wiped out is 21.

(iii)
Q(n) 1617  20 n  7
2

The maximum point of graph y Q ( n) occurs when n 7 .


Since it only takes 7 days for the amount of bacteria present to be
reduced when using the synthetic medication as compared to the 13
days required when using the antibody, hence the synthetic medication
is more effective in reducing the amount of bacteria present.

12 (i)
Let x be the distance run in t seconds
dx
p
dt
x
tan T
10
dT 1 dx
sec 2 T
dt 10 dt
10
when x 10, tanT 1
10
S
T
4
2
dT 1 p p§ 1 · p
radians / s
dt 10 §S · 10 ¨© 2 ¸¹ 20
sec 2 ¨ ¸
©4¹
(ii)
2qt qt
tan D  P
10 5
qt
tan D
10
§ qt ·
D P tan 1 ¨ ¸
©5¹
§ qt · § qt ·
P tan 1 ¨ ¸  tan 1 ¨ ¸
©5¹ © 10 ¹
(iii)
q q
dP 5
Let 2
 10 2 0
dt § qt · § qt ·
1 ¨ ¸ 1 ¨ ¸
©5¹ © 10 ¹
q § § qt ·

q § § qt · ·
2
¨1  ¨ ¸ ¸  ¨ 1  ¸ 0
5 ¨© © 10 ¹ ¸ 10 ¨ ¨© 5 ¸¹ ¸
¹ © ¹
q q3 2 q q3 2
 t   t 0
5 500 10 250
q q3 2
t
10 500
50 5 2
t
q2 q
500 § q 50 · 1 § q 50 ·
when t ,P tan 1 ¨¨ . ¸¸  tan ¨¨ . ¸¸
q © 5 q ¹ © 10 q ¹
2
tan 1 2  tan 1 0.340radians
2
2q 3t 2q 3t
2  
d P 125 1000

dt 2 § § qt · 2 ·2 § § qt ·2 ·
2

¨1  ¨ ¸ ¸ ¨1  ¨ ¸ ¸
¨ ©5¹ ¸ ¨ © 10 ¹ ¸
© ¹ © ¹
50
when t 2
q2
2q 3 § 50 · 2q 3 § 50 ·
 ¨ ¸  ¨ ¸
d 2P 125 © q ¹ 1000 © q ¹

dt 2 1  2 2 § 1·
1 
2

¨ ¸
© 2¹
§ 1 4 · 50 2
2q 2 50 ¨  ¸  q 0
© 125(9) 1000(9) ¹ 1125
Hence value obtained is a maximum.
CATHOLIC JUNIOR COLLEGE
General Certificate of Education Advanced Level
Higher 2
JC1 Promotional Examination
CANDIDATE
NAME
INDEX
CLASS
NUMBER

MATHEMATICS 9758/01
Paper 1 07 October 2019
3 hours
Candidates answer on the Question Paper.

Additional Materials: List of Formulae (MF26)

READ THESE INSTRUCTIONS FIRST

Write your name and class on all the work you hand in.
Write in dark blue or black pen.
You may use an HB pencil for any diagrams or graphs.
Do not use staples, paper clips, highlighters, glue or correction fluid.

Answer all the questions.


Write your answers in the spaces provided in the question paper.
Give non-exact numerical answers correct to 3 significant figures, or 1 decimal place in the case of angles
in degrees, unless a different level of accuracy is specified in the question.
The use of an approved graphing calculator is expected, where appropriate.
Unsupported answers from a graphing calculator are allowed unless a question specifically states
otherwise.
Where unsupported answers from a graphing calculator are not allowed in a question, you are required to
present the mathematical steps using mathematical notations and not calculator commands.
You are reminded of the need for clear presentation in your answers.

The number of marks is given in brackets [ ] at the end of each question or part question.
The total number of marks for this paper is 100.

Question 1 2 3 4 5 6 7 8 9 10 11 12 Total

Marks
Total 3 6 7 7 7 7 8 10 9 10 12 14 100

This document consists of 28 printed pages and 2 blank pages.

9758/01/Promos/2019 [Turn over


2

1 y

The diagram shows the graph of y f ( x) . The graph crosses the x-axis at x = 2, and the
graph has a minimum point 0,8 and a maximum point 3, 5 . The lines x 1 , x 1
and y x are the asymptotes of the graph.

Sketch the graph of y f ' x , indicating clearly the equations of any asymptotes and the
coordinates of any points of intersection with the x- and y-axes. [3]

ax 2  bx  a
2 The curve C has an equation y where a and b are constants.
x2  b
(i) Given that two of the asymptotes of C are x 2 and y 3 , determine the values of a
[2]
and b.

(ii) Sketch the curve C, giving the coordinates of the point(s) where the curve crosses the
axes, stationary points and the equations of all asymptotes. [4]

9758/01/Promos/2019
3

3 The function f is defined by


f:x x2  4 x  5
5,, x  , x d 2 .
(i) Show that f has an inverse. [2]

The function g is defined by


g:x e x  1, x  , x d 0 .
(ii) Determine whether the composite function gf exists. [2]

(iii) It is known that fg exists. Find an expression for fg(x). Hence or otherwise, find the
range of fg. [3]

4 dy
Find for each of the following, simplifying your answers.
dx
(a) y cos1 1  x2 , 0  x  1 [3]

(b) x 2  xy  y 2 sin x [4]

5 P S

1 cm

Q R

A regular trapezium PQRS with height 1 cm is such that PQ  QR  RS 4 cm . PQ and RS are


π π
each inclined to the line QR at an angle T , where  T  , as shown in the diagram.
6 2
(i) By considering the length of QR, show that the area, A cm 2 , of the trapezium PQRS
is 4  2 cosecT  cot T . [3]

(ii) Hence, find the maximum area of the trapezium PQRS, giving your answer in the form
M  N 3 , where M and N are integers. [4]

9758/01/Promos/2019 [Turn over


4

1
6 It is given that f x .
3
8  3x
(i) Find the binomial expansion for f x , up to and including the term in x 2 . Give the
coefficients as exact fractions in their simplest form.
State the range of values of x for which the expansion is valid. [4]

1
(ii) By putting x into the expansion found in part (i), find an approximate value of
16
3 a
16 . Leave your answer in the form of in its lowest term, where a and b are
b
positive integers to be determined. [3]

7 (a) Without using a calculator, solve the inequality


x 2  3x  5
d 1.
x2 [4]

(b) Sketch the graphs of y x  3 and y ln x  2 on the same diagram, indicating


clearly the coordinates of any points of intersection with the axes, the coordinates of
the points of intersection of the graphs and equations of any asymptotes.

Hence solve
ln x  2 d x  3. [4]

n
8 (i) Find ¦4
r 1
2 r
in terms of n. [2]

2 n
1
(ii) Express
2r  1 2r  3
in partial fractions. Hence find ¦ 2r  1 2r  3
r 1
in terms

of n. [5]

n ª 2( r 1) 1 º 41
(iii) Deduce that ¦ «
r 1¬
4  »
2r  1 2r  3 ¼ 240
. [3]

9758/01/Promos/2019
5

9 Planes p1 and p2 are respectively represented by cartesian equations


x  2 y  2z 3,
2 x  y  2 z 6 ,
and they meet in a line l.

(i) Find a vector equation for line l. [2]

Plane p3 is represented by cartesian equation


M ( x  2 y  2 z  3)  (2 x  y  2 z  6) 0 , where M is a constant.

(ii) Write down a normal vector to plane p3 , in terms of M. [1]

(iii) Explain why plane p3 contains line l, for any value of M. [2]

(iv) Show that the cosine of the acute angle between planes p1 and p3 is
M
, if M > 0.
M 2 1 [4]

§0· §D ·
10 Relative to the origin O, the position vectors of the points A and B are ¨ 1 ¸ and ¨ 1 ¸ , D  0
¨ ¸ ¨ ¸
¨ 2 ¸ ¨ 1¸
© ¹ © ¹
respectively.

1
(i) Show that area of triangle OAB = 1  5D 2 . [2]
2

(ii) The point C lies on AB produced such that 3 AB BC , and that OB is perpendicular
to OC. Find the value of D . [3]

(iii) Using your answers in parts (i) and (ii), deduce the exact perpendicular distance from
B to OA. Hence find ON , where N is the foot of perpendicular from B to OA. [5]

9758/01/Promos/2019 [Turn over


6

11 A curve C has parametric equations


1 2
x , y e t , where t ! 1 .
t 1

(i) Sketch C, stating the equations of asymptotes and coordinates of any turning points. [3]

dy
t  1
2 3
(ii) Show that  4tet . [2]
dx

5
(iii) The normal at t meets the y-axis at the point P. Given that the point Q on the
4
curve C has a parameter q, find the value of q such that the length PQ is a minimum.
Find the minimum length PQ. [7]

12 (a) To purchase a grand piano, Ken took a loan of $12,000 from a finance company on 1st October
2019. The company charges a compound interest at a rate of 0.5% on the outstanding amount
owed on the last day of every month, while Ken decides to repay the loan by paying back a
fixed instalment of $P on the first day of each month starting from 1st November 2019.
(i) How much would Ken owe after paying the second instalment on 1st December
[2]
2019?

(ii) Show that the amount Ken owes after paying the n th instalment is

$ ª¬12000(1.005n )  200P(1.005n  1) º¼ . [3]

(iii) Ken intends to repay the loan completely by 1 st December 2021.


Find the minimum monthly instalment that Ken needs to pay, giving your
answer to the nearest cent. [2]

9758/01/Promos/2019
7

(b) The fundamental frequencies of sound tones produced by successive keys on a piano follow
a geometric progression. On a modern 88-key piano, the 49th key produces a standardised
frequency of 440 hertz (cycles per second). The 61st key produces a frequency of 880 hertz.
(i) Determine the frequency produced by the first key on the piano. [2]

(ii) Hence, show that the frequency produced by the nth key on the piano is given
n 1
by un k (2 m ) hertz, where k and m are constants to be found.

Explain why the logarithm of the frequencies produced by successive piano keys
follow an arithmetic progression. [3]

In music, a chord is a harmonic set of tones produced by three or more keys.


The major triad chord is produced by the nth, ( n  4 )th and ( n  7 )th keys sounded
together.
(iii) Show that the ratio of ascending frequencies produced in a major triad chord, is
independent of n. [2]

9758/01/Promos/2019 [Turn over


1

2019 JC1 H2 Promotional Examination Suggested Mark Scheme

1
y

x
2

2 a(i) x2  b x  b x  b
So, x b and x  b are asymptotes.
Hence, b 2 Ÿ b 4.

ax 2  bx  a bx  ab  a
y a
x b
2
x2  b
So, y a is an asymptote.
Hence, a 3 .
6m (ii)
x 2 y x 2

y 3

(0.578, 0.461) 3 x
(0,  )
4
3

3 (i) y
7m

Since any horizontal line intersects the graph of f at most once, f is one-one and it
has an inverse. (shown)
(ii) Rf [9, f)

Since R f [9, f) ‫ م‬Dg, therefore gf does not exists.

(iii) fg : x (e x  1) 2  4(e x  1)  5
e 2 x  2e x  8, x  , x d 0
y

y=–8
(0, – 9)

R fg [9, 8)
4

4 dy 1 1 1
 ˜ ˜ 2 x
1  x 2 2

dx
2
2
1 1  x2

1 1
 ˜ ˜ 2 x
(a) x 2
2 1  x2
1 1
˜ ˜x x ! 0
x 1  x2
1
1  x2
(b) d 2
5
dx
x  xy  y 2 ddx sin x
dy dy
2 x  x ˜  y 1  2 y cos x
dx dx
dy
x  2 y cos x  2 x  y
dx
dy cos x  2 x  y
dx x  2y
5

5 (i) 1
sin T Ÿ PQ cosecT
PQ
QR 4  PQ  RS 4  2cosecT
PS QR  2 cot T 4  2 cosecT  2 cot T
1
A PS  QR 1
2
1
4  2 cosecT  2 cot T  4  2 cosecT
2
4  2 cosecT  cot T shown
7m (ii) dA
2 cosecT cot T  cosec 2T
dT
dA
For maximum area, 0.
dT
2 cosecT cot T  cosec 2T 0
2 cosecT cosecT cos T  cosec 2T 0
cosec 2T 2 cos T  1 0
2 cos T  1 0 or cosecT 0 No solution
1
cos T
2
S
T
3

 
§S · S §S ·
¨ ¸ ¨ ¸
T ©3¹ 3 ©3¹

dA
!0 0 0
dT

Hence, A is a maximum.
mum
m.
S S
Maximum Areaa 4  2 cosec
coossec  cot
c t
co
3 3
4 1
4 
3 3
3
4
3
4 3
6

6 (i) 1
8  3x

3

1
1 
§ 3 ·
 3
8 ¨1  x ¸
3

© 8 ¹
1

1§ 3 · 3
¨1  x ¸
2© 8 ¹
§ § 1 ·§ 4 · ·
¨ ¨  ¸¨  ¸ 2 ¸
¨ 1  ¨§  ¸· ¨§  x ¸·  © 3 ¹ © 3 ¹ ¨§  x ¸·  ... ¸
1 1 3 3
2 ¨ © 3 ¹© 8 ¹ 2 © 8 ¹ ¸
¨ ¸
© ¹
1§ 1 2 2 ·
¨ 1  x  x  ... ¸
2© 8 64 ¹
1 1 1 2
 x x  ...
2 16 64
3
 x 1
8
8 8
œ x
3 3
Method 2:
1
8  3x

f ( x) 3

4
8  3x

f ' ( x) 3

7
4 8  3x

f '' ( x) 3

1
f (0)
2
1
f ' (0)
16
1
f '' (0)
32

1
1 1 1 2
8  3x

3  x x  ..
....
16
2 1 6 64
64

3 8 8
 x 1œ   x 
8 3 3
7

7m (ii) 1 1 1§1· 1 §1·


2

 ¨ ¸  ¨ ¸  ...
§1· 2 16 © 16 ¹ 64 © 16 ¹
3 8  3¨ ¸
© 16 ¹
1 1 1 1
   ...
125 2 256 16384
3
16
3
16 8192  64  1
|
5 16384
41285
3
16 |
16384
8

7 (a) x 2  3x  5
d1
8m x2
x 2  3x  5
1 d 0
x2
x 2  3x  5  x  2
d0
x2
x2  2 x  3
d0
x2
x  3 x  1 d 0
x2

Hence, x d 3 or 1 d x  2 .
Method 2:

x 2  3x  5
d1
x2
x  3x  5 x  2 d x  2
2 2

x  3x  5 x  2  x  2 d 0
2 2

x  2 x  1 x  3 d 0

3 1 2
x d 3 or 1 d x  2 .
(b)

Hence, 2  x d 1.99 or x t 4.94 .


9

8 (i) 1§ § 1 · ·
n


¨ ¨ ¸ ¸
1
n 42 ¨© © 42 ¹ ¸¹
¦ 4 2 r
1
r 1
1 2
4
1§ 1 ·
¨1  4 n ¸
15 © 2 ¹
10m (ii) 2 A B

2r  1 2r  3 2r  1 2r  3
1 2
when r  ,A 1
2 § 1·
2¨  ¸  3
© 2¹
3 2
when r  , B 1
2 § 3·
2¨  ¸ 1
© 2¹
2 1 1

2r  1 2r  3 2r  1 2r  3
n
1 1 n 2
¦ 2r  1 2r  3
r 1
¦
2 r 1 2r  1 2r  3
1 n § 1 1 ·
¦ ¨  ¸
2 r 1 © 2r  1 2r  3 ¹
1 ª§ 1 1 ·
¨  ¸
2 «¬© 3 5 ¹
§1 1·
¨  ¸
©5 7¹
...
§ 1 1 ·
¨  ¸
© 2n  1 2n  1 ¹
§ 1 1 ·º
¨  ¸»
© 2n  1 2n  3 ¹ ¼
1 ª1 1 º

2 ¬ 3 2n  3 ¼»
«
1 1

6 2 2n  3
10

(iii) n ª 1 º
¦ «4
r 1¬
2( r 1)
 »
2r  1 2r  3 ¼
n n
1
¦
r 1
ª
¬ 4 2( r 1)
¼ ¦ 2r  1 2r  3
º 
r 1
n
1 1
¦ ª¬4
r 1
2 2 r
4 º¼  
6 2 2n  3
1 n 2 r 1 1
¦
16 r 1
4  
6 2 2n  3
1 1§ 1 · 1 1
¨1  4 n ¸  
16 15 © 2 ¹ 6 2 2n  3
1 1 1 1
=   4n 
16(15) 6 2 2 2n  3
41 1 1
 (deduced) since ! 0 and !0
240 2 4n
2 2n  3
11

9 (i) Solving the cartesian equations representing planes S 1 and S 2 (on the GC)
produces

­ x 3  2O § x · § 3· § 2·
° ¨ ¸ ¨ ¸ ¨ ¸
® y 2O Ÿ ¨ y ¸ ¨ 0 ¸  O ¨ 2 ¸
°z O ¨ ¸ ¨ ¸ ¨ ¸
¯ © z ¹ ©0¹ ©1¹
§ 3· § 2·
¨ ¸ ¨ ¸
Ÿ l:r ¨ 0 ¸  O ¨ 2 ¸ , O 
¨0¸ ¨1¸
© ¹ © ¹

(ii) p3 : M ( x  2 y  2 z  3)  (2 x  y  2 z  6) 0
( M  2) x  (2M  1) y  (2M  2) z 3M  6
§ M 2 · § M 2 ·
¨ ¸ ¨ ¸
r ˜ ¨ 2M  1 ¸ 3M  6 , A normal vector to plane p3 is ¨ 2 M  1 ¸ .
¨ 2M  2 ¸ ¨ 2M  2 ¸
© ¹ © ¹

9m (iii) (Elegant) method :


Let ( x, y , z ) be a point on line l.
Line l is the common line of intersection between planes p1 and p2 , ( x, y , z )
is on plane p1 as well as plane p2 .
? x  2 y  2z 3,
2 x  y  2 z 6 .

The expression
M ( x  2 y  2 z  3)  (2 x  y  2 z  6)
M ( 3  3)  (  6  6)
M (0)  (0)
0
? ( x, y, z ) lies on plane p3 .
pla p3 , ? line l lies on plane p3 .
Any point on line l lies on plane

Alternative standard
dardd method
meth
methoodd :
§ 2·
¨ ¸
Direction
io
on vve
vector
eccttor
o ffor ne ll,, m ¨ 2 ¸ .
oorr lline
in
¨1¸
© ¹
§ M 2 ·
¨ ¸
Normal vector for plane p3 , n3 ¨ 2M  1 ¸ .
¨ 2M  2 ¸
© ¹
12

§ 2 · § M 2 ·
¨ ¸ ¨ ¸
m ˜ n3 ¨ 22 ¸ ˜ ¨ 2M  1 ¸
¨ 1 ¸ ¨ 2M  2 ¸
© ¹ © ¹
2( M  2)  2(2 M  1)  (2 M  2) 0 ,
line l is parallel to plane p3 .
Point (3, 0, 0), a point on line l.
The expression
M 3  2(0)  2(0)  3  2(3)  0  2(0)  6
M (3  3)  (6  6) .

M (0)  0 0,
this point is also on plane p3 .

Line l // plane p3 , and


l contains a point (3, 0, 0) that’s also on p3 ,
? line l lies on plane p3 .

§1·
(iv) Normal vector for plane S , n ¨ 2 ¸ .
1 1 ¨ ¸
¨ 2¸
© ¹
§ M 2 ·
¨ ¸
Normal vector for plane S 3 , n3 ¨ 2M  1 ¸
¨ 2M  2 ¸
© ¹
Let T be the angle formed between normal vectors n1 and n3 .
n1 ˜ n3
? cosT
n1 n3

§1· § M  2 ·
¨ ¸ ¨ ¸
n1 ˜ n3 ¨ 2 ¸ ˜ ¨ 2M  1 ¸
¨ 2 ¸ ¨ 2M  2 ¸
© ¹ © ¹
( M  2)  2(2 M  1)  2(2
2 M  2)
2(
9M
n1 12  22  22 3
n3 2))2  ((2
(M  2 1)) 2  (2
2M  1 (2 M  2) 2

( M 2  4 M  4)
4)  (4 M 2  4 M  1)  (4 M 2  8M  4)
9M 2  9
3 M 2 1
13

n1 ˜ n3
cos T
n1 n3
9M M
!0 if M ! 0

3 3 M 2 1 M 2 1

M
? The cosine of the acute angle between planes p1 and p3 is .
M 2 1
14

10 1
Area of triangle OAB OA u OB
2
§ 0 · §D ·
1¨ ¸ ¨ ¸
1 u 1
2 ¨¨ ¸¸ ¨¨ ¸¸
© 2 ¹ © 1 ¹
§ 1  2 ·
1 ¨ ¸
2 ¨  0  2D ¸
(i) ¨ 0 D ¸
© ¹
§ 1 ·
1¨ ¸
¨ 2D ¸
2¨ ¸
© D ¹
1
1  2D  D
2 2

2
1
1  5D 2
2
10m (ii) A  OC
3OA
3OA
By the Ratio Theorem, OB
4
§D · ª §0· º
¨ ¸ 1« ¨ ¸ »
¨1¸ « 3 ¨ 1 ¸  OC »
¨ 1¸4 ¨ ¸
© ¹ «¬ © 2 ¹ »¼
§ 4D · § 4D ·
¨ ¸ ¨ ¸
OC ¨ 4  3 ¸ ¨ 1 ¸
¨ 4  6 ¸ ¨ 2 ¸
© ¹ © ¹
§ D · § 4D ·
¨ ¸ ¨ ¸
OB is perpendicular to OC, ¨ 1 ¸ x ¨ 1 ¸ 4D 2  1 0
¨ 1¸ ¨ 2 ¸
© ¹ © ¹
1
D  D  0
2

(iii) Exact perpendicular distance from B to OA = BN

1 1 1 §1·
From (ii), D  Ÿ Area of triangle
trria
iang
n OAB 1  5D 2 1  5¨ ¸
2 2 2 ©4¹
1 9 1
A
BN OA
2 4 2
3
N 1 4
BN
2
3
BN
2 5
2 2 2
OB ON  BN
§1 · 2 9
¨  1  1¸ ON 
©4 ¹ 4 5
15

2 9
ON
5
§0·
¨ ¸
¨1¸ § 0 ·
3 ¨© 2 ¸¹ ¨ 3 ¸
ON ¨ 5 ¸
5 1 4 ¨ 6 ¸
© 5¹
16

11

(i)

As x o f, t o 1 Ÿ y o e
1
2
e

12m (ii) 1 dx 1
x Ÿ  3
t 1 dt 2 t  1
2 dy 2
y et Ÿ 2te t
dt
2
dy 2tet
t  1
2 3
 4tet (shown)
dx 1
t  1
3
2
(iii) 5 1 1
Gradient of normal at t is  0.012421
4 dy 80.50628
dx t 5
4

5 § 2 16
25
· §2 ·
When t , the point on C is ¨ , e ¸ or ¨ , 4.7707 ¸
4 ©3 ¹ ©3 ¹

5
The normal
orm
rrm
rmal
ma at
a t meets
ts tthe
he y-axis at the point P:
4
At y-axis::
25
§ 2·
y  e16 0.012421
01 ¨0  ¸
© 3¹
y 4.76245
P: 0, 4.76245
17

§ 1 2 ·
Given Q: ¨ ,eq ¸
¨ q 1 ¸
© ¹
2
§ 1 ·

2 2
PQ = ¨ ¸  e q  4.76245
¨ q 1 ¸
© ¹

Min PQ = 0.667 units when q= 1.25

OR
For PQ to be a minimum, PQ must be perpendicular to the tangent of Q [B1],
5
thus Q must be q [B1]
4
2
§ · 2
¨ 1 ¸ § 25 ·
Thus Min PQ = ¨ ¸  ¨ e 16  4.76245 ¸ =0.667 [B1]
¨ 5 ¸ ¨ ¸
¨ 1 ¸ © ¹
© 4 ¹

OR
For PQ to be a minimum, PQ must be perpendicular to the tangent of Q [B1]
Thus
1
§ 5· 3
4qe q q  1
2 3
1
¨ ¸ §5
 4 5 e© 4 ¹ ¨  1 ¸
4 ©4 ¹
·

Solving gives q = 1.25 [B1]


1]
2
§ · 2
¨ 1 ¸ § 25 ·
in PQ
Thus Min PQ = ¨ ¸  ¨ e  4.76245 ¸ =0.667 [B1]
16

¨ 5 ¸ ¨ ¸
¨ 1 ¸ © ¹
© 4 ¹

OR
For PQ to be a minimum, gradient of the line PQ must 0
18

§ 1 2 ·
2
dy e q  4.7625
P: 0, 4.76245 , Q: ¨ , e q ¸ , thus
¨ q 1 ¸ dx 1
© ¹
q 1
2
dy e q  4.7625
Solving for 0
dx 1
q 1
Ÿ q 1.25 (to 3 sf) [B1]
2
· § 16 ·
2 25
§ 1
Thus Min PQ = ¨ ¸ ¨ ¨ e  4.76245 ¸ =0.667
© 1.2493  1 ¹ © ¸
¹
19

12 (a)
(i) Month Outstanding amt. owed
1st day of month aft. instalment Last day of month aft. interest
Oct-2019 12000 12000(1.005)
Nov-2019 12000(1.005)  P 12000(1.0052 )  P(1.005)
Dec-2019 12000(1.0052 )  P(1.005)  P

Amount owed upon paying the 2nd instalment (in $)


= 12000(1.0052 )  P(1.005)  P
14m (a)
(ii) Instal Outstanding amt. owed
-ment 1st day of month aft. instalment Last day of month aft. interest
0 12000 12000(1.005)
1 12000(1.005)  P 12000(1.0052 )  P(1.005)
2 12000(1.0052 )  P(1.005)  P 12000(1.0053 )  P(1.0052 )  P(1.005)
3 12000(1.0053 )  P(1.0052 )  P(1.005)  P …
… … …
n 1
n 12000(1.005 )  P(1.005
n
)...  P(1.005)  P

Amount owed upon paying the nth instalment (in $)


12000(1.005n )  P(1 005n 1 )...  P(1.005)
(1.005 (1 005)  P
Geometric series, n terms
tric series
Starting from right : first term  P , common ratio 1.005

P(1.005n  1)
12000(1.005n ) 
1.005  1
12000(1.005 )  200 P(1.005n  1)
n
(shown)

(a) Intended last instalment on 1st December 2021.


(iii) - 2019 : 2 instalments
- 2020 : 12 instalments
- 2021 : 12 instalments
Total no. of instalments = 26
Amount owed upon paying
y ng the 2 th instalment (in $)
the 26
12000(1.00526 )  2
200 1.00526  1) d 0 if loan is repaid completely
00 P((1.005
formulate eqn.

1220
0000
0(((1.
1..00
1
12000(1.005 .00 26
26
005 ) § 1.005 ·26
?P t 60
60 ¨¨ ¸¸
200(1.005
2
20
00(
0(1.0
00 052626
 1)
1) 26
© 1.005  1 ¹
493.3395...
Minimum monthly instalment needed = $493.34
(nearest cent)

(b) Let a hertz be the frequency of the tone produced by the 1st key on the piano, and
(i) r be the common ratio of the frequency between successsive keys.
20

the 49th key produces 440 hertz, ar 48 440 (1)


the 61st key produces 440 hertz, ar 60 880 (2)
(2) 1
Eliminating a, : r12 2, r 212
(1)

440 440
From (1) : a
r 48
2 1 48
12

440 440
= 27.5 (frequency of 1st key in hertz)
24 16

(b) Frequency produced by the nth key on the piano (in hertz),
1
(ii) u ar n 1 27.5 (212 )n 1
n
n 1
27.5 (2 12 )

Sequence of logarithm of the frequencies produced by successive piano keys :


ln(u1), ln(u2 ), ln(u3 ), ... , ln(un ), ...

The difference between consecutive terms of the sequence


ln(un )  ln(un1 )
§ u ·
ln ¨ n ¸ , using properties of the ln function
© un1 ¹
1
ln(r ) , u1 , u2 , u3 , ... , un , ... is in a G.P., with r 212
1
ln(212 ) , a constant

? ln(u1), ln(u2 ), ln(u3 ), ... , ln(un ), ... is in an A.P.

(b) Ratio of ascending frequencies produced in a major triad chord, sounded by the
th th th
(iii) n , ( n  4 ) and ( n  7 ) keys is
ar n1 : ar ( n 4)1 : ar ( n7)
7 1

ar n1 (1) : ar n1 (r 4 ) : ar n1 (r 7 )


1 : r4 : r7
1 1
1 21122 )4 : ((2
: ((2 212 )7
1 7
1 : 23 : 212 ,
which is independent of n. (shown)
Name: Index Number: Class:

DUNMAN HIGH SCHOOL


Promotional Examination
Year 5

MATHEMATICS (Higher 2) 9758/01


Paper 1 3 October 2019
3 hours
Additional Materials: Answer Paper
List of Formulae (MF26)

READ THESE INSTRUCTIONS FIRST

Write your Name, Index Number and Class on all the work you hand in.
Write in dark blue or black pen on both sides of the paper.
You may use an HB pencil for any diagrams or graphs.
Do not use staples, paper clips, glue or correction fluid.

Answer all the questions.


Give non-exact numerical answers correct to 3 significant figures, or 1 decimal place in the case of
angles in degrees, unless a different level of accuracy is specified in the question.
You are expected to use an approved graphing calculator.
Unsupported answers from a graphing calculator are allowed unless a question specifically states
otherwise.
Where unsupported answers from a graphing calculator are not allowed in a question, you are required
to present the mathematical steps using mathematical notations and not calculator commands.
You are reminded of the need for clear presentation in your answers.

At the end of the examination, fasten all your work securely together.
The number of marks is given in brackets [ ] at the end of each question or part question.

For teachers’ use:


Qn Q1 Q2 Q3 Q4 Q5 Q6 Q7 Q8 Q9 Q10 Q11 Q12 Total
Score
Max
6 4 6 8 7 7 8 9 10 11 12 12 100
Score

© DHS 2019 This question paper consists of 6 printed pages (including this cover page).
2
1
1 (i) Obtain the expansion of 1  x 2 up to and including the term in x 2 . [1]

(ii) In the triangle ABC, AC = 1, BC = 3 and angle ACB = T  π


6 radians. Given that T is a
sufficiently small angle, show that

AB | 1  pT  qT 2 ,

where p and q are constants to be determined in exact form. [5]

n
r n2 n
2 Using the result ¦
r 1 2
r
2
2n
, show that ¦ (r  n)(2 r  1)
r 1

can be expressed in the form

§ 1 ·
C ¨1  n ¸  Dn(n  1), where C and D are constants to be determined. [4]
© 2 ¹

3 The equations of the lines l1 and l2 are as follows:

§ 3· §3 · x2
l1 : r ¨1 ¸  O ¨ D ¸ , O  , l2 : y  4, z 4.
¨ 3¸ ¨1 ¸ 4
© ¹ © ¹

where D is a constant.

(i) Write down a vector equation of line l2 . [1]

The two lines l1 and l2 intersect at point P.

(ii) Find the value of D and the position vector of P. [3]

(iii) Find the acute angle between the lines l1 and l2 . [2]

4 A curve C is defined by the parametric equations

t t
x , y , where 0 d t  1.
1 t2 1 t2

2
(i) Find the equation of the tangent to C at the point where x 5. [4]

(ii) Find the exact area of the finite region bounded by C, the tangent found in part (i) and
the x-axis. [4]

DHS 2019 Year 5 H2 Mathematics Promotional Examination


3

5 Express
2r  1
r (r  1)(r  2)

in partial fractions and hence use the method of differences to find the sum of the series

5 7 9 2n  1
   ...  . [5]
24 60 120 n n  1 n  2

5 7 9
Give a reason why the series    ... converges, and write down its value. [2]
12 30 60

6 The shaded region R is bounded by the curves x 3e 2y


, x 1
2  e 2 y and the x-axis as shown
in the diagram below.

R O
x

(i) Find the exact area of the region R. [5]

(ii) Find the volume of the solid of revolution formed when R is rotated through 4 right angles
about the y-axis, giving your answer correct to 2 decimal places. [2]

7 With respect to the origin O, the position vectors of two points A and B are given by a and b
respectively where a 1, b 2 and a ˜ b 1. Point C lies on AB such that AC 2CB . Point N
is the foot of perpendicular from C to the line OB.

(i) Find the position vector of the point C in terms of a and b. [1]

3
(ii) Show that the length of projection of OC onto OB is and deduce the position vector
2
of N. [4]

(iii) Find the value of O such that C, D and N are collinear where OD a  Ob . [3]

DHS 2019 Year 5 H2 Mathematics Promotional Examination


4

8 The function f is defined by

2x
f :x , for x  , x z 2 .
x2

(i) Find f 1 x , stating the domain of f 1 . [2]

(ii) Solve the equation f x f 1 x . [1]

The function g is defined as follows.

2
g:x , for x  , x z 3 , x z 4 .
x3

(iii) Find fg x . [1]

(iv) Solve the inequality fg x  x for all x in the domain of fg. [3]

(v) Find the range of fg . [2]

9 (a) (i) Find the values of a, b and c such that x ax  b x  1  c x 2  2 x  2 for all real
values of x. [2]

´ x
(ii) Hence find µ dx. [4]
¶ x  1 x  2 x  2
2

(b) (i) By using a graphical approach, solve 2 x  x  5 ! 0. [1]


2
(ii) Given that ³
2
2 x  x  5 dx p 3  13 q 7  1 for some real values of p and q, find
the values of p and q. [3]

DHS 2019 Year 5 H2 Mathematics Promotional Examination


5

b
10 The curve C1 is defined by y 2x 1 , x z  a, where a and b are constants. Given that
xa
C1 has two stationary points, what can be said about the values of a and b? [3]

It is given that a b 1.

Sketch C1, indicating in your graph any points where C1 crosses the axes and the equations of
any asymptotes. [2]

(a) The curve C2 is defined by x  p  y 2 R 2 , where p is a real constant and R ! 0. Sketch


2

the graph of C2 on the same diagram as C1. [1]

It is given that C1 and C2 intersect. By considering the stationary points of C1, find the
minimum value of R as p varies and state the corresponding value of p. Express both your
answers in exact form. [3]

1
(b) Sketch the graph of y f '( x), where f ( x) 2x 1 . [2]
x 1

11 An interior design company can only furnish one house at a time. For a residential project A, it
takes 160 hours to furnish the first house. Each subsequent house takes k hours shorter to furnish
than the previous one due to the similar design.

(i) If the company takes a total of 3800 hours to furnish the first 25 houses, find the exact time
it would need to furnish the next 25 houses. [4]

(ii) The company would like to achieve a target completion time of less than 140 hours for the
nth house. Find the value of n when the target is first met. [2]

To reduce the overall furnishing time, the company decides to double its manpower.
For project B, a similarly designed residential project like A, it still takes 160 hours to furnish the
first house. However, the time taken to furnish each subsequent house is 4% shorter than the
previous one.

(iii) Find the number of houses that can be fully furnished in 3800 hours. If productivity is defined
as the number of houses fully furnished per man-hour, calculate the percentage improvement
in productivity for project B compared to A based on the houses fully furnished in 3800 hours.

[Note : Man-hour for a task = number of people assigned × time taken to complete the task] [4]

(iv) For project B, show that the total time taken to furnish will not exceed 4000 hours, no matter
how many houses there are. Briefly comment on your answer in the context of this
question. [2]

DHS 2019 Year 5 H2 Mathematics Promotional Examination


6

12 [A circular cone with base radius r, vertical height h and slant height l, has curved surface area
πrl and volume 13 πr h. ]
2

A capsule made of metal sheet of fixed volume p cm3 is made up of three parts.

x The top is modelled by the curved surface of a circular cone of radius r cm. The ratio of its
height to its base radius is 4 : 3.
x The body is modelled by the curved surface of a cylinder of radius r cm and height H cm.
x The base is modelled by a circular disc of radius r cm.

The cost of making the body of the capsule is $k per cm2, while that of the top and the base of
the capsule is $2k per cm2, where k is a constant. The total cost of making the capsule is $C.
Assume the metal sheet is made of negligible thickness.

p 4r
(i) Show that H  . [2]
πr 2 9

A
(ii) Express C in the form  Br 2 , where A and B are expressions in terms of k and p.
r
Use differentiation to show that C has a minimum as r varies. [8]

(iii) Hence determine the ratio of H to r when C is a minimum. [2]

End of Paper

DHS 2019 Year 5 H2 Mathematics Promotional Examination


2019 Year 5 H2 Math Promotional Examination solution

Qn Suggested Solution
1(i) 1
1  x 2
1 1
1 x  x 2  ...
2 8
ii) Using cosine rule,

3 §
2
AB 2 12   2(1)( 3) cos ¨ T  ¸ B
© 6¹
S S
= 4  2 3 §¨ cos T cos  sin T sin ·¸
© 6 6¹
§ 3 1 ·
= 4  2 3 ¨¨ cos T  sin T ¸¸
© 2 2 ¹
= 4  3cos T  3 sin T A 1 C
§ 1 2·
| 4  3 ¨ 1  T ¸  3T
© 2 ¹
3
= 1  3T  T 2
2
1
§ 3 ·2
AB ¨1  3T  T 2 ¸
© 2 ¹
2
1§ 3 · 1§ 3 ·
= 1  ¨ 3T  T 2 ¸  ¨ 3T  T 2 ¸  ...
2© 2 ¹ 8© 2 ¹
3 3 3
= 1 T  T 2  T 2  ...
2 4 8
3 3
| 1 T  T2
2 8
3 3
where p , q
2 8

1
Qn Suggested Solution
2 n n
§ r n ·
¦ (r  n)(2
r 1
r
 1) ¦ ¨© 2
r 1
r
r
2 r
 n¸
¹
§ n2· §1 · § 12 1  ( 12 ) n ·
¨ 2  n ¸  ¨ n(n  1) ¸  n ¨¨ ¸  n2
¸
© 2 ¹ ©2 ¹ 1 2 1
© ¹
n 2 1 § · 2
1
2  n  n(n  1)  n ¨1  n
¸n
2 2
n
2 © 2 ¹
§ 1 · n 1 n
2 ¨1  n ¸  n  n(n  1)  n(n  1)  n
© 2 ¹ 2 2 2
§ 1 · 1 1
2 ¨1  n ¸  n( n  1) where C 2, D 
© 2 ¹ 2 2

Qn Suggested Solution
3(i) § 2 · § 4 ·
¨ ¸ ¨ ¸
l2 : r ¨ 4 ¸  P ¨ 1 ¸ , P 
¨4 ¸ ¨0 ¸
© ¹ © ¹
(ii) Consider
§ 3· § 3 · § 2 · § 4 ·
¨ ¸ ¨ ¸ ¨ ¸ ¨ ¸
¨1 ¸  O ¨ D ¸ ¨ 4 ¸  P ¨1 ¸ ,
¨ 3¸ ¨1 ¸ ¨ 4 ¸ ¨0 ¸
© ¹ © ¹ © ¹ © ¹
§3 · § 4 · § 5 ·
O ¨ D ¸  P ¨¨ 1¸¸ ¨¨ 3 ¸¸
¨ ¸
¨1 ¸ ¨ 0 ¸ ¨1 ¸
© ¹ © ¹ © ¹
Solving: 3  4P 5 Ÿ P 2, O 1, D 1 ,
§6·
¨ ¸
? OP ¨ 2 ¸ .
¨ 4¸
© ¹
(iii) Let T be the angle between the two lines.
§ 3 · § 4 ·
¨ ¸¨ ¸
¨1 ¸ . ¨1 ¸
¨1 ¸ ¨ 0 ¸
© ¹© ¹ 11
11
cos T
11 1
11 177 11 1
11 7
17

§ 11·
11
T cos 1 ¨¨ ¸¸ 3
36.448o 36.4o (1 d.p.)

© 17

2
Qn Suggested Solutions
4(i) t t
x , y
1 t 2
1 t2
dx 1 t2 dy 1 t2
dt 1  t 2 2 dt 1  t 2 2

§ dy ·
dy ¨ ¸ 1 t2 1 t2 § 1 t2 ·
3
© dt ¹ y ¨ 2 ¸
dx § dx · 1  t 2 2 1  t 2 2 © 1 t ¹
¨ ¸
© dt ¹
When x 52 ,
2 t
5
1 t2
2t 2  5t  2 0
(2t  1)(t  2) 0
t 1
2 or t 2 (reject 0 d t  1)
3
§ 1  1 2 · 125 1
When t 1 , dy
¨ 22 ¸ ,y 2 2
2 ¨ 1 1 ¸
dx 27 1  12
2 3
© 2 ¹
The equation of the tangent at x 52 is :
y  23 125
27 x  52
y 125 x  32
27 27
(ii) When x 0, t 0 y
2 1 C
When x t 5, 2
x-intercept of tangent : y 0
125 x  32 0
27 27
?x 32
125 O x

3
Required area
Area under C  Area of triangle
2
1 § 2 32 ·§ 2 ·
³0
5
y dx  ¨  ¸¨ ¸
2 © 5 125 ¹© 3 ¹
§ ·
1

t ·¨ 1 t2 ¸ 6
³0 ¨© 1  t 2 ¸¹ ¨ 1  t 2 2 ¸ dt  125
© ¹
1 § ·
¨ t ¸ dt  6
³0
2
¨ 1  t ¸
2 2
125
© ¹
1 12 6
2t 1  t 2 dt 
 2

2 ³0 125
1
1 ª 1 º2 6
 « 2»

2 ¬1  t ¼ 0 125
1 6

10 125
13
unit 2
250

Suggested Solution
olu
llu
uti
tion
on
5 2r  1 1 1 3
  (by cover-up rule)
r (r  1)(r  2) 2r r  1 2(r  2)

4
Suggested Solution
5 7 9 2n  1
   ... 
24 60 120 n n  1 n  2
n 2r  1
¦
r 2 r r  1 r  2

n § 1 1 3 ·
¦ ¨¨   ¸¸
r 2 2r
© r  1 2 r  2 ¹
1 n §1 2 3 ·
¦¨   ¸
2 r 2 © r r 1 r  2 ¹
1 ª1 2 3
«  
2 ¬2 3 4
1 2 3
  
3 4 5
1 2 3
  
4 5 6

1 2 3
  
n  2 n 1 n
1 2 3
  
n 1 n n 1
1 2 3 º
  
n n  1 n  2 »¼
1 ª1 2 1 3 2 3 º
    
2 ¬ 2 3 3 n  1 n  1 n  2 »¼
«
1 ª3 1 3 º
 
2 ¬ 2 n  1 n  2 »¼
«

1 ª3 1 3 º 1 ª3 º 3
As n o f, «   » o «  0  0»
2 ¬ 2 n 1 n  2 ¼ 2 ¬2 ¼ 4
5 7 9 § 5 7 9 ·
   ... 2 ¨    ... ¸
12 30 60 © 24 600 1120
20
20 ¹
f
2r  1 3 3
2¦ 2u (finite vvalue)
(finite
r 2 r r  1
r  2 4 2
3
? the series
ess cconverges
onverges aand
nd tthe
he sum to infinity is .
2

Qn Suggested Solution

5
6(i) 1 2y
e 3e 2 y
2
1
e2 y   3e 2 y 0
2
2e  e 2 y  6
4y
0
e 2y
 2 2e  3
2y
0
3
e2 y 2 or e 2 y  (rej. since e 2 y ! 0)
2
2y ln 2
ln 2
y
2
ln 2
§1 ·
Exact area of region R ³ 2
3e 2 y  ¨  e 2 y ¸ dy
0
©2 ¹
ln 2
ª § e 2 y · 1 e2 y º 2
 « 3 ¨ ¸  y  »
¬ © 2 ¹ 2 2 ¼0
ª§ 3 § 1 · ln 2 · § 3 1 · º
 «¨ ¨ ¸   1¸  ¨  ¸ »
¬© 2 © 2 ¹ 4 ¹ © 2 2 ¹¼
ln 2  1
unit 2
4
(ii) Volume of solid of revolution
ln 2 2

3e2 y  §¨© 12  e2 y ·¸¹ dy


2
S³ 2
0

4.24 unit 3 (2 d.p.)

Qn Suggested Solution

6
7(i) 1
OC (a  2b )
3
(ii) Length of projection of OC onto OB
1
OC b (a  2b) b
3
b 2
1 1 2
a b  2b b a b2 b
3 3
2 2
1
1  2(2) 2
6
3
2
3b 3
ON b
2b 4
(iii) 3 1 1 1
CN b  (a  2b)  a  b
4 3 3 12
3 3
ND (a  Ob)  b a  (O  )b
4 4
Since the points C, N and D are collinear,
ND D CN for so D 
f some
§ 3· § 1 1 ·
i.e. a  ¨ O  ¸ b D ¨  a  b ¸
© 4¹ © 3 12 ¹
Since a is not parallel to b and a and b are non-zero ,
D 3 D
1  and O 
3 4 12
1
?D 3 and O
2

Alternative
ND b 0

3 § 3·
ND b [((a  Ob)  b] b [a  ¨ O  ¸ b
b]] b 0
4 © 4¹
§ 3·
a b  ¨O  ¸b b 0
© 4¹
3
?1  4(O  ) 0
4
1
Thus O = .
2

Qn Suggested Solution
8(i) 2x
Let y f ( x )
x2
7
Qn Suggested Solution
xy  2 y 2 x
x y  2 2 y
2y
x
y2
2y
f 1 y
y2
2x
? f 1 x
x2
Domain of f 1 = \ ^2` .
(ii) f x f 1 x
Since both graphs intersects at all points in the domain of f
x  \ ^2`
(iii) § 2 ·
fg x = f ¨ ¸
© x3¹
§ 2 ·
2¨ ¸
© x 3¹
=
2
2
x 3
4
= x 3
2  2 x  3
x 3
4
=
8  2x
2
=
4 x

(iv) fg x  x

8
Qn Suggested Solution
2
x
4 x
y=x

(3,2)

x=4

Intersection points between the 2 graphs.


ie 0.586, 0.586 and 3.41, 3.41

For the inequality to be satisfied,


x ! 4 or 0.586  x  3 or 3  x  3.41
x ! 4 or x  0.586,3.41 \ ^3`

Alternative
2
x
4 x
2
x0
4 x
2  4 x  x2
0
4 x
x  2
2
2
0
4 x
x  2  2 x  2  2  0
4 x

x ! 4 or 2  2  x  3 or 3  x  2  2

x ! 4 or x  2  2, 2  2 \ ^3`
(v) For the range of fg

9
Qn Suggested Solution
Dg 
g
o Rg \ ^0,, 2` 
f
o R fg \ ^0, 2`

Qn Suggested Solution
9(a) x ax  b x  1  c x 2  2 x  2
(i)
a  c x 2  a  b  2c x  b  2c
Comparing coefficients,
a  c 0
a  b  2c 1
b  2c 0
Solving, we have a 1, b 2, c 1.

Alternative
Sub x 1, 1 0  c 1  2  2 Ÿ c 1
Compare coeff x 2 : a  c 0 Ÿ a 1 0 Ÿ a 1
Compare coeff x : a  b  2c 1 Ÿ 1  b  2 1 Ÿ b 2
? a 1, b 2, c 1
(ii) ´ x
µ dx
¶ x  1 x  2 x  2
2

´ x  2 x  1  x 2  2 x  2
µ dx
µ
¶ x  1 x 2
 2 x  2
´ x2 1
µ 2  dx
¶ x  2x  2 x 1
´ x 1 1 1
µ x2  2 x  2   dx
x  1  1 x  1
2

1 ´ 2( x  1) ´ 1 1
µ 2 dx  µ  dx
2 ¶ x  2x  2 ¶ x  1  1 x  1
2

1
2
2

ln x  1  1  taan 1 x  1  ln
ttan ln x  1  c
(b)(i)

(ii)

Qn Suggested Solution
10 b
y 2x 1 , x z a
xa
10
Qn Suggested Solution
dy b
2 , x z a.
x  a
2
dx

Given that C1 has two stationary points,


dy b
2 0 has a solution.
x  a
2
dx
b
x  a
2
has a solution ---- (*)
2
Thus we need b ! 0 (Note b 0 gives C1: y 2 x  1 which has no stationary
points)

Alternative 1
dy b
2 0 has a solution.
x  a
2
dx
2 x  a  b 0 has a solution
2

2 x 2  4ax  2a 2  b 0 has a solution


Discriminant = 16a 2  4(2)(2a 2  b) 8b t 0
Thus b ! 0 (Note b z 0 ) and a .

Alternative 2
b
For a rational function curve y 2x 1 , x z  a to have stationary points,
xa
need y o 2 x  1 as x o f


Thus b ! 0 and a .

11
Qn Suggested Solution

x
O

(i) x  p R is a circle centre  p, 0 and radius R .


2
 y2

1 1
From (*), for stationary points of C1, x  1
2
Ÿx 1 r
2 2
1 § 1 · 1
x 1  , y 2 ¨ 1  ¸  1  1 1  2 2
2 © 2¹ 2

Min R for C1 and C2 to intersect occurs when the centre of the circle is right beneath
§ 1 ·
the point ¨ 1  , 1  2 2 ¸ .
© 2 ¹
Thus min R 2 2 1
§ 1 · 1
Corresponding value of p  ¨ 1  ¸ 1
© 2¹ 2
(ii)
y

x
O

12
Qn Suggested Solution
11(i) Project A: Total time to furnish first 25 houses,
25
S 25 3800 > 2(160)  k (25  1) @
2
320  24k 304
2
k
3

Time to furnish next 25 houses


S50  S 25
50 ª 2 º
« 2(160)  (50  1) »  3800
2 ¬ 3 ¼
1
3383 h or 3383h 20min
3

Alternative
Time needed to furnish the 26th house,
2 1
u26 160  (26  1) 143 h
3 3
Total time needed to furnish the next 25 houses
25 ª 1 2 º
2(143 )  (25  1) »
2 «¬ 3 3 ¼
1
3383 h or 3383h 20min
3
(ii) 2
160  (n  1)  140
3
n  1 ! 30
n ! 31
? n t 32
For target time of < 140h to be first met, n = 32

(iii) 160(1  0.96 n )


Sn 3800
1  0.96
1  0.96 n 0.95
n
0.96 0.05
ln 0.05
n 4
73.4
ln 0.96
Number of ho
houses
ouses
uses
uses tthat
hat ca
ha ccan
an bbee fu
ful
fully
lly furnished = 73

Let x be the nnumber


umbe
um
umbe
ber of m
man used for project A
houses fully furnished 25
Productivity forr A =
man-hours 3800x
houses fully furnished 73
Productivity for B =
man-hours 3800 u 2x

13
Improvement in productivity
73 25

7600 x 3800 x 46%
25
3800 x
(iv) 160(1  0.96n ) 160 
Sn < since 0.96n  1, n 
1  0.96 1  0.96
160
Sf 4000
1  0.96
? the total time taken will not exceed 4000h.

As the geometric series is convergent, the time taken for furnishing a house will
eventually be negligible in the long run. But this is not possible in reality since a
substantial amount of time will be required to furnish a house.

Qn Suggested Solutions
12(i) 4r
Height of cone, h
3
1 4
Volume p πr 2 H  πr 2 h πr 2 H  πr 3
3 9
1 § 4 · p 4r
H 2 ¨
p  πr 3 ¸ 2
 (shown)
πr © 9 ¹ πr 9

(ii) 2
§ 4r · 25r 2 5r
Slant height of the cone l r ¨ ¸
2

© 3¹ 9 3
C k 2πrH  2k πrl  2k πr 2
§ p 4r · § 5r ·
2kπr ¨ 2  ¸  2kπr ¨ ¸  2kπr 2
© πr 9 ¹ © 3¹
2 2
2kp 8kπr 10kπr
   2kπr 2
r 9 3
2
2kp 40kπr

r 9
40
0kkππ
A 2kp, B
9

dC 2kp p 80
80kπr
 2

dr r 9
dC 2kp 80kπr
For minimum C,   0
dr r2 9
2kp 80kπr
r2 9
18kp 9 p
r3
80kπ 40π
14
Method 1 [2nd derivative]
d 2C 4kp 80kπ d 2C
 . k , p ! 0
0, ! 0.
dr 2 r3 9 dr 2
9p
When r 3 , C is minimum.
40π

Method 2 [1st derivative]


dC 2kp 80kπr § 2 p 80π ·
 2  kr ¨ 3  ¸ **
dr r 9 ©r 9 ¹
9p  9p 9p 
r 3 3 3
40π 40π 40π
dC
ve 0 +ve
dr
Tangent \ – /
9p
When r 3 , C is minimum.
40π
(iii) H § p 4r ·§ 1 ·
¨ 2  ¸¨ ¸
r © πr 9 ¹© r ¹
p 4

πr 3 9
p 4

§ 9p · 9
π¨ ¸
© 40π ¹
40 4

9 9
4

?H : r 4 :1

15
EUNOIA JUNIOR COLLEGE
JC1 Promotional Examination 2019
General Certificate of Education Advanced Level
Higher 2

CANDIDATE
NAME

CLASS INDEX NO.

MATHEMATICS 9758/01
02 October 2019

Paper 1 [100 marks] 3 hours

Candidates answer on the Question Paper

Additional Materials: List of Formulae (MF26)

READ THESE INSTRUCTIONS FIRST

Write your name, civics group and question number on all the work you hand in.
Write in dark blue or black pen.
You may use an HB pencil for any diagrams or graphs.
Do not use paper clips, highlighters, glue or correction fluid.

Answer all questions.

Give non-exact numerical answers correct to 3 significant figures, or 1 decimal place in the case of angles in
degrees, unless a different level of accuracy is specified in the question.
You are expected to use an approved graphing calculator.
Unsupported answers from a graphing calculator are allowed unless a question specifically states otherwise.
Where unsupported answers from a graphing calculator are not allowed in a question, you are required to
present the mathematical steps using mathematical notations and not calculator commands.
You are reminded of the need for clear presentation in your answers.
The number of marks is given in brackets [ ] at the end of each question or part question.

This document consists of 28 printed pages (including this cover page) and 2 blank page.

For markers’ use:


Q1 Q2 Q3 Q4 Q5 Q6 Q7 Q8 Q9 Q10 Q11 Q12 Total
1 At the start of the year, Mr Toh invested in three types of savings bonds, namely “Ucare”, “Ushare”
and “Ugain”. The amount invested in “Ugain” is equal to the sum of the amounts invested in the
other 2 bonds. In addition, the sum of the amount invested in “Ushare” and twice the amount invested
in “Ugain” is 8 times the amount invested in “Ucare”.

At the end of the year, “Ucare”, “Ushare” and “Ugain” paid out interest at a rate of 2.5%, 1.75%
and 3% respectively. Mr Toh received a total of $657.60 in interest.

Express this information as 3 linear equations and hence find the amount invested in savings bond
“Ushare”. [4]

2x 1
2 Sketch the curve y 3 , stating the equations of the asymptotes.
x 1

2x 1
Hence, solve the inequality ! x5. [5]
x 1

3 The curve C has equation a 2 x2  y 2  2(ax  2 y  2) 0 where a is a constant such a  \ ^0` .


dy a ax  1
Show that .
dx y2
Hence, find the coordinates of the points on C at which the tangent is parallel to y-axis. [6]

4 Referred to the origin O, the points A, B, and C have position vectors a, b, and c respectively.

(i) Given that non-zero numbers λ and μ are such that λa + μb + c = 0 and λ + μ + 1 = 0 with
P ! 0 . Show that A, B, and C are collinear and find the ratio CA : CB in terms of μ. [4]

(ii) F is another point such that the line passing through A, B, and C does not contain it. Find
BF u BC
in terms of μ. [2]
AF u AC

2019 JC1 H2 Mathematics Promotional Examination [Turn over


2n
5 (a) Find ¦ 3
r n
r
 n . [3]

§ r2 ·
(b) Express ln ¨ 2 ¸ as A ln r  1  B ln r  C ln r  1 , where A, B and C are integers to
© r 1 ¹
be determined.
n
§ r2 ·
Hence, find ¦ ln ¨ 2 ¸ , leaving your answer as a single logarithmic function in terms
r 2 © r 1 ¹
of n. [4]

6 The sum, S n , of the first n terms of a sequence ^u n ` is given by Sn n 2  (k  2)n , where k is a non-
zero real constant.
(i) Prove that the sequence ^u n ` is an arithmetic sequence. [3]

(ii) Given that u8 , u4 and u2 are the first 3 terms in a geometric sequence, find the value of k.
[2]
(iii) Give a reason why the geometric series converges and find the value of the sum to infinity.
[2]

7 The function f is defined by


bx b
f :x , ffor x  , x z ,
ax  b a
where a and b are non-zero constants.
(i) Find f 1 x and state the domain of f 1 . [3]
(ii) Hence show that f
2
x x , and write down f n
x where n is an odd number. [2]
The function g is defined by g : x 2  e x , for x  , x t 0. If a 2 and b 1 for function f,
(iii) explain why the composite function gf does not exist. [1]
(iv) find an expression for fg x and state the domain and exact range of fg . [3]

2019 JC1 H2 Mathematics Promotional Examination [Turn over


1
8 The diagram shows the curve y = f(x). The curve has a minimum point ( , 3k ) and a maximum
2
point (3,  k ) where k ! 0 . It has asymptotes x 2 and y 3  x .

x=2

O (3, – k)
y=3–x

Sketch, on separate diagrams, the graph of


(i) y f(x  2)  k , [3]
showing clearly the stationary points, axial intercepts and equations of all asymptotes.

1
(ii) y , [3]
f(x )
showing clearly the stationary points, axial intercepts and equations of all asymptotes.

(iii) y f ' x , [3]


showing clearly the stationary points, axial intercepts and equations of all asymptotes.

2019 JC1 H2 Mathematics Promotional Examination [Turn over


1 4x
9 (a) Find
³ 4 x2  1
dx . [4]
S2 S
(b) Use the substitution u = x to show that ³0
4
x sin x dx 2³ 2 u 2 sin u du .
0
[2]
S2
Hence, evaluate the integral ³ 4
x sin x dx exactly. [4]
0

S
10 The curve C is defined parametrically by the equations x 2cosecT , y 2 cot T , where 0 < θ < .
2
dy
(i) Show that sec θ . [2]
dx

(ii) Find the equation of the tangent to the curve at the point P 2cosec p, 2 cot p and show that
the equation of the normal to the curve at P is y 4 cot p  x cos p . [3]

(iii) The tangent and normal at P cut the x-axis at T and N respectively.
Show that T has coordinates ( 2 sin p , 0).

By finding the x-coordinates of N in the simplest form, evaluate ON OT . [4]

(iv) Find the rate of change of y at (2 2, 2) , given that x is increasing at a constant rate of 1 unit
per second. [3]

2019 JC1 H2 Mathematics Promotional Examination [Turn over


11 Engineers are building a greenhouse on an inclined grass slope. Points ( x, y, z ) are defined relative
to a base point O (0, 0, 0) , where units are in metres.

The greenhouse is in the shape of a tetrahedron. The corners O, A and B lie on the grass slope, while
the point C is the highest point of the tetrahedron (also called apex), as shown in the diagram below.

X
A

M
O

The coordinates of A, B and C are (100,100,10) , (160, 20,8) and (80,59, 20) respectively.

(i) Show that the Cartesian equation of the grass slope is 3x  4 y  70 z 0. [2]

(ii) Find the angle of inclination of the grass slope. [2]


(iii) To secure the greenhouse, the architect plans to build a central pillar from the apex directly to
the floor of the greenhouse such that the central pillar is perpendicular to the horizontal plane.
Find the point where the central pillar meets the grass slope. [2]
Unfortunately, the workers misunderstand the instructions and build the central pillar from the apex
to the floor of the greenhouse such that the pillar is perpendicular to the grass slope.

(iv) Find the length of this central pillar and use this length to find the volume enclosed by the
greenhouse OABC. [4]
1
[The volume of a tetrahedron is given by u base area u height .]
3
Another straight supporting beam is to be constructed from point M, the midpoint of OB, to a point X
on OC such that XM is perpendicular to OB.
(v) Find the coordinates of X, correct to 1 decimal place. [3]

2019 JC1 H2 Mathematics Promotional Examination [Turn over


12 In a model making competition, each contestant is given a square cardboard of side n cm to make a
model. John takes part in the Pyramid Category and he is given a n u n cardboard to make a pyramid
with the maximum volume. John recalled his calculus during his schooling days and came out with
a design as shown in the diagrams below.
A n B

P E

x x

O
S Q n
P Q

O x

R S x R
Figure 2
D C
Figure 1

Figure 1 shows a square piece of cardboard ABCD of side n cm, where n is a positive constant. Four
triangles (ABP, BCQ, CDR, DAS) are removed from each side of ABCD. The remaining shape is now
folded along PQ, QR, RS and SP to form a pyramid (as shown in Figure 2) with a square base PQRS
of side x cm. The point O is the centre of both ABCD and PQRS. The vertex E is the point where A,
B, C and D meet. Let V be the volume of the pyramid.

(i) Show that OE 2


2

1 2
n  2nx . [3]

(ii) Show that V


2

18

x4 2

n  2nx . Hence, by using differentiation, find in terms of n, the exact

value of x which John uses to obtain a maximum value of V. [7]


(iii) John’s pyramid has a volume that is greater than 45 cm3. Find the smallest n u n cardboard
given, where n   . [2]
1
[The volume of a pyramid is given by u base area u height .]
3

2019 JC1 H2 Mathematics Promotional Examination [Turn over


EUNOIA JUNIOR COLLEGE
JC1 Promotional Examination 2019
General Certificate of Education Advanced Level
Higher 2

CANDIDATE
NAME

CLASS INDEX NO.

MATHEMATICS 9758/01
02 October 2019

Paper 1 [100 marks] 3 hours

Candidates answer on the Question Paper

Additional Materials: List of Formulae (MF26)

READ THESE INSTRUCTIONS FIRST

Write your name, civics group and question number on all the work you hand in.
Write in dark blue or black pen.
You may use an HB pencil for any diagrams or graphs.
Do not use paper clips, highlighters, glue or correction fluid.

Answer all questions.

Give non-exact numerical answers correct to 3 significant figures, or 1 decimal place in the case of angles in
degrees, unless a different level of accuracy is specified in the question.
You are expected to use an approved graphing calculator.
c
Unsupported answers from a graphing calculator culat
attor are a allowed unless a question specifically states otherwise.
Where unsupported answers from a graphing graph
p in
ph ing
g calculator
calc
ca are not allowed in a question, you are required to
present the mathematical steps using ng
n gmmathematical
ath
at he matititiccal notations
ema n and not calculator commands.
You are reminded of the need d for
ed or clear
for clea
ear presentation
prresen
p e ta
en tatitition
on in
i your answers.
The number of marks is given
giv
iveen
n in
in brackets
brac
br ackke
ets
ts [ ] at the
t end of each question or part question.

This document
ntt c
co
consists
ons
nsis
ists
is t of 28 printed pages (including this cover page) and 2 blank page.

For markers’ use:


Q1 Q2 Q3 Q4 Q5 Q6 Q7 Q8 Q9 Q10 Q11 Q12 Total
1 At the start of the year, Mr Toh invested in three types of savings bonds, namely “Ucare”, “Ushare”
and “Ugain”. The amount invested in “Ugain” is equal to the sum of the amounts invested in the
other 2 bonds. In addition, the sum of the amount invested in “Ushare” and twice the amount invested
in “Ugain” is 8 times the amount invested in “Ucare”.

At the end of the year, “Ucare”, “Ushare” and “Ugain” paid out interest at a rate of 2.5%, 1.75%
and 3% respectively. Mr Toh received a total of $657.60 in interest.

Express this information as 3 linear equations and hence find the amount invested in savings bond
“Ushare”. [4]

Suggested solution
Let c, s and g be the amount of investments in saving bonds ‘Ucare’, ‘Ushare’ and ‘Ugain’
respectively.
Amount invested in bond ‘Ugain’ equals the sum of the amounts invested in the other 2 bonds:
cs g

csg 0 ----- (1)

The sum of the amount invested in “Ushare” and twice the amount invested in “Ugain” is 8 times the
amount invested in “Ucare”.

s  2g 8c

8c  s  2 g 0 -----(2)

Total interest of $657.60:


0.025c  0.0175s  0.03 g 657.60 ----(3)

Solving simultaneously,
c $4384, s $8768 and g $13152

Total amount of investments for Ushare


hare
ree is
is $8768
$876
$8 7

2019 JC1 H2 Mathematics Promotional Examination [Turn over


2x 1
2 Sketch the curve y 3 , stating the equations of the asymptotes.
x 1

2x 1
Hence, solve the inequality ! x5. [5]
x 1

Suggested solution

C
B

2x 1 3
Note that since y 2 , the horizontal asymptote is given by y = 5.
x 1 x 1

2x 1 2x 1
! x 5 Ÿ 3 ! x8
x 1 x 1

Sketch the graph of y x  8 on the same axes.

y
y:
ven bby:
By GC, the intersection points are given
A 3.65, 4.35
B 0.606,8.61
C 1.65, 9.65

Therefore, the solution is::

x  3.65 or 0.606  x  1 or 1  x  1.65

(Note that x 1 is not part of the solution)

2019 JC1 H2 Mathematics Promotional Examination [Turn over


3 The curve C has equation a 2 x2  y 2  2(ax  2 y  2) 0 where a is a constant such a  \ ^0` .
dy a ax  1
Show that .
dx y2
Hence, find the coordinates of the points on C at which the tangent is parallel to y-axis. [6]

Suggested solution

a 2 x 2  y 2  2(ax  2 y  2) 0
dy dy
2a 2 x  2 y  2(a  2 ) 0
dx dx
dy dy
2a 2 x  2 y  2a  4 0
dx dx
dy
2  y a  a2 x
dx
dy a  a 2 x a ax  1
dx 2 y y2

The point on C at which the tangent is parallel to y-axis


dx
Ÿ 0
dy

Ÿ 2 y 0
Ÿy 2

a 2 x2  y 2  2(ax  2 y  2) 0

a 2 x 2  22  2(ax  2(2)  2) 0
a 2 x 2  2ax 0
ax > ax  2@ 0
2
x or x 0
a

§2 ·
? (0, 2) and ¨ , 2 ¸ are the tw
ttwo
wo po
ppoints
oiin
nts
ts w
which
hiich iits
ts ttangent is parallel to to the y-axis.
©a ¹

2019 JC1 H2 Mathematics Promotional Examination [Turn over


4 Referred to the origin O, the points A, B, and C have position vectors a, b, and c respectively.

(i) Given that non-zero numbers λ and μ are such that λa + μb + c = 0 and λ + μ + 1 = 0 with
P ! 0 . Show that A, B, and C are collinear and find the ratio CA : CB in terms of μ. [4]

(ii) F is another point such that the line passing through A, B, and C does not contain it. Find
BF u BC
in terms of μ. [2]
AF u AC

Suggested solution
(i)
Method 1
We first show A, B, and C are collinear by CA k CB , where k is a constant to be found in terms
of P , and note that we can then also use the value of constant k to determine the ratio CA: CB later.

Using λa + μb + c = 0, and λ + μ + 1 = 0
CA a  c CB b  c
a  Oa  Pb b  Oa  Pb
a  ( P  1)a  P b b  ( P  1)a  Pb
P (b  a) P  1 (b  a)
P
Thus, CA CB ,
P 1
since CA is a scalar multiple of CB , and C is a common point, so A, B, and C are collinear.
P
CA CB
P 1
CA P
CB P 1
Thus CA : CB = P :1  P

Method 2
(− μ − 1) a + μb + c = 0 λ =−μ− 1
μ (b − a) + (c − a) = 0
PA
AB  AC 0
1
AB  AC
P

Ÿ Since AB is a scalar multiple of AC , and A is a common point, A, B, and C are collinear.

2019 JC1 H2 Mathematics Promotional Examination [Turn over


 P 1 a  P b  c 0
 Pb  c
a
 P 1
Pb  c
a
P +1
Thus, A divides BC in the ratio 1: μ Ÿ CA : CB = P :1  P
(ii)
F

C A 1 B

1
BF u BC u BF u BC
2
AF u AC 1
u AF u AC
2
area 'BFC
area 'AFC
1 P
( the two triangles have the same height.)
P
Method II:

Consider the fixed height h:

F u BC
BF AF u AC
h
BC AC
BF u BC BC 1 P
Ÿ (from
om
m (i
(i))
i))))
AF u AC AC P

2019 JC1 H2 Mathematics Promotional Examination [Turn over


2n
5 (a) Find ¦ 3
r n
r
 n . [3]

§ r2 ·
(b) Express ln ¨ 2 ¸ as A ln r  1  B ln r  C ln r  1 , where A, B and C are integers to
© r 1 ¹
be determined.
n
§ r2 ·
Hence, find ¦ ln ¨ 2 ¸ , leaving your answer as a single logarithmic function in terms
r 2 © r 1 ¹
of n. [4]

Suggested solution
2n 2n 2n

¦ 3r  n
r n
¦ 3r  ¦ n
r n r n

3 3
n n 1
 1
 n n  1
2
3n 3n 1  1  2n n  1
2
§ r · 2
§ r ·2
ln ¨ 2 ¸ ln ¨¨ ¸¸
© r 1 ¹ © r  1 r  1 ¹
ln r 2  ln r  1  ln r  1
 ln r  1  2 ln r  ln r  1

n
§ r2 · n

¦ ln ¨ 2 ¸ ¦ ª¬ ln r  1  2 ln r  ln r  1 º¼
r 2 © r 1 ¹ r 2

[ ln1  2 ln 2  ln 3
 ln 2  2 ln 3  ln 4
 ln 3  2 ln 4  ln 5
 ln 4  2 ln 5  ln 6
...
 ln n  3  2 ln n  2  ln n  1
 ln n  2  2 ln n  1  ln n
 ln n  1  2 ln n  ln n  1 ]
 ln1  lln n n  ln n  1
n 2  lln
§ 2n ·
ln ¨ ¸
© n 1¹

2019 JC1 H2 Mathematics Promotional Examination [Turn over


6 The sum, S n , of the first n terms of a sequence ^u n ` is given by Sn n 2  (k  2)n , where k is a non-
zero real constant.
(i) Prove that the sequence ^u n ` is an arithmetic sequence. [3]

(ii) Given that u8 , u4 and u2 are the first 3 terms in a geometric sequence, find the value of k.
[2]
(iii) Give a reason why the geometric series converges and find the value of the sum to infinity.
[2]

Suggested solution

(i) Sn n 2  (k  2)n

un S n  S n 1
n 2  (k  2)n  ª¬(n  1) 2  (k  2)(n  1) º¼
n 2  kn  2n  ª¬ n 2  2n  1  kn  2n  k  2 º¼
2n  1  k
un  un 1
2n  1  k  > 2(n  1)  1  k @
2 is a constant independent of n

?the sequence ^u n ` is an arithmetic sequence

(ii) u8 2(8)  1  k 17  k

u4 2(4)  1  k 9k

u2 2(2)  1  k 5k

9k 5k
17  k 9k
9  k 17  k 5  k
2

81  18k  k 2 85  22k  k 2
4k  4 0
k 1

Alternative Solution:
on
n:
Given that un is a AP, so un = a + 2(n – 1), where a is the first term in the AP.

2019 JC1 H2 Mathematics Promotional Examination [Turn over


u8 a  2(7) a  14
u4 a  2(3) a  6
u2 a  2(1) a  2

Since u8 , u4 , u2 are first three terms in a GP,

a6 a2
a  14 a  6
(a  6)2 (a  2)(a  14)
a 2  12a  36 a 2  16a  28
4a 8
a 2

S1 a
1  (k  2)(1)
2
2
k 1
9k 8 1
(iii) r
17  k 16 2
1
Since r  1 , the geometric series is convergent and
2
u8 16
? Sf 32
1 r 1  12

2019 JC1 H2 Mathematics Promotional Examination [Turn over


7 The function f is defined by
bx b
f :x , for x  , x z ,
ax  b a
where a and b are non-zero constants.
(i) Find f 1 x and state the domain of f 1 . [3]
(ii) Hence show that f 2 x x , and write down f n x where n is an odd number. [2]
x
The function g is defined by g : x 2  e , for x  , x t 0. If a 2 and b 1 for function f,
(iii) explain why the composite function gf does not exist. [1]
(iv) find an expression for fg x and state the domain and exact range of fg . [3]
Suggested solution
bx b
(i) f :x , for x  , x z ,
ax  b a
bx
Let y Ÿ y ax  b bx
ax  b
Ÿ axy  bx by
by
Ÿx
ay  b
bx b
f 1 : x , for x  , x z ,
ax  b a
(ii) Since f ( x) f ( x) , f x f ª¬f 1 x º¼
1 2
x

bx
f 3 x f ª¬f 2 x º¼ f x , therefore for f n x f x for n odd,
ax  b
(iii) If a 2 and b 1 for function f,
x 1
f :x , for x  , x z ,
2x 1 2
­1 ½
Rf \® ¾
¯2¿
D g > 0, f
As R f ΠD g , gf does not exist.
(iv) fg x f 2  e x
2  e x
2 2  e x  1
2  e x
, xt0
3  2e  x
Dfg Dg [0, f)

ª3 2 ·
Range of R fg = « , ¸
¬5 3 ¹

2019 JC1 H2 Mathematics Promotional Examination [Turn over


1
8 The diagram shows the curve y = f(x). The curve has a minimum point ( , 3k ) and a maximum
2
point 3  k ) where k ! 0 . It has asymptotes x 2 and y 3  x .

x=2

O (3, – k)
y=3–x

Sketch, on separate diagrams, the graph of


(i) y f(x  2)  k , [3]
showing clearly the stationary points, axial intercepts and equations of all asymptotes.

1
(ii) y , [3]
f(x )
showing clearly the stationary points axial intercepts and equations of all asymptotes

(iii) y f ' x , [3]


showing clearly the stationary points,
nts, aaxial
xia intercepts and equations of all asymptotes.
xi

2019 JC1 H2 Mathematics Promotional Examination [Turn over


Suggested solution
(i)

y f(x  2)  k

(ii)

2019 JC1 H2 Mathematics Promotional Examination [Turn over


(iii)

y f ' x

1
( , 0) (3, 0)
2
O
y 1

x 2

2019 JC1 H2 Mathematics Promotional Examination [Turn over


1 4x
9 (a) Find
³ 4 x2  1
dx . [4]
S2 S
(b) Use the substitution u = x to show that ³0
4
x sin x dx 2³ 2 u 2 sin u du .
0
[2]
S2
Hence, evaluate the integral ³ 4
x sin x dx exactly. [4]
0

Suggested solution
1 4x § 1 4x ·
(a)
³
4 x2  1
dx ¨ 2  2
³
© 4x 1 4x 1 ¹
¸ dx
1 2 1 8x
2
2 (2 x)  1
dx 
³
2 4 x2  1
dx
³
1
2
1
tan 1 2 x  ln 4 x 2  1  C ,
2

where C is an arbitrary constant.

Alternative:
1 1 1 1 §¨ 1 ·¸ 1 x
³ 4x 2
1
dx
4 ³ x  1
2
2
dx
4¨ 1 ¸
© 2¹
tan
1
2
2
1
tan 1 2 x  C
2

(b) Given substitution: Let u = x Ÿ u2 = x


S2 S
When x 0, u 0 x ,u
4 2
du
Differentiating with respect to x, we have 2u 1
dx
du 1 § du 1 ·
? ¨ or dx ¸
dx 2u © 2 x¹
S2 S
Hence, ³ 0
4
x sin x dx 2³ 2 u 2 sin u du
0

S S S
2³ 2 u 2 sin u du = 2 ª¬u 2  cos u º¼ 2  2³ 2 2u  cos u du (By parts) =
0 0 0
S S
2 ª¬ u 2 cos u º¼ 2  4³ 2 u cos u du
du ((s
(simplify)
siim
mp
pllify))
0 0
S
S
ª S º
oss u ¼º 2  4 «>u sin u @02  ³ 2 sin u du »
= 2 ª¬ u 2 ccos
o =
0 0
¬ ¼
S
2 ª¬ u 2 cos u  2u sin u  2 cos u º¼ 2 =
0

§§ §S · S · ·
2 ¨ ¨ 0  2 ¨ ¸ sin  0 ¸  0  0  2 ¸ = 2 S  2
©© ©2¹ 2 ¹ ¹

2019 JC1 H2 Mathematics Promotional Examination [Turn over


S
10 The curve C is defined parametrically by the equations x 2cosecT , y 2 cot T , where 0 < θ < .
2
dy
(i) Show that sec θ . [2]
dx

(ii) Find the equation of the tangent to the curve at the point P 2cosec p, 2 cot p and show that
the equation of the normal to the curve at P is y 4 cot p  x cos p . [3]

(iii) The tangent and normal at P cut the x-axis at T and N respectively.
Show that T has coordinates ( 2 sin p , 0).

By finding the x-coordinates of N in the simplest form, evaluate ON OT . [4]

(iv) Find the rate of change of y at (2 2, 2) , given that x is increasing at a constant rate of 1 unit
per second. [3]

Suggested solution
(i) x 2 cosecT y 2 cot T
dx dy
2 cosecT cot T 2 cosec 2T
dT dT
dy d y d T
dx dT dx
2 cosec 2T
2 cosecT cot T
cosecT tan T
1 sin T
sin T cos T
sec T
(ii)Equation of tangent at point P 2 cos ec p, 2 cot p with T p :
y  2 cot p sec p x  2cosec p
y 2(cot p  cosec p sec p)  x sec p

Equation of normal at pointt P 2 ccos cot p with T


os ecc p, 2 cot p:
1
y  2 cot p  x  22c ec p
2cosec
osec
cos
sec p
§ 2 ·
y 2 cot p  (cos p ) ¨ x  ¸
© sin p ¹
y 4 cot p  x cos p

(iii)At point T, y 0
0 2(cot p  cosec p sec p )  x sec p

2019 JC1 H2 Mathematics Promotional Examination [Turn over


2(cot p  cosec p sec p )
x
 sec p
2 ( cos 2 p cosec p  cosec p )
2cosec p (1  cos 2 p )
2cosec p (sin 2 p )
2sin p

At point N, y 0
0 4 cot p  x cos p

4 cot p
x
cos p
4cosec p

ON OT 2sin t 4 cosec t
8

dy dy dx
(iv) ˜
dt dx dt
(sec T )(1)
S
At ( x, y ) (2 2, 2) , T ,
4
dy S
sec 2.
dt 4

2019 JC1 H2 Mathematics Promotional Examination [Turn over


11 Engineers are building a greenhouse on an inclined grass slope. Points ( x, y, z ) are defined relative
to a base point O (0, 0, 0) , where units are in metres.

The greenhouse is in the shape of a tetrahedron. The corners O, A and B lie on the grass slope, while
the point C is the highest point of the tetrahedron (also called apex), as shown in the diagram below.

X
A

M
O

The coordinates of A, B and C are (100,100,10) , (160, 20,8) and (80,59, 20) respectively.

(i) Show that the Cartesian equation of the grass slope is 3x  4 y  70 z 0. [2]

(ii) Find the angle of inclination of the grass slope. [2]


(iii) To secure the greenhouse, the architect plans to build a central pillar from the apex directly to
the floor of the greenhouse such that the central pillar is perpendicular to the horizontal plane.
Find the point where the central pillar meets the grass slope. [2]
Unfortunately, the workers misunderstand the instructions and build the central pillar from the apex
to the floor of the greenhouse such that the pillar is perpendicular to the grass slope.

(iv) Find the length of this central pillar and use this length to find the volume enclosed by the
greenhouse OABC. [4]
1
etrah
heddro
ron is given
[The volume of a tetrahedron given by u base area u height .]
3
ght ssupporting
Another straight uuppppoort
rtin
ing beam
beaam
be m iiss to
to bbe constructed from point M, the midpoint of OB, to a point X
att XM
on OC such that XM is is perpendicular
perp
pe rpen
ndi
dicu to OB.
(v) oord
rdin
rd inat of X, correct to 1 decimal place.
in
Find the coordinates [3]

2019 JC1 H2 Mathematics Promotional Examination [Turn over


Suggested solution
§ 10
100 · §160 · § 600 · § 3 ·
¨ ¸ ¨ ¸ ¨ ¸ ¨ ¸
(i) OA u OB ¨ 10 100 ¸ u ¨ 20 ¸ ¨ 800 ¸ 200 ¨ 4 ¸
¨ 10 ¸ ¨ 8 ¸ ¨ 14000 ¸ ¨ 70 ¸
© ¹ © ¹ © ¹ © ¹
Since the origin lies on the plane OAB.
§ 3 ·
¨ ¸
Plane OAB: r ˜ ¨ 4 ¸ 0
¨ 70 ¸
© ¹
3x  4 y  70 z 0 (shown)
(ii) Angle of inclination
§ 3 · §0·
¨ ¸ ¨ ¸
¨ 4 ¸˜¨0¸
¨ 70 ¸ ¨ 1 ¸ 70
1 © ¹ © ¹
cos cos 1 4.1q (to 1 d.p.)
§ 3 · §0· 4925
¨ ¸ ¨ ¸
¨ 4 ¸ ¨0¸
¨ 70 ¸ ¨ 1 ¸
© ¹ © ¹
(iii) Method I: Let F be a point directly below C. Hence coordinate of F is (80,59, f ) .
Since F is on plane OAB,
§ 80 · § 3 ·
¨ ¸ ¨ ¸ 34
¨ 59 ¸ ˜ ¨ 4 ¸ 0 Ÿ 476  70 f 0 Ÿ f 5
6.8
¨ f ¸ ¨ 70 ¸
© ¹ © ¹
34
Hence the coordinates of the point F is (80, 59, )
5

Method II: Let the line from apex parallel to k be Lc .


§ 80 · §0·
¨ ¸ ¨ ¸
Lc : r = ¨ 59 ¸  O ¨ 0 ¸ O 
¨ 20 ¸ ¨1¸
© ¹ © ¹

At point of intersection with planee OAB,


OAB
OA B,

§ 80 · § 3 ·
¨ ¸ ¨ ¸ 66
¨ 59 ¸ ˜ ¨ 4 ¸ 0 Ÿ 92
9 2
24
924 470O
70 0ŸO 
¨ 20  O ¸ ¨ 70 ¸ 5
© ¹ © 0¹
34
Hence the coordinates
naate
tes of the point F is (80, 59, )
5

2019 JC1 H2 Mathematics Promotional Examination [Turn over


(iv) Perpendicular distance from C to the plane OAB
§ 3 · § 80 · § 3 ·
¨ ¸ ¨ ¸ ¨ ¸
OC ˜ ¨ 4 ¸ ¨ 59 ¸ ˜ ¨ 4 ¸
¨ 70 ¸ ¨ 20 ¸ ¨ 70 ¸
© ¹ © ¹ © ¹
§ 3 · § 3 ·
¨ ¸ ¨ ¸
¨ 4 ¸ ¨ 4 ¸
¨ 70 ¸ ¨ 70 ¸
© ¹ © ¹
924 924
13.16645 | 13.2 m (to 3.s.f.)
4925 5 197

Volume enclosed by the greenhouse


1
Area of 'OAB Perpendicular distance from C to plane OAB
3
1§ 1 · § 924
9 ·
¨ OA u OB ¸¸¨¨ ¸
3© 2 ¹ © 4925 ¹
§ 3 ·
1 ¨ ¸ § 924 ·
200 ¨ 4 ¸ ¨ ¸
6 ¨ 70 ¸ © 4925 ¹
© ¹
3
30800 m
§880 ·
¨ ¸
(v) Since X lies on OC, OX 5 ¸ , for some t  .
t ¨ 59
¨ 20 ¸
© ¹
Since XM is perpendicular to OB, XM ˜ OB 0

OM  OX ˜ OB 0
§ § 80 · § 80 · · §160 ·
¨¨ ¸ ¨ ¸¸ ¨ ¸
¨ ¨ 10 ¸  t ¨ 59 ¸ ¸ ˜ ¨ 20 ¸ 0
¨ ¨ 4 ¸ ¨ 20 ¸ ¸ ¨ 8 ¸
©© ¹ © ¹¹ © ¹
13032 14140t 0
3258
t
3535
§8 0·
80
325
2558 ¨ ¸
3258
Therefore, OX 59 .
59
3535 ¨¨ ¸¸
3535
20 ¹
© 20
re ((73.7,
Coordinates of X aar
are 73.77 554.4, 18.4) .

2019 JC1 H2 Mathematics Promotional Examination [Turn over


12 In a model making competition, each contestant is given a square cardboard of side n cm to make a
model. John takes part in the Pyramid Category and he is given a n u n cardboard to make a pyramid
with the maximum volume. John recalled his calculus during his schooling days and came out with
a design as shown in the diagrams below.
A n B

P E

x x

O
S Q n
P Q

O x

R S x R
Figure 2
D C
Figure 1

Figure 1 shows a square piece of cardboard ABCD of side n cm, where n is a positive constant. Four
triangles (ABP, BCQ, CDR, DAS) are removed from each side of ABCD. The remaining shape is now
folded along PQ, QR, RS and SP to form a pyramid (as shown in Figure 2) with a square base PQRS
of side x cm. The point O is the centre of both ABCD and PQRS. The vertex E is the point where A,
B, C and D meet. Let V be the volume of the pyramid.

(i) Show that OE 2


2

1 2
n  2nx . [3]

(ii) Show that V


2

18

x4 2

n  2nx . Hence, by using differentiation, find in terms of n, the exact

obt
b a a maximum value of V.
value of x which John uses to obtain [7]
(iii) John’s pyramid has a volume
volu hatt is greater than 45 cm3. Find the smallest n u n cardboard
me that
ume tha
ha
given, where n   . [2]
1
olum
ume of
um
[The volume of a ppyramid
yrraam
y mid iiss ggiven by u base area u height .]
3

2019 JC1 H2 Mathematics Promotional Examination [Turn over


Suggested solution
(i) Method I
BD n2  n2 2n
2n  x
Let M be the midpoint of SR, ? DM EM
2
EM 2 OE 2  OM 2
OE 2 EM 2  OM 2
2
§ 2n  x · § x ·
2

¨¨ ¸ ¨ ¸
© 2 ¸¹ © 2 ¹
1
4
2n  2 2nx  x  x
2 2 2

1
4
2n  2 2nx
2

1
2
n  2nx shown
2

Method II (Not recommended)


PR x2  x2 2x
n 2 x
Let M be the midpoint of DC, ? RM
2
2
§n· §n 2 x·
2

DR 2
DM  RM 2 2
¨ ¸  ¨¨ ¸¸ ER 2
©2¹ © 2 ¹
ER 2 OE 2  OR 2
OE 2 ER 2  OR 2
ª§ n · 2 § n  2 x · 2 º § 2 x · 2
«¨ ¸  ¨ ¸¸ »  ¨¨ ¸¸
«© 2 ¹ ¨© 2 ¹ »¼ © 2 ¹
¬
n 2 n 2  2 2 nx  2 x 2 2 x 2
 
4 4 4
1
4

2n 2  2 2 nx
1 2
2

n  2 nx shown
(ii)
1
V base area height, OE
3
1 2 1 2
3
x
2
n  2nx
ŸV 2 x4 2
18
n  2nx shown

2019 JC1 H2 Mathematics Promotional Examination [Turn over


Method I (Implicit differentiation and 2nd derivative test)

V 2 x4 2
18

n  2nx
18

n2 4
x 
2n 5
18
x
Diff wrt x,

dV n2 2n
2V
dx18
4 x3 
18
5x4
2n 2 3 5 2n 4
x  x
9 18
dV 2n 2 3 5 2n 4
When 0 Ÿ x  x 0
dx 9 18

Ÿ
nx3
18

4n  5 2 x 0
4n
Ÿx or x 0 rej. x ! 0
5 2
2 2
n
5
dV 2n 2 3 5 2n 4
Consider: 2V x  x
dx 9 18

Diff wrt x,

§ dV · § dV · d 2V 2n 2 5 2n
2¨ ¸¨
© dx ¹ © dx
¸
¹
 2V
dx 2 9
3x 2 
18
4 x3
2
§ dV · d 2V 2n 2 2 10 2n 3
2¨ ¸  2V x  x
© dx ¹ dx 2 3 9

2 2 n dV
Substituting x , 0,
5 dx
2 3
d 2V 2n 2 § 2 2n · 10 2n § 2 2n ·
2 0  2V 2
2
¨ ¸  ¨ ¸
dx 3 ¨© 5 ¸¹ 9 ¨© 5 ¸¹
d 2V 2n 2 § 8n 2 · 10 2n § 16
16 2 n 3 ·
2V 2 ¨ ¸ ¨ ¸
dx 3 © 25 ¹ 9 ¨© 112525 ¸¹
16n 4 6464n 4

755 2225
25
4
16n

225
2
dV 8n 4
  0 since V , n ! 0
dx 2 225V
2 2
?V is maximum when x n.
5

2019 JC1 H2 Mathematics Promotional Examination [Turn over


Method II (Making V the subject, differentiating and using 1st derivative test) – Not
recommended

V
1 2 1 2
3
x
2

n  2nx
1
1
3 2

x 2 n 2  2nx 2

1 ­° ª1 º ½°
1 1
dV


®2 x n  2nx  x « n 2  2nx
2 2 2 2
2n »¾
dx 3 2 ¯° ¬2 ¼ ¿°
­ ª º½
1 °° « » °°
1
 2n
®2 x n  2nx
2
2
 x2 « 1 »¾
3 2°
°¯
« 2 n 2  2nx
¬ 2 »°
¼ °¿
­ ª º½
°
x ° 4 n 2
 2 nx «
 x«
 2n » °°
® 1 1 »¾
6 2° 2
¯°
n  2nx 2 « n 2  2nx
¬ 2 »°
¼ ¿°
ª º
x 4n  4 2nx  2nx »
« 2

« 1 »
6 2«
¬
n  2nx
2 2 »
¼
ª º
nx « 4n  5 2 x »
« 1 »
6 2« 2
¬
n  2nx 2 »
¼
When
dV
dx
0 Ÿ x 4n  5 2 x 0

4n
Ÿx or x 0 rej. x ! 0
5 2
2 2
n
5
 
§2 2 · 2 2 §2 2 ·
x ¨¨ n ¸¸ n ¨¨ n ¸¸
© 5 ¹ 5 © 5 ¹
4n  5 2 x ! 0 4n  5 2 x  0
Explanation 4n  5 2 x 0
x!0 x!0
dV
+ve
+v
+ve 0 –ve
dx
Slope / – \

2 2
?V is maximum when x n.
5

2019 JC1 H2 Mathematics Promotional Examination [Turn over


(iii) Method I (Inequalities without GC table method)
2 2
V is maximum when x n, V ! 45 Ÿ V 2 ! 452
5

From part (ii), V 2 x4 2


18

n  2nx n2 4
18
x 
18
2n 5
x
4 5
n2 § 2 2 · 2n § 2 2 ·
¨¨ n ¸¸  ¨¨ n ¸¸ ! 452
18 © 5 ¹ 18 © 5 ¹
32 6 128 6
n  n ! 2025
5625 28125
32 6
n ! 2025
28125
n ! 11.008 5s.f.
Least integer n is 12. Smallest cardboard given is 12cm u12cm .

Method II (Inequalities using GC table method) - Recommended


2 2
V is maximum when x n, V ! 45
5

From part (ii), V 2

18

x4 2
n  2nx Ÿ V 18

x4 2
n  2nx
4 5
n2 § 2 2 · 2n § 2 2 ·
¨¨ n ¸¸  ¨ n ¸ ! 45
18 © 5 ¹ 18 ¨© 5 ¸¹

Using GC,
When n 11, V 44.9  45
When n 12, V 58.3 ! 45

Smallest cardboard given is 12cm u12cm .

2019 JC1 H2 Mathematics Promotional Examination [Turn over


-&+0DWK3URPR([DP3DSHU+&,
1

1 The rates from three different utilities retailers Kappol Utilities, Super Power and uSwitch
are shown below.

Daily Fees Electricity Gas Water

$1.20/day 17.75 cents per kWh 17.15 cents per kWh $2.852/m3

Monthly
hly contract: $40/
$40/mth
Electricity: 23.65 cents per kWh
h
Electricity: $0.1762/kWh only!
Gas: 22.79 cents per kWh
Wh
Gas: $0.1698/kWh only!
Water: $3.672/m3
Water: $2.741/m3 only!

Based on Mr Lim’s utilities consumption for the 30 days in the month of June, Mr Lim
found that if he subscribed all his electricity, gas and water services from a single utilities
retailer, he would have to pay Kappol Utilities, Super Power and uSwitch a bill of $143.06,
$140.78 and $144.96 respectively.
Find the amount of electricity and gas in kWh, and water in m 3 that Mr Lim used in the
month of June. [3]

2 (i) Differentiate tan 1 x k , where k is a positive integer, leaving your answer in


terms of k. [1]
(ii) Hence find ³x
k 1
tan 1
x dx
k
in terms of k. [3]

3 (i) Show that the x-coordinate of the point(s) of intersection between a horizontal
ax  b
line y k where k  and the curve y where a, b  would satisfy
x2  1
the equation
kx 2  ax  k  b 0 . [1]
(ii) Given that 1 d k d 4 , find the values of a and b. [4]

An arithmetic sequence a1 , a2 , a3 , ... has common difference d, where d z 0 and


4
a1 z 0 .
Another geometric sequence b1, b2 , b3 , ... has common ratio r, where r ! 0, r z 1 . It is
given that a1 b1 , a3 b3 and a7 b5 .
If an bm where n, m   , find n in terms of m. [6]

[Turn over
2

5
B

O
A C

The diagram above shows a circle centred at O with radius 1 unit.


2 1
It is given that OA a , OB b and OC  a  b .
3 3
(a) Show that the area of triangle ABC is given by a u b square units. Given that
π
‘ACB , find the exact area of triangle ABC . [5]
3

(b) The point M lies on CB produced such that the area of triangle AMC is three
times that of the area of triangle ABC . Find OM in terms of a and b . [2]

d
6 (i) Find
dT
sec n T . [1]

2 x
(ii) Using the substitution x 2 tan 2 T , show that ³ 0
x 1
2
dx can be expressed as

k ³ sec5 T tan T  sec3 T tan T dT ,


b

a
where a, b and k are constants to be determined. [4]

2 x
(iii) Hence find the exact value of ³0
x 1
2
dx , expressing your answer in the form

A
2  B , where A and B are constants to be determined. [2]
3

7
y

x
O

42
The curves C1 and C2 have equations x2  y 2 4 and y respectively.
1  x2
(i) Find the exact coordinates of the points of intersection of C1 and C2. [2]

(ii) Find the area of the shaded region, giving your answer correct to 3 decimal places.
[2]
(iii) Find the exact volume of the solid obtained when the shaded region is rotated
through S radians about the x-axis. [3]

1 1§ 1 1 ·
8 (i) Show that ¨  ¸. [2]
(2r  3) 2r  1  (2r  1) 2r  3 2 © 2r  1 2r  3 ¹
n
1
(ii) Given that Sn ¦
r 1 (2r  3) 2r  1  (2r  1) 2r  3
, find S n in terms of n. [3]

(iii) Find the smallest value of n for which the difference between S n and S f is less than
0.05. [3]

9 The parametric equations of a curve are given by


§ S· S
x sin t cos t , y cos ¨ t  ¸ , where 0 d t d .
© 4¹ 2
dy 1
(i) Show that . [2]
dx 2 sin t  cos t
(ii) Hence find the equation of the tangent parallel to the y-axis. [2]
The curve cuts the y-axis at the points P and Q.
(iii) Find the exact coordinates of P and Q. [2]
§ § S ··
(iv) The point R on the curve has coordinates ¨ sin T cos T , cos ¨T  ¸ ¸ .
© © 4 ¹¹
2
Show that the area of triangle PQR is given by sin 2T . Hence find the value of
4
T for which the area of triangle PQR is a maximum. (You need not prove that it is
a maximum.) [2]

[Turn over
4

1
10 (i) Describe a sequence of transformations which transforms the graph of y
x
3x  5
to the graph of y . [3]
x2
3x  5
(ii) Sketch the graphs of y and 16( x  3)2  9( y  4)2 144 on a single
x2
diagram, indicating clearly any axial intercepts, points of intersection of the two
graphs and the equations of asymptotes. [5]
(iii) Hence find the set of values of x that satisfies the inequality
3x  5 144  16( x  3)2
! 4 . [2]
x2 9

11 The function f is defined by


2 4  x  e , where x  .
2
f :x
(i) Sketch the graph of y f x , showing clearly the exact coordinates of any axial
intercepts and turning points. [2]
1
(ii) Show that f does not exist. [1]
(iii) The domain of f is restricted to a d x  e , where a  .
1
State the smallest exact value of a for which f 1 exists. Hence find f in similar
form. [5]

The function g is defined by


x
g:x , where x  , x z 1.
x 1
(iv) Using the domain in part (iii), show that gf 1 exists and find the exact range of
gf 1 . [3]
5

12

Goal line
Goal

3m
Water polo player
P
m
xm
4.5 m

Side of pool
The diagram (not drawn to scale) shows part of a rectangular water polo pool with a fixed
width of y m . A goal 3 m wide is placed on the goal line at one end of the pool with the
y
centre of the goal m from the side of the pool. A water polo player at a distance of x m
2
perpendicular to the goal line and a distance of 4.5 m away from the side of the pool swims
directly towards a point P on the goal line. A visual angle T of the goal is the angle
subtended at the eye of the water polo player by the goal.
12 x
(i) Show that tan T where A y  6 y  12 . [3]
4x2  A
(ii) The desired visual angle of the goal is obtained when T is a maximum. Find by
differentiation, the value of x such that the desired visual angle of the goal is
obtained. Leave your answer in exact form in terms of A. [4]
For the rest of the question, let y 20 .
(iii) Show that the water polo player needs to be 2 7 m away from the goal line in
order to obtain the desired visual angle. [2]
(iv) The water polo player swims at a constant speed of 50 m per minute. Find the rate
of change of T at the instant when the water polo player is 15 m away from the goal
line. [3]

[Turn over
6

13

Night light
Room

In the diagram (not drawn to scale), a night light with a light emitting diode L housed in
a transparent acrylic case is placed in a room. A triangular sticker is stuck onto one face
of the acrylic case. Referred to the origin O, the three vertices of the triangular sticker have
coordinates A (2, 3, 3) , B (4, 3, 4) and C (2, 2, 5) , where the units of measurement are
in centimetres. When the night light is switched on, a triangular shadow A ' B ' C ' of the
triangular sticker is cast on a vertical plane wall p1 on one side of the room.

o
§ 3 · o
§9·
It is given that OA ' ¨¨ 13 ¸¸ and OB ' ¨ ¸.
¨ 13 ¸
¨8¸ ¨ 14 ¸
© ¹ © ¹

o
§ 3·
¨ ¸
(i) Show that OL ¨ 1 ¸ . [4]
¨ 2¸
© ¹

o
(ii) Given that p1 is parallel to the x-z plane, find OC ' . [4]

The slanted ceiling p2 is a plane with equation y  4 z 1053 .


(iii) Find the shortest distance between L and p2 . [2]
(iv) Find the line l when p2 meets p1 . [2]
HCI 2019 H2 Mathematics Promotional Exam Solutions

Qn Solution
1 Let e, g, w be amount of electricity and gas in kWh and water in m3 used by Mr Lim in June
respectively.
0.1775e  0.1715 g  2.852w 143.06  36
0.1775e  0.1715 g  2.852w 107.06     (1)

0.2365e  0.2279 g  3.672w 140.78     (2)

0.1762e  0.1698 g  2.741w 144.96  40


0.1762e  0.1698 g  2.741w 104.96     (3)

By G.C.,
e 266.4558551 266
g 115.0629925 115
w 14.03603875 14.0

Mr Lim used 266kWh of electricity, 115kWh of gas and 14.0 m3 of water in June.
2(i) d kx k 1
tan 1 x k
dx 1  x2k
2(ii) u tan 1 x k v ' x k 1
kx k 1 xk
u' v
1  x2k k

³x
k 1
tan 1 x k dx
xk x 2 k 1
tan 1 x k  ³ dx
k 1  x2k
xk 1 2kx 2 k 1
tan 1 x k 
2k ³ 1  x 2 k
dx
k
xk 1
tan 1 x k  ln 1  x 2 k  c
k 2k
since 1  x ! 0
2k

3(i) a xb
k
x2  1

k x2  k a xb
k x 2  ax  (k  b) 0 --- ------------(1)
---
- ----
----
---
- --(1 (shown)
3(ii) For (1) to have real all roots,
ro
ooottss,
(  a )  4 k ( k  b) t 0
2

a 2  4k 2  4kb t 0
4k 2  4bk  a 2 d 0

Find critical values

Page 1 of 16
HCI 2019 H2 Mathematics Promotional Exam Solutions

Qn Solution

4b r 16b 2  4 4 a 2
k
2 4
b r b2  a 2
k
2

Therefore
4k 2  4bk  a 2 d 0
b  b2  a 2 b  b2  a 2
dkd
2 2

Since 1 d k d 4,
b 1  4
Ÿb 3
2 2
b  b2  a 2
4 Ÿ 9  a2 5Ÿa r4
2

4 Let a1 b1 a

a3 b3 Ÿ a  2d a r2 ------------(1)
a7 b5 Ÿ a  6d ar 4
------------(2)

Eliminate d, (1) u 3 – (2):


2a 3 a r 2  a r 4
a r 4  3 r 2  2 0

Since a z 0 ,
r4  3r2  2 0 -------------(3)
- --
--- -----
----
- --
--
Ÿ r 2 1, or r 2 2

Since r ! 0, r z 1 , therefore,
the
here
reffo
ore
orree, r 2.
Sub r 2 into (1):
(1)
1):
a
a  2d 2a Ÿ d
2

If an bm :

Page 2 of 16
HCI 2019 H2 Mathematics Promotional Exam Solutions

Qn Solution
a  (n  1)d a r m1
a
a  (n  1) a r m 1
2
(n  1) m 1
1 r
2
(n  1) 1
( m 1)
22
2
1
( m1)
n 22 1
5(a) 1
Area of ABC AC u BC
2
1§ 2 1 · § 2 1 ·
¨  a  b  a¸u¨  a  b  b ¸
2© 3 3 ¹ © 3 3 ¹
1 § 5 1 · § 2 4 ·
¨  a  b¸u¨  a  b¸
2 © 3 3 ¹ © 3 3 ¹
1 § 1 ·§ 2 ·
¨ ¸ ¨ ¸ 5 a  b u a + 2 b
2 © 3 ¹© 3 ¹
1
5 a u a  b u a  10 a u b  2 b u b
9
1
9a u b
9
aub (shown)

Since ‘AOB 2 u‘ACB since ‘ at centre = 2 u‘ at circumference.

Area of ABC aub


2S
a b sin nˆ
3
§ 3·
1 1 ¨¨ ¸¸ 1
© 2 ¹
3
its 2
units
2

Page 3 of 16
HCI 2019 H2 Mathematics Promotional Exam Solutions

Qn Solution
5(b) Since MC 3BC ,
M  2OC
OM M 2 B 1 C
OB
3
OM 3OB  2OC
§ 2 1 ·
3b  2 ¨  a  b ¸
© 3 3 ¹
4 11 O
a b
3 3
Alternatively,
OM OC  CM
OC  3CB
3OB  3OC
6(i) d
sec n T n sec n 1 T sec T tan T
dT
n sec n T tan T
6(ii) dx
x 2 tan 2 T Ÿ 4 tan T sec 2 T
dT
When x 0, tan T 0 Ÿ T 0
S
When x 2, tan T 1Ÿ T
4
2 x
³0
x 1
2
dx
S

³ 4
2 tan 2 T 1  tan 2 T 4 tan T sec 2 T dT
0
S

³ 0
4
8 tan 3 T sec T sec 2 T dT
S
§ S·
³ 0
4
8 tan 3 T sec3 T dT ¨ sec T ! 0 for 0  T  ¸
© 4¹
S
8³ 4 tan T sec 2 T  1 sec3 T dT
0
S
8³ 4 sec5 T tan T  sec3 T tan
tan T dT
0
S
where k 8, a 0,b
4

Page 4 of 16
HCI 2019 H2 Mathematics Promotional Exam Solutions

Qn Solution
6(iii) 2 x
³0 x 1  2 dx
S
8³ 4 sec5 T tan T  sec3 T tan T dT
0
S
ª sec5 T sec3 T º 4
8« 
¬ 5 3 »¼ 0
§ 2 5
·
3

¨ 2 ¸ 8§ 1 1 ·
¨ 5  3 ¸  ¨© 5  3 ¸¹
8
¨ ¸
© ¹
16
15

2 1
16
A ,B 1
15
7(i) To find exact points of intersection, sub C2 into C1 :
4
x2  4
1  x2
x2  x4  4 4  4x2
x 4  3x 2 0
x x  3 0
2 2

x 0 or x r 3

(or using G.C,)


When x 0, y 2 4Ÿ y r2
When x r 3, y 2 1Ÿ y r1
? 0, 2 , 0, 2 ,
3,1 ,  3,1 ,
3, 1 ,  3, 1
7(ii) Method 1: Using ³ x dy
Area of shaded region
ª 1 2 4 º
4 « ³ 4  y 2 dy  ³  1 d y »
¬
0 1 y2 ¼
11.260 units to 3d.p
2

OR
2 4
4S  4³ 4  y2   1 dy
1 y2

Method 2: Using ³ y dx

Page 5 of 16
HCI 2019 H2 Mathematics Promotional Exam Solutions

Qn Solution
Area of shaded region
ª 3 2 2 º
4 «³ dx  ³ 4  x 2 dx »
¬
0
1  x2 3
¼
11.260 units to 3d.p
2

OR
3 4
4S  2³ 4  x2  dx
 3 1  x2
7(iii) ª 3 4 2 º
Vx 2S « ³ dx  ³ 4  x 2 d x »
¬ 0 1 x
2 3
¼
ª 3 ª x2 º º
2

2S « ¬ª 4 tan x ¼º  « 4 x  » »
1

«¬ 0
¬ 3 ¼ 3»
¼
§ 4S 8 ·
2S ¨ 8 4 3  3¸
© 3 3 ¹
2S
3
4S  16  9 3 units 3
OR
4 ª 3 4 º
S 2  S «³ 4  x2 
3
Vx dx »
3 ¬  3 1 x 2
¼
8(i) From RHS:

1§ 1 1 · 1 2r  3  2r  1
¨  ¸
2 © 2r  1 2r  3 ¹ 2 2r  1 2r  3

1 2r  3  2r  1 2r  3  2r  1
2 2r  1 2r  3 2r  3  2r  1
1 (2r  3)  (2r  1)
=
2 (2r  3) (2r  1)  (2r  1) (2r  3)
1
=
(2r  3) (2r  1)  (2r  1) (2r  3))
=LHS

OR

From LHS:
1
(2r  3) (2r  1)  (2
2r  1) (2r  3)
1
=
(2r  3) (2r  1) ª¬ (2r  3)  (2r  1) º¼

Page 6 of 16
HCI 2019 H2 Mathematics Promotional Exam Solutions

Qn Solution
(2r  3)  (2r  1)
=
(2r  3) (2r  1) > 2r  3  2r  1 @
2r  3  2r  1
=
2 2r  1 2r  3
1§ 1 1 ·
= ¨  ¸
2 © 2r  1 2r  3 ¹
=RHS
8(ii) n
1
Sn ¦ (2r  3)
r 1 (2r  1)  (2r  1) (2r  3)

1 n § 1 1 ·
¦ ¨
¨
2 r 1 © (2r  1)

(2r  3)
¸
¸
= ¹ ---------------(1)

ª 1 1 º
« 3 5 »
« »
« 1 1 »
« 5 7 »
1« »
= «... »
2« »
« 1 
1 »
« 2n  1 2n  1 »
« »
« 1 
1 »
«¬ 2n  1 2n  3 »¼

1§ 1 1 ·
= ¨  ¸
2 © 3 2n  3 ¹

8(iii) 1
n o f, o0
2n  3
1
lim S n
x of 2 3
1
Sn   0.05
2 3

1§ 1 1 · 1
¨  ¸  0.05
0.0055
0. ------------(2)
-----
--
2© 3 2n  3 ¹ 2 3

1
 0.05
2 2n  3

Page 7 of 16
HCI 2019 H2 Mathematics Promotional Exam Solutions

Qn Solution
1
 0.05
2 2n  3
2n  3 ! 10
2n  3 ! 100
n ! 48.5

OR
1
From GC, n = 48, = 0.05025
2 2n  3

1
n = 49, = 0.04975
2 2n  3
therefore the least value n is 49.

9(i) dx
cos 2 t  sin 2 t
dt
dy d§ S S·
¨ cos t cos  sin t sin ¸
dt dt © 4 4¹
2 d
cos t  sin t
2 dt
2
 sin t  cos t
2

dy dy dt
? .
dx dt dx
2
 sin t  cos t
2
cos 2 t  sin 2 t
sin t  cos t

2 cos t  sin t cos t  sin t
1
shown
2 sin t  cos t

9(ii) For tangent parallel to y  aaxis,


xis,
dy
d Ÿ ssin
is undefined in t  cos
cos t 0
dx
an t 1
ttan
S
t
4

Page 8 of 16
HCI 2019 H2 Mathematics Promotional Exam Solutions

Qn Solution
Ÿ Equation of tangent is:
2
S S § 2· 1
x sin cos ¨¨ ¸¸
4 4 © 2 ¹ 2
1
?x
2
9(iii) Method 1
When x 0,
sin t cos t 0
2sin t cos t 0
sin 2t 0
2t 0 or S
§ S· 2
t 0Ÿ y cos ¨ 0  ¸
© 4¹ 2
S §S S · 2
or t Ÿy cos ¨  ¸ 
2 ©2 4¹ 2

Method 2
When x 0,
sin t cos t 0
§ S· 2
sin t 0Ÿt 0Ÿ y cos ¨ 0  ¸
© 4¹ 2
S §S S · 2
cos t 0Ÿt Ÿy
cos ¨  ¸ 
2 ©2 4¹ 2
§ 2· § 2·
? P ¨¨ 0, ¸¸ , Q ¨¨ 0,  ¸
© 2 ¹ © 2 ¸¹

9(iv)
y

From the diagram, area of triangle PQR is given by

Page 9 of 16
HCI 2019 H2 Mathematics Promotional Exam Solutions

Qn Solution
1 § 2·
u 2¨ ¸ u sin T cos T
2 © 2 ¹
2 §1 ·
¨ sin 2T ¸
2 ©2 ¹
sin 2T
2 2
2 sin 2T
shown
4
S
Since 0 d sin 2T d 1 for 0 d T d ,
2
S
max area of triangle PQR is when sin 2T 1ŸT
4
10 3x  5 1
y 3
(i) x2 x2

Method 1:
1 1 1 1
 1
o 
2
o  
3
o 3
x x2 x2 x2

1. Translate 2 units in the negative x-direction


2. Reflect in x-axis
3. Translate 3 units in the positive y-direction
Method 2:

1 1 1 1
o  o  o 3
x x x2 x2

1. Reflect in y-axis
2. Translate 2 units in the negative x-direction
3. Translate 3 units in the positive y-direction

Students should recognize the equation


tionn of
of the ellipse.
16 x  3  9 y  4 144
2 2

x  3 y  4
2 2

 1
32 42
When using GC,
C student
stude
ttu
ude
dent
nt should
sho
h ulld use
use zoon-square to ensure that the axis are both equal in scale.

Page 10 of 16
HCI 2019 H2 Mathematics Promotional Exam Solutions

Qn Solution

10
(ii)
y

(-2.21,
-2 21 7.86)
7 86)

(-3,4)
4
y=3
(-5.95, 3.25)
(-0.240, 2.43)
(-0.2
(-1.61, 0.458)

-33 x
x = -2

10 From the graph, compare the curve with the lower half of the ellipse,
(iii)
^x  :  5.95  x  2 or  1.61  x  0.240`

11
y
(i)
((e,8) y = f(x)

(0,2e2–8)
(e–2, 0) (e+2, 0) x

To find exact x-coordinates of x inte


intercepts:
terc
terceep
rc ep
4  x  e 0
2

x  e
2
4
x  e r2
x er2
To find exact y-coordinates
orrdi
din of y intercept:
2 ª 4  0  e º y
2
¬ ¼
y 2e  82

Page 11 of 16
HCI 2019 H2 Mathematics Promotional Exam Solutions

Qn Solution
NOTE:
­2 ª 4  x  e 2 º ,e  2 d x d e  2
° ¬ ¼
2 4  x  e
2
®
°2 ª 4  x  e º
2
,x  e  2 or x ! e  2
¯ ¬ ¼
11 Since f e  2 f e  2
(ii)
Therefore f is not a one to one function. Hence f 1 does not exist.

y=4

Alternatively,
Since the horizontal line y 4 (give a specific counter-example) intersects y f x more
than once, therefore f is not a one to one function. Hence f 1 does not exist.
11 The function f has an inverse if its domain is restricted to a d x  e .
(iii) Smallest a e  2

Therefore
y 2 x  e  8
2

y
x  e
2
 4
2
y
x  e
2
4
2
y
xe r 4
2
y
x er 4
2
Since e  2 d x  e
y
x e 4
2
x
f 1 : x e 4 , 0 d x 8
2
11 From part (iii),
(iv) e2d x  e
R f 1 D f > e  2, e
Dg \ ^1`

Page 12 of 16
HCI 2019 H2 Mathematics Promotional Exam Solutions

Qn Solution
Since R f 1 Ž Dg , gf 1 exists

ªe  2 e ·
R gf 1 = « , ¸
¬ e 1 e 1 ¹

12 y
 4.5  1.5
(i) 2 y6
Let the angle α and E be such that tan D and
x 2x
y
 4.5  1.5
2 y  12
tan E
x 2x
tan T tan D  E
tan D  tan E
1  tan D tan E
y  6 y  12

2x 2x
§ y  6 · § y  12 ·
1 ¨ ¸¨ ¸ α
© 2x ¹ © 2x ¹ β
12 x
x
4 x  y  6 y  12
2

12 x
4 x2  A
12 Differentiate T implicitly
im
mp
pllic
i it
ittly
ly tthroughout
ly hrou
hr oughou
ghou
ghoutt with
w respect to x
(ii)

Page 13 of 16
HCI 2019 H2 Mathematics Promotional Exam Solutions

Qn Solution
§ dT · 12 4 x  A  12 x 8 x
2

sec T ¨
2
¸ 2
© dx ¹ ª¬ 4 x 2  Aº¼

§ dT · 12 ª¬ A  4 x º¼
2

sec T ¨
2
¸ 2
© dx ¹ ª 4 x 2  A º
¬ ¼

§ dT · 12
sec2 T ¨
A  2x A  2x
¸ 2
© dx ¹ ª¬ 4 x 2  A¼º
dT
For 0,
dx
A
x ( x ! 0)
2

Using 1st Derivative Test below,

x  
A A A
2 2 2
dT +ve 0 -ve
dx
Shape of tangent

A
Thus for maximum T , the player should be m away from the width of the pool
2
12 y 20 or A 112
(iii) 112
maximum T , the player should be 2 7 m away from the width of the pool.
2
12 Using Chain Rule
(iv) 45 dx
Since x 15 , tan T , A 112 an
and
nd 50 ,
253 dt
dT dT dx
u
dt dx dt
12 112  48 15
2

u 50
ª § 4
2 5 · º
45
2
ª 4 15  112
2
º
12 «1  ¨
12
¬ ¼ « © 253 ¸¹ »»
¬ ¼
0.447 rad/min to 3 sf
13 Since L lies on the lines of AA ' and BB ' , we need to find the equation of these 2 lines.
(i)

Page 14 of 16
HCI 2019 H2 Mathematics Promotional Exam Solutions

Qn Solution
§ 2· § 3  2 · § 2· § 5 ·
l AA ' : r = ¨ 3 ¸  O ¨ 13  3 ¸ ¨ ¸ ¨ ¸
¨ ¸ ¨ ¸ ¨ 3 ¸  O ¨ 10 ¸
¨ 3¸ ¨ 83 ¸ ¨ 3¸ ¨5¸
© ¹ © ¹ © ¹ © ¹
§ 4· § 94 · § 4· §5·
lBB ' : r = 3  P ¨ 13  3 ¸
¨ ¸ ¨ ¸ ¨ ¸
¨ ¸ ¨ ¸ ¨ 3 ¸  P ¨ 10 ¸
¨ 4¸ ¨14  4 ¸ ¨ 4¸ ¨ 10 ¸
© ¹ © ¹ © ¹ © ¹
§ 2· § 5 · § 4· §5·
¨ ¸ ¨ ¸ ¨ ¸ ¨ ¸
¨ 3 ¸  O ¨ 10 ¸ ¨ 3 ¸  P ¨ 10 ¸
¨ 3¸ ¨5¸ ¨ 4¸ ¨ 10 ¸
© ¹ © ¹ © ¹ © ¹
1 1
By G.C., O  and P 
5 5
§ 2· § 5 · § 3·
¨ ¸ 1¨ ¸ ¨ ¸
OL ¨ 3 ¸  5 ¨ 10 ¸ ¨1¸
¨ 3¸ ¨ 5¸ ¨ 2¸
© ¹ © ¹ © ¹

13 §0·
(ii) ¨ ¸
p1 : r ˜ ¨ 1 ¸ = k
¨0¸
© ¹
§ 3 · § 0 ·
¨ ¸ ¨ ¸
Since ¨ 13 ¸ ˜ ¨ 1 ¸ 13 , k 13 .
¨ 8 ¸ ¨0¸
© ¹ © ¹
§ 3· § 2  3· § 3· § 1·
lLC r = ¨ 1 ¸  O ¨ 2  1 ¸ ¨ ¸ ¨ ¸
: ¨ ¸ ¨ ¸ ¨1¸  O ¨ 1 ¸
¨ 2¸ ¨5  2¸ ¨ 2¸ ¨3¸
© ¹ © ¹ © ¹ © ¹
§§ 3· § 1· · § 0 ·
¨¨ ¸ ¨ ¸¸ ¨ ¸
¨ ¨ 1 ¸  O ¨ 1 ¸ ¸ ˜ ¨ 1 ¸ 13
¨¨ 2¸ ¨ 3 ¸¸ ¨0¸
©© ¹ © ¹¹ © ¹
1  O 13
O 1
12
§ 3· § 1· § 9 ·
¨ ¸ ¨ ¸ ¨ ¸
OC ' ¨ 1 ¸  12 ¨ 1 ¸ ¨ 13
13 ¸
¨ 2¸ ¨3¸ ¨ 38 ¸
© ¹ © ¹ © 38 ¹

13 Since 0,1053, 0 is on p2 , shortest distance is


(iii)

Page 15 of 16
HCI 2019 H2 Mathematics Promotional Exam Solutions

Qn Solution
§ 03 · §0·
¨ ¸ 1 ¨ ¸ 1044
¨1053  1¸ 17 ¨ 1 ¸ 17
| 253 cm (3 s.f.)
¨ 02 ¸ ¨ 4¸
© ¹ © ¹
13 Solving p1 : y 13 and p2 : y  4 z 1053 ,
(iv)
x x
y 13 .
z 260
§ 0 · §1·
¨ ¸ ¨ ¸
So, l : r ¨ 13 ¸  s ¨ 0 ¸ , where s  .
¨ 260 ¸ ¨ 0 ¸
© ¹ © ¹

Page 16 of 16
+0DWK3URPR1<-&

Questions from 2019 NYJC Promos

1 removed (not in syllabus)


2 Referred to the origin O, three distinct points A, B and C are such that OA a , OB b and
OC c. If OA is perpendicular to BC and OB is perpendicular to CA , show that OC is

perpendicular to AB . [4]

3 It is given that f ( x) ax 3  bx 2  cx  d , where a, b, c and d are constants. The graph y f ( x)

has a minimum point at x 1 and a maximum point at x 3 . The graphs y f ( x ) and

y f ( x) pass through (3, 25) and (1,7) respectively. Find two possible expressions for

f ( x) . [5]

5
4 (i) Using an algebraic method, solve the inequality d 1. [5]
4  2x  x 2
5
(ii) Hence, solve d 1. [2]
4  2 x  x2

5 removed (not in syllabus)

6 In this question, you may use expansions from the List of Formulae (MF 26).
(i) Find the Maclaurin expansion of ln(1  sin 2 x) in ascending powers of x, up to and
S S
including the term in x 4 . State any value(s) of x in the interval  d x d for which
4 4
the expansion is not valid. [4]
(ii) It is given that the first three terms found in part (i) are equal to the first three terms in the

series expansion of ax 1  bx for small x. Find the exact values of the constants a, b
c

and c and use these values to find the coefficient of x 4 in the expansion of ax 1  bx ,
c

giving your answer as a simplified rational number. [5]


7 (a) The diagram below shows the graph with equation y f ( x) . It has a maximum point at
(4, 2) and asymptotes x 2 , y 3 and y 0 .
y

x
y=0 O
(4, –2)

y = –3

x=2

Sketch on separate diagrams, the graphs of


(i) y 3f (2 x  1) , [3]
1
(ii) y . [3]
f ( x)
1
(b) A curve y g( x) undergoes, in succession, the following transformations to get y :
x
1
A: A scaling with scale factor parallel to the y-axis
3
B: A reflection in the y-axis
C: A translation of 1 unit in the direction of the x-axis.
Find g( x) , showing your workings clearly. [3]
8 The functions f, g and h are defined as follows.
f :x ( x  1)(3  x) , x ! k, k  ,
g:x mx  1,
1 x  , m ! 1,
h:x ( x  1)(3  x) , x  .

(i) State the least value of k such that f 1 exists. [1]


Using the value of k found in part (i),

find f x and state the domain of f 1 ,


1
(ii) [4]

(iii) sketch on the same diagram the graphs of y f ( x), y f 1( x) and y ff 1 ( x) . [3]

§1 ·
(iv) On a separate diagram, sketch the graph of y gh ¨ x ¸ , indicating clearly the
©2 ¹
coordinates of the turning point. [2]

9 Planes p1 and p2 have equations x  2 y  z 4 and ax  3 y  2 z 1 respectively, where a is


a constant.
(i) Show that p1 and p2 have a common point (0, 7, 10). [1]

(ii) Find, in terms of a, the vector equation of the line l where p1 and p2 meet. [3]

Plane p3 has equation 2 x  y  2z b , where b is a constant and l has a distance of 5 units

from p3 .

6
(iii) Show that a . [1]
5
(iv) Find the possible values of b. [3]
(v) Find the acute angle between p1 and p2 . [2]

10 A curve D has parametric equations


x = 2t3, y = 3t2.
(i) Find the equation of the tangent to D at the point with parameter t. [2]
(ii) Points P and Q on D have non-zero parameters p and q respectively. The tangent at P
meets the tangent at Q at the point R. Show that the x-coordinate of R is pq p  q , and

find the y-coordinate of R in terms of p and q. Given that pq = 1, show that R lies on
the curve E with equation y  x2  1 . [5]
(iii) It is given that D and E meet at the point M where the x-coordinate of M is positive. Find
the exact coordinates of M. [3]
11 removed (not in syllabus)

12 The diagram shows an open tank made of metal with negligible thickness and a fixed capacity
of 108 m3. The open top EFGH, base ABCD, sides ADHE and BCGF are rectangles, while
ABFE and DCGH are trapeziums with AE perpendicular to AB and EF. The dimensions of the
3
tank are AB = x m, EF = 2x m, AE = x m and EH = y m.
2
H G

E 2x F

D C

A x B
(i) The interior of the tank needs to be treated to prevent rusting. The cost of treating the
interior base is $3 per m2 and the cost of treating the interior sides is $4 per m2. Show

that the total cost of treatment is $ §¨  18 x 2 ·¸ , where K is an exact constant to be


K
© x ¹
determined. [3]
(ii) Find, by differentiation, the values of x and y such that the cost of treatment is minimum
and show that it is less than $420. [5]
(iii) The tank is mounted on a wall and filled with a liquid to maximum capacity. However,
due to poor construction, the liquid begins to leak and spread in a circle of uniform
thickness
2 mm on the floor. Assuming that the liquid is leaking from the tank at a constant rate of
8.4 u107 m3s1 and the leak is discovered one minute after it started, find the rate at which
the radius of the circle of liquid is increasing at this instant, giving your answer in
ms1. [4]
2 Vectors
3 Equations & Inequalities 9 3 27 2 81 157
f ( x)  x  x  x
16 16 16 16
or f ( x)  x  3x  9 x  2
3 2

4 Equations & Inequalities x  1  5 or x 1 or x ! 1  5


(i)
(ii) x  1  5 or x ! 1  5
6 Maclaurin & Binomial 2 4 3 4 4 S
Series i) 2 x  2 x  x  x  ; x z 
3 3 4
1 20
ii) a 2 , c 3 , b ; coefficient of x 4 
3 27
7 Graphs & Transformations 3
(iii) g( x)
1 x
8 Functions (i) k 3
(ii) f 1 x 2  x  1 ; Df 1 (0, f)

9 Vectors § 0 · § 1 ·
¨ ¸ ¨ ¸
(ii) r ¨ 7 ¸  O ¨ 2  a ¸,O 
¨ 10 ¸ ¨ 2a  3 ¸
© ¹ © ¹
(iv) b 12 or b 42
(v) T 8.7q (to 1 dec. pl.)
10 Differentiation & x
Applications (i) y   t2
t
(ii) yR =  pq  q 2  p 2
§ 1 3·
(iii) ¨¨ ,  ¸¸
© 2 2¹
12 Differentiation &
Applications
(i)
K = 48 9  2 13
(ii) x | 2.79m ; y | 6.19m
(iii) 7.46 u104 ms1.
*

NATIONAL JUNIOR COLLEGE


SENIOR HIGH 1
Promotional Examinations
NAME

SUBJECT REGISTRATION
CLASS 1ma2 NUMBER

H2 MATHEMATICS 9758
02 October 2019
3 hours
Candidates answer on the Question Paper.

Additional Materials: List of Formulae (MF26)


Writing Paper

READ THESE INSTRUCTIONS FIRST Question Marks Marks


Number Possible Obtained
This paper constitutes 50% of your overall score for SH1 H2
Mathematics. 1 4

Write your name, class and registration number in the boxes above. 2 4
Please write clearly and use capital letters.
3 6
Write in dark blue or black pen.
You may use an HB pencil for any diagrams or graphs. 4 7
Do not use paper clips, glue or correction fluid.
5 7
Answer all the questions.
Write your answers in the spaces provided in the question paper. 6 8
Give non-exact numerical answers correct to 3 significant figures, or
1 decimal place in the case of angles in degrees, unless a different 7 8
level of accuracy is specified in the question.
The use of an approved graphing calculator is expected, where 8 8
appropriate.
Unsupported answers from a graphing calculator are allowed unless 9 8
a question specifically states otherwise.
Where unsupported answers from a graphing calculator are not
10 14
allowed in a question, you are required to present the mathematical
steps using mathematical notations and not calculator commands.
11 14
You are reminded of the need for clear presentation in your answers.
Up to 2 marks may be deducted for improper presentation. 12 12

The number of marks is given in the brackets [ ] at the end of each Presentation Deduction – 1 / –2
question or part question.
TOTAL 100

This document consists of 31 printed pages and 1 blank page.

© NJC 2019 [Turn_over


2

Given that f ( x ) = tan §¨ − x ¸· , find f ( 0 ) , f ′ ( 0 ) and f ′′ ( 0 ) . Hence write down the first three
ʌ
1
©4 ¹
non-zero terms in the Maclaurin series for f ( x ) . [4]

4
The function f ( x ) is a quadratic polynomial such that f ( 4 ) = 3 and ´
20
2 µ f ( x ) dx = . If the
¶0 3
curve with equation y = f ( x ) is transformed by a translation of 1 unit in the negative x-
3
direction, the new curve has a y-intercept at . Find f ( x ) . [4]
2

3 The diagram below shows the curve with equation y = f ( x ) . It has turning points A ( a, 5 ) and
1
B (1, 9 ) and asymptotes with equations y = 2, x = m and y = x + 7 . The curve also crosses
3
the axes at the points C ( c, 0 ) and D ( 0, d ) . The gradient of the curve at D is −3 .

Sketch, on separate diagrams, the following curves.


1
(a) y = [3]
f ( x)
(b) y = f ′( x) [3]
Label the coordinates of the points corresponding to A, B, C, and D (where applicable), the
points where the curve crosses the axes, and the equations of any asymptotes.

© NJC 2019
3

4 A curve C has equation y = e x ͘


(a) Find the exact volume of revolution when the region bounded by C, the line y = 3 and
the y-axis is rotated completely about the y-axis. [5]
(b) Describe a pair of transformations which transforms the curve with equation y = e1+ 2 x on
to the curve C. [2]

5 A curve C has parametric equations


x = sin θ , y = sin 2θ , for 0 ≤ θ ≤ ʌ.
(i) Sketch C, labelling the coordinates of any axial intercepts. [2]
(ii) Find the exact area of the region bounded by C. [5]

6 A vessel is formed by removing a smaller cone of radius 5 m from a bigger cone whose semi-
vertical angle is α , where tan α = 0.5 . Water flows out of the vessel at a rate of k h m 3 per
minute, where k is a positive constant. At time t minutes, the height of the water surface from
the hole is h m (see diagram).

1 ª
ʌ ( h + 10 ) − 1000º .
3
(i) Show that the volume of the water V, in m3 , is given by [4]
12 ¬ ¼
(ii) Find the rate of change of h, in terms of k, when V = 120ʌ . [4]

7 Determine the constants A and B such that


1 A B
= 2 + 2 .
( 3x + 1)( x + 3) 3x + 1 x + 3
2 2

Hence, find the exact value of p, where p > −1 , such that


p
´
1
´ 8 3
µ dx = µ x dx ,
¶0 ( 3 x + 1)( x + 3)
2 2
¶−1 9
giving your answer in terms of ʌ . [8]

8 A curve C has equation ln ( y + 1) = 1 + tan −1 x , where y > −1 .

(i) Explain why C has no tangent parallel to the y-axis. [2]

(ii) Without using a calculator, find the equation of the normal to C at the point where it
crosses the y-axis. [3]

(iii) Determine the range of values of x for which C is concave downwards. [3]

© NJC 2019 [Turn_over


4

9 Non-zero and non-parallel vectors a, b and c are such that b × 3c = c × a .

(i) Determine the relationship between c and a + 3b , justifying your answer. [2]

It is given that a and 3b are unit vectors and that the angle between a and b is 60° .

(ii) Evaluate a + 3b . [3]

(iii) Given further that a + 3b makes an angle of 60° , 120° and 135° with the positive x-, y-
and z-axes respectively, show that c is parallel to i − j − 2k .
[3]

10 Functions f and g are defined by


8
f :x 6 2− x+ , x ∈ , x ≠ −2, x > k ,
x+2
g : x 6 x 2 − 6 x + a , x ∈ , x > 0,
where a is a constant.

(i) State the least value of k for which the function f −1 exists. [1]

Using this value of k,

(ii) Without finding f −1 , sketch, on the same diagram, the graphs of y = f ( x ) , y = f −1 ( x )


and y = f −1f ( x ) , showing clearly their geometrical relationship. State the equations of
any asymptotes. [4]

(iii) Find the smallest integer value of a for which the composite function fg exists and use
this value to state the range of fg. [4]

(iv) Given instead that a = 10 , solve the inequality fg ( x ) + g ( x ) ≤ 4 algebraically. [5]

11 The plane p passes through the points A, B and C with coordinates ( −3, − 4, 11) , (1, − 2, 0 ) and
( −5, 2, − 1) respectively. The point M has position vector given by m = 3i + 6 j + 15k .

JJJJG
(i) Show that AM is perpendicular to p. [3]

(ii) Find the coordinates of N which is the mirror image of M in p. [2]

(iii) Find a vector equation of the line which is a reflection of the line MB in p. [2]

The plane q has equation 11x + 6 y − z − 4 + k ( − x − 3 y + z − 4 ) = 0 for some constant k.

(iv) Given that q contains M, find a cartesian equation of q. [2]

(v) By finding the line of intersection between p and q, or otherwise, find a cartesian equation
of the plane which is a reflection of q in p. [5]

© NJC 2019
5

x2
12 It is given that f ( x ) = 9 −
. The diagram below shows a vertical cross section of a building.
6
The cross section of the roof of the building can be modelled by the curve y = f ( x ) , where
−6 ≤ x ≤ 6 and x is in metres. The ground level is represented by the x-axis.

The cross section of the living space under the roof can be modelled by a rectangle ABCD with
points D ( − a, 0 ) and A ( a, 0 ) , where 0 < a ≤ 6 .

The P/A ratio is a measure of the thermal insulation of a space. The lower the P/A ratio, the
smaller the amount of insulation is required, thus saving costs. The P/A ratio can be defined by
P (a)
the function I ( a ) = where P ( a ) is the perimeter and A ( a ) is the area of the rectangle
A(a)
ABCD.

12 1
(i) Show that I ( a ) = + . [3]
54 − a a
2

(ii) Given that a = a1 is the value of a which gives the minimum value of I, show that a1
3
satisfies the equation a 2 + 24a 2 − 54 = 0 . [3]
(iii) Find a1 , correct to 3 decimal places, and show that it minimises I. [2]

The developer would prefer the living space ABCD to be at least 80% of the area ABECD,
which is currently not satisfied. It is suggested to replace the curve y = f ( x ) to model the roof
with part of the curve in part (iv).

(iv) Sketch the curve with equation ( 2 y − 19 ) − 2 x 2 = 1 for y ≤ 9 for all values of x, stating
2

the equations of any asymptotes. [2]

(v) Based on the value of a1 found in part (iii), determine if the developer would accept the
suggestion. [2]

© NJC 2019 *
National Junior College Mathematics Department

Qn Suggested Solutions
1 §ʌ ·
f ( x ) = tan ¨ − x ¸ Ÿ f ( 0) = 1
©4 ¹
§ʌ ·
f ′ ( x ) = − sec2 ¨ − x ¸ Ÿ f ′ ( 0 ) = −2
©4 ¹
§ʌ · §ʌ ·
f ′′ ( x ) = 2sec2 ¨ − x ¸ tan ¨ − x ¸ Ÿ f ′′ ( 0 ) = 4
©4 ¹ ©4 ¹
2
x
f ( x ) = 1 + ( −2 ) x + ( 4 ) + !
2!
= 1− 2x + 2x +!
2

2 Let f ( x ) = ax 2 + bx + c
f ( 4) = 3
a ( 4) + b ( 4) + c = 3
2

16a + 4b + c = 3 " (1)

4 20
³ 0
ax 2 + bx + c dx =
3
4
ªa 3 b 2 º 20
«¬ 3 x + 2 x + cx »¼ = 3
0

64 20
a + 8b + 4c = " (2)
3 3

When translated 1 unit in the direction of the negative x-direction, the equation of curve
becomes
y = a ( x + 1) + b ( x + 1) + c .
2

he y-int
At tthe -intercept,
3
a+b+c = " ( 3)
2
1
Solving
Sol
oollvi
ving (1),
ving (1)
1), (2)
(2) an
aand (3) , a = , b = −2, c = 3 .
d (3
2
1 2
Hence f ( x) = x − 2 x + 3
He
2

2019 / SH1 H2 Maths / Promos / Suggested Solutions Page 1 of 13


National Junior College Mathematics Department

Qn Suggested Solutions
3(a)

3(b)

4(a) y = e x Ÿ x = ln y

³ ( ln y )
2
dy
´ § 2 ln
ln y ·
= y ( ln y ) − µ
2
y¨ ¸ dy
¶ © y ¹
= y ( ln y ) − 2 ³ ln
2
ln y dy
ª ´ §1· º
= y ( ln y ) − 2 « y ln y − µ y ¨ ¸ dy »
2

¬ ¶ © y¹ ¼
= y ( ln y ) − 2 y ln y + 2 y + c
2

volume of the solid formed


= ʌ ³ ( ln y ) dy
3 2
1
3
= ʌ ª y ( ln y ) − 2 y ln y + 2 y º
2
¬ ¼1
= ʌ ª3 ( ln 3) − 6 ln 3 + 6 − 2 º
2
¬ ¼
= ʌ ª3 ( ln 3) − 6 ln 3 + 4 º units3
2
¬ ¼

2019 / SH1 H2 Maths / Promos / Suggested Solutions Page 2 of 13


National Junior College Mathematics Department

Qn Suggested Solutions
1+ 2 x
4(b) y=e
x
↓ Replace x by
2
y = e1+ x
↓ Replace x by x − 1
y = ex

1. Scaling by a factor of 2 parallel to the x-axis.


2. Translation of 1 unit in the positive x-direction.

Alternatively
y = e1+ 2 x
1
↓ Replace x by x −
2
y = e2 x
x
↓ Replace x by
2
y = ex

1
1. Translation of unit in the positive x-direction.
2
2. Scaling by a factor of 2 parallel to the x-axis.

5(i) y




x


2019 / SH1 H2 Maths / Promos / Suggested Solutions Page 3 of 13


National Junior College Mathematics Department

Qn Suggested Solutions
5(ii) When x = 0, When x = 1,
sin θ = 0 sin θ = 1
θ = 1. ʌ
θ= .
2
dx
= cos θ .

Method 1: Using the form f ' ( x ) ª¬ f ( x ) º¼ .
n

Area of region bounded by C


ʌ
= 2 ³ 2 sin 2θ cos θ dθ
0
ʌ
= 2 ³ 2 2sin θ cos 2 θ dθ
0
ʌ
= −4 ³ 2 ( − sin θ ) cos 2 θ dθ
0
ʌ
ª cos3 θ º 2
= −4 « »
¬ 3 ¼0
§ 1·
= −4 ¨ 0 − ¸
© 3¹
4
= units 2
3

Method 2: Using the factor formula.


Area of region bounded by C
ʌ
= 2 ³ 2 sin 2θ cos θ dθ
0
ʌ
= ³ sin 3θ + sinn θ dθ
2
0
ʌ
cos 3θ
ª cos
co º2 
= «− coos θ »
− cos
¬ 3 ¼0
§ 1 ·
= 0 − ¨ − − 1¸
© 3 ¹
4
= units 2
3











2019 / SH1 H2 Maths / Promos / Suggested Solutions Page 4 of 13


National Junior College Mathematics Department

Qn Suggested Solutions
6(i) Let h′ be the height of the smaller cone
5
tan α = 0.5 =
h′
Ÿ h′ = 10
Let R and H be the radius and height of the bigger cone respectively.
R
tan α = 0.5 =
H
Ÿ H = 2R
Also, we have H = h + 10
1 1
V = ʌR 2 H − ʌ ( 5 ) (10 )
2

3 3
= ʌ ( R 2 H − 250 )
1
3
1 ª§ H · º
2

= ʌ «¨ ¸ H − 250 »
3 «¬© 2 ¹ »¼
1 § H3 ·
= ʌ¨ − 250 ¸
3 © 4 ¹
ʌ ( H 3 − 1000 )
1
=
12
1
= ʌ ª( h + 10 ) − 1000 º
3

12 ¬ ¼

6(ii) dV 1
= ʌ ª3 ( h + 10 ) º
2

dh 12 ¬ ¼

ʌ
= ( h + 10 )
2

4
dV dV dh
= ×
dt dh dt
1 dh
− k h = ʌ ( h + 10 ) ×
2

4 dt
dh −4 k h
=
dt ʌ ( h + 10 )2
1 ª
ʌ ( h + 10 ) − 1000 º = 120ʌ
3
When V = 120ʌ , ͘
12 ¬ ¼
( h + 10 ) − 1000 = 1440
3

( h + 10 ) = 2440
3

h + 10 = 3 2440
h = 3 2440 − 10
dh −4k 3
2440 − 10
When h = 3 2440 − 10 , =
( )
2
dt ʌ 3
2440
= −0.0131k m/min (to 3 s.f.)

2019 / SH1 H2 Maths / Promos / Suggested Solutions Page 5 of 13


National Junior College Mathematics Department

Qn Suggested Solutions
7 1 A ( x + 3) + B ( 3 x 2 + 1)
2

=
( 3x + 1)( x2 + 3)
2
( 3x 2
+ 1)( x 2 + 3)
Comparing coefficients,
A + 3 B = 0 (1) and 3 A + B = 1 (2)
3 1
Solving (1) and (2), A = and B = −
8 8
1
´ 8
µ dx
¶0 ( 3 x + 1)( x + 3)
2 2

1

3 1
µ − 2 dx
¶0 3 x + 1 x + 3
2

1
´ 1 1
dx − ´
1
=µ µ 2 dx
µ 2 1 ¶ x + 3
¶0 x + 0
3
1
´ ´
1

= µ 1
dx − µ
1
dx
µ 2
µ 2 § 1 ·
( )
2 2

¶0 x + ¨ ¶0 x + 3
¸
© 3¹
1
ª 1 § x ·º
( )
1
= ª 3 tan −1 3x º − « tan −1 ¨ ¸»
¬ ¼0 ¬ 3 © 3 ¹¼ 0
1
ª 1 § 1 · º
( )
1
= ª 3 tan −1 3 − 0º − « tan −1 ¨ ¸ − 0»
¬ ¼0 ¬ 3 © 3 ¹ ¼0
ʌ 3 1 §ʌ·
= − ¨ ¸
3 3©6¹
5 3ʌ
=
18
18
3
Consider
Cons
Cons
Co idder the
nside graaph ooff y =
he graph
the x
9

2019 / SH1 H2 Maths / Promos / Suggested Solutions Page 6 of 13


National Junior College Mathematics Department

Qn Suggested Solutions
3
Observe that area of the triangle bounded by y = − x , the negative x-axis and x = −1
9
is
1 § 3· 3 5 3ʌ
(1) ¨¨ ¸¸ = < . Thus p > 0 .
2 © 9 ¹ 18 18

Hence,
5 3ʌ 3 1 § 3 ·
= + ( p ) ¨¨ p ¸¸
18 18 2 © 9 ¹
5ʌ = 1 + p 2 
p = 5ʌ − 1

8(i) Differentiating ln ( y + 1) = 1 + tan −1 x w.r.t. x, we get


1 dy 1
=
y + 1 dx 1 + x 2
dy y + 1
= .
dx 1 + x 2

dy
Since 1 + x 2 > 0 for all real values of x, thus is defined for all values of x and y.
dx
Therefore, C has no tangent parallel to the y-axis.

8(ii) At x = 0 , we have ln ( y + 1) = 1 + tan −1 0


Ÿ y = e − 1, and
dy e − 1 + 1
Ÿ = =e.
dx 1 + 02
en the ggradient
Then radi
radiennt of m at point where x = 0 is −e−1 .
of nnormal
o ma
or

Theerrref
Th effoorre tth
Therefore he eq
he
the qua
uatiion ooff no
equation nnormal
r at point where x = 0 is
Ÿ y − ( e − 1) = ( −e ) ( x − 0 )
−1

Ÿ y = −e −1 x + e − 1

2019 / SH1 H2 Maths / Promos / Suggested Solutions Page 7 of 13


National Junior College Mathematics Department

Qn Suggested Solutions
8(iii) d2 y
Where C is concave downwards, <0.
dx 2
dy y + 1
=
dx 1 + x 2

d2 y (
1 + x 2 ) − ( y + 1) 2 x
dy
Ÿ 2 = dx
(1 + x2 )
2
dx

d 2 y ( y + 1) − ( y + 1) 2 x ( y + 1)(1 − 2 x )
Ÿ = = <0
(1 + x2 ) (1 + x2 )
2 2
dx 2

Ÿ (1 − 2 x ) < 0
1
Ÿx>
2

9(i) b × 3c = c × a
Ÿ b × 3c − c × a = 0
Ÿ 3b × c + a × c = 0
Ÿ ( 3b + a ) × c = 0

Hence c is parallel to a + 3b .

9(ii) a + 3b = ( a + 3b ) ⋅ ( a + 3b )
2

= a ⋅ a + 6a ⋅ b + 9b ⋅ b
2 2
= a + 9 b + 6a ⋅ b
= a + 9 b + 6 ( a b cos 60° )
2 2

§ a 1·
= 1+1+ 6¨ a ¸=3
© 3 2¹
Thus
T
Th us a + 3b = 3 .
us
9(iii)
99(
(ii
(ii
iii)
i Since
Si
S c a + 3b m
inncce
inc makes
akes an angle of 60° , 120° and 135° with the x-, y- and z-axes
ak
respectively,
re
esp
spececti
ect ve
tiv ly then we have
§1 ·
¨ ¸
- ( a + 3b ) • ¨ 0 ¸ = a + 3b 1 cos 60°
¨0¸
© ¹
§0·
- ( a + 3b ) • ¨¨1 ¸¸ = a + 3b 1 cos120°
¨0¸
© ¹
§0·
- ( a + 3b ) • ¨¨ 0 ¸¸ = a + 3b 1 cos135°
¨1 ¸
© ¹

2019 / SH1 H2 Maths / Promos / Suggested Solutions Page 8 of 13


National Junior College Mathematics Department

Qn Suggested Solutions
The LHS of all 3 expressions simplify to be the x-, y- and z-components of a + 3b
respectively.
With the information of part (ii), the RHS of the expressions simplify to
3 3 3
3 cos 60° = , 3 cos120° = − and 3 cos135° = − .
2 2 2
§ 3 ·
¨ ¸
¨ 2 ¸ § 1 ·
¨ 3¸ 3¨ ¸
Therefore a + 3b = ¨ − ¸= ¨ −1 ¸
¨ 2 ¸ 2 ¨ ¸
¨ 3¸ ©− 2 ¹
¨¨ − ¸
© 2 ¸¹
§ 1 ·
3¨ ¸
Since c is parallel to a + 3b which can be expressed as ¨ −1 ¸ , then c is parallel to
2 ¨ ¸
©− 2¹
i − j − 2k .

10(i) Minimum value of k = −2

10(ii)

2019 / SH1 H2 Maths / Promos / Suggested Solutions Page 9 of 13


National Junior College Mathematics Department

Qn Suggested Solutions
10(iii) g( x) = x − 6 x + a
2

= ( x − 3) + ( a − 9 )
2

Rg = [ a − 9, ∞ ) Df = ( −2, ∞ )
For fg to exist, Rg ⊆ Df
a − 9 > −2
a>7
Thus, smallest integer a = 8
When a = 8 , Rg = [ −1, ∞ )
8
f (−1) = 2 − ( −1) + = 3 + 8 = 11
−1 + 2
Hence range of fg is ( −∞,11] .

10(iv) fg( x) + g( x ) ≤ 4
8
2 − g ( x) + + g ( x) ≤ 4
g ( x) + 2
8
2− ≥0
g ( x) + 2
Since g ( x ) + 2 = x 2 − 6 x + 12 = ( x − 3) + 3 > 0 for all real values of x,
2

2 ( x 2 − 6 x + 12 ) − 8 ≥ 0
2 x 2 − 12 x + 16 ≥ 0
x2 − 6 x + 8 ≥ 0
( x − 2 )( x − 4 ) ≥ 0
∴ x ≤ 2 or x≥4
he domai
In addition, the ainn of g is x > 0 .
ai
domain
Thus,, the range of x iss 0 < x ≤ 2 or x ≥ 4 
r ngge of
ra

11(i)
11
1(i
(i) A ( −33,, − 44,, 111) , B (1,
1 − 22,, 0 ) , C ( −5, 2, − 1) , M(3, 6, 15)
JJJJG
A = 4i + 2 j − 11
AB 1k
JJJJ
JJJJG
BC = −6i −6i + 4 j − k
JJ
JJJG
AC = −2i + 6 j − 12k
JJJJG
AM = 6i + 10 j + 4k
§6· § 4 · § 6 · § −6 ·
JJJJG JJJG ¨ ¸ ¨ ¸ JJJJG JJJG ¨ ¸ ¨ ¸
AM • AB = ¨10 ¸ • ¨ 2 ¸ = 24 + 20 − 44 = 0 AM • BC = ¨10 ¸ • ¨ 4 ¸ = −36 + 40 − 4 = 0
¨ 4 ¸ ¨ −11¸ ¨ 4 ¸ ¨ −1 ¸
© ¹ © ¹ © ¹ © ¹
JJJJG JJJJG
Since AM is perpendicular to 2 of the vectors lying on p, then AM is perpendicular to
p.

2019 / SH1 H2 Maths / Promos / Suggested Solutions Page 10 of 13


National Junior College Mathematics Department

11(ii) By ratio theorem,


JJJG 1 JJJJG JJJG JJJG JJJG
( )
OA = OM + ON or AN = MA
2
M

§ −3 · § 3 · § −9 · p
JJJG ¨ ¸ ¨ ¸ ¨ ¸
ON = 2 ¨ −4 ¸ − ¨ 6 ¸ = ¨ −14 ¸ A
¨ 11 ¸ ¨ 15 ¸ ¨ 7 ¸
© ¹ © ¹ © ¹
N
N(–9, –14, 7)

11(iii) § −9 · § 1 · § −10 ·
JJJG ¨ ¸ ¨ ¸ ¨ ¸
BN = ¨ −14 ¸ − ¨ −2 ¸ = ¨ −12 ¸ M
¨ 7 ¸ ¨ 0¸ ¨ 7 ¸
© ¹ © ¹ © ¹ p
B
A
The equation of the reflected line is
§ 1· § 10 · N
¨ ¸ ¨ ¸
r = ¨ − 2 ¸ + λ ¨ 12 ¸ , λ ∈  .
¨ 0¸ ¨ −7 ¸
© ¹ © ¹

11(iv) Since M lies on q, we can sub x = 3, y = 6, z = 15 into q


§ 3 · § 11 − k ·
¨ ¸ ¨ ¸
¨ 6 ¸ • ¨ 6 − 3k ¸ = 4k + 4 or 11x + 6 y − z − 4 + k ( − x − 3 y + z − 4 ) = 0
¨ 15 ¸ ¨ −1 + k ¸
© ¹ © ¹
33 + 36 − 15 − 4 + k ( −3 − 18 + 15 − 4 ) = 0
50 − 10k = 0
k =5
quati of q is 6 x − 9 y + 4 z = 24
Sub k = 5 and the equation
JJJG
JJJJ
11(v) nce AM
Since AM iiss perpendicular
perrppen
pe e di
dicu
cula
cu l r to p, the equation of p is
3x + 5 y + 2 z = −7
U
Us
Usin
sinng GC,
Using GC, we
GC we can
an find
fin
indd the
in the line
lin of intersection of p and q to be
§ ·
1 −
§ 3·2
§1· § −2 ·
¨ ¸ ¨ ¸ ¨ ¸ ¨ ¸
r = ¨ −2 ¸ + α ¨ 0 ¸ or r = ¨ −2 ¸ + α ¨ 0 ¸ .
¨ 0¸ ¨ 1 ¸ ¨ ¸ ¨ ¸
© ¹ © ¹ © 0¹ © 3¹
§ −10 ·
JJJG ¨ ¸
Note: B(1, –2, 0) is on line of intersection of p and q, so NB = ¨ −12 ¸ and N(–9, –14, 7)
¨ 7 ¸
© ¹
reflection of M is on q.
The vector equation of the reflected plane is
§1· § −2 · § 10 ·
¨ ¸ ¨ ¸ ¨ ¸
r = ¨ −2 ¸ + α ¨ 0 ¸ + β ¨ 12 ¸
¨ 0¸ ¨ 3¸ ¨ ¸
© ¹ © ¹ © −7 ¹

2019 / SH1 H2 Maths / Promos / Suggested Solutions Page 11 of 13


National Junior College Mathematics Department

§ −2 · § 10 · § 0 − 36 · § −36 · §9 ·
¨ ¸ ¨ ¸ ¨ ¸ ¨ ¸ ¨ ¸
¨ 0 ¸ × ¨ 12 ¸ = ¨ 30 − 14 ¸ = ¨ 16 ¸ = −4 ¨ −4 ¸ q
¨ 3 ¸ ¨ −7 ¸ ¨ −24 − 0 ¸ ¨ −24 ¸ ¨ 6¸ M(3, 6, 15)
© ¹ © ¹ © ¹ © ¹ © ¹
§9· §1· §9·
¨ ¸ ¨ ¸ ¨ ¸ p
r • ¨ −4 ¸ = ¨ −2 ¸ • ± ¨ −4 ¸ = 9 + 8 + 0 B(1,–2,0)
¨ 6¸ ¨ 0¸ ¨ 6¸
© ¹ © ¹ © ¹ § −10 ·
¨ ¸

§9· N ¨¨ −712 ¸¸
© ¹
¨ ¸
r • ¨ −4 ¸ = 17
¨ 6¸
© ¹
A cartesian equation of the reflected plane is 9 x − 4 y + 6 z = 17 .

12(i) 2 ( 2a + f ( a ) )
I (a) =
2 af ( a )
2a + f ( a )
=
af ( a )
2a f (a)
= +
af ( a ) af ( a )
2 1
= +
f (a) a
2 1
= +
a2 a
9−
6
12 1
= +
54 − a a
2

12(ii) dI (−1)(12)(
( 2)(−2a) 1
1)(1
)(12)(
= − 2
( 54 − a 2 ) a
2
dda
a
dI
Wh
Wheenn
When = 0 , we have
hav
ave
da
(−11)(12 −2a ) 1
1)(12)(
)(12)(
)(
(12)(
12
= 2 
( 54 − a 2 ) a
2

24a 3 = ( 54 − a 2 )
2

3
24a 2 = 54 − a 2 (54 − a 2 > 0 ' a 2 ≤ 36)
3
a 2 + 24a 2 − 54 = 0






2019 / SH1 H2 Maths / Promos / Suggested Solutions Page 12 of 13


National Junior College Mathematics Department

3
12(iii)
a 2 + 24a 2 − 54 = 0
Using GC, a = 3.947
3 3.947 4
dI 17 23
− 0
da 225 5776
Shape \ _ /
By the first derivative test, 3.947 gives the minimum value of I.

Alternatively,
d2 I
By the second derivative test, = 0.0752 > 0 , thus 3.947 gives the minimum
da 2 x = 3. 947
value of I.

12(iv)

12(v) § 19 − 1 + 2 ( 3.947 )2 ·
2 × 3.947 × ¨ ¸
¨ 2 ¸
area of ABCD
= © ¹
3.947
area of ABECD ´ 19 − 1 + 2 x 2

µ dx
¶−3.947 2
52.61047582
52.6
52 .61047
.6
=
62.94592466
622.9
.945
4592
45 9 4
= 0.835804321
0 835
0.
This
T iiss iimplies
Th mpli
mplies
li that the living space takes up 83.6%, which satisfy the condition, thus the
e tha
developer
devvelo
de l p will take up the suggestion.

2019 / SH1 H2 Maths / Promos / Suggested Solutions Page 13 of 13


-&+0DWK3URPR([DP3DSHU5,

+0DWKHPDWLFV3URPRWLRQ([DPLQDWLRQ6ROXWLRQVIRU6WXGHQWV

 )LQG WKH HTXDWLRQ RI WKH WDQJHQW WR WKH FXUYH y    xy   x     DW WKH SRLQW   
>@
 y   xy   x   
  

>@ $SSO\SURGXFWUXOHDQGFKDLQ

'LIIHUHQWLDWLQJLPSOLFLWO\ZUWx
UXOHWRGLIIHUHQWLDWHxy
Gy § Gy ·
 y   ¨ y    xy ¸   x 
Gx © Gx ¹ $SSO\FKDLQUXOHWR
:KHQx  y  GLIIHUHQWLDWHy
Gy § Gy ·
 
  ¨      ¸    
Gx © Gx ¹ 5HDGTXHVWLRQ
Gy Gy FDUHIXOO\ILQGWKH
  Ÿ  HTXDWLRQRIWKH
Gx Gx
WDQJHQWOLQHQRW
7KHHTXDWLRQRIWKHWDQJHQWLV y    x    ILQGJUDGLHQWDW
LH y x   RQO\

 $Q DULWKPHWLF VHULHV KDV ILUVW WHUP a DQG FRPPRQ GLIIHUHQFH d ZKHUH a DQG d DUH
QRQ]HUR7KHVWDQGUGWHUPVRIWKHDULWKPHWLFVHULHVDUHDQGUHVSHFWLYHO\*LYHQ
DOVRWKDWWKHVXPRILWVILUVWnWHUPVLV)LQGWKHYDOXHVRIadDQGn >@
 n
Sn ª a  n   d º¼  
>@ ¬ un a  n   d
n d   an  nd       

n
Sn >a  n   d @ 
a  d    
a  d     

6ROYLQJ  DQG  XVLQJ*&a DQG d   1RQHHGWRVROYHWKH


6XEaDQGdLQWR   HTXDWLRQVE\KDQG
 n    n    n     8VH*&WRHYDOXDWHa
dDQGn
n    n   

8VLQJ*&n  ±QRWDFFHSWHGDVn! 

 7KHVHTXHQFH u  u  u   LVJLYHQE\ un WDQ n   WDQ n   IRU n t  

L WDQ A  WDQ B
%\FRQVLGHULQJ WDQ A  B VKRZWKDW
  WDQ A WDQ B
WDQ n    WDQ n  
 u n    >@
WDQ

LL  n
+HQFHILQG ¦ u r LQWHUPVRIn >@
r 
 L  un WDQ n   WDQ n    7KLVLVD6+2:TXHVWLRQVR
>@ GHWDLOHGZRUNLQJVKRXOGEH
WDQ n    WDQ n  
WDQ n    n   VKRZQ6KRZH[SOLFLWO\
  WDQ n   WDQ n   WDQ A  B
WDQ n    WDQ n   WDQ n    n   
WDQ 
  un  WDQ   WDQ
  un WDQ  WDQ n    WDQ n   RU
WDQ A  B
WDQ n    WDQ n  
un   VKRZQ WDQ n    n   
WDQ
WDQ
GHSHQGLQJRQZDVFKRVHQIRU
ADQGB
LL  n n
ª WDQ r    WDQ r   º 6XIILFLHQWZRUNLQJQHHGVWR
>@ ¦ ur ¦« ¬ WDQ
 »  EHVKRZQ ILUVWDQGODVW
r  r ¼ URZV 
 n n

¦
WDQ r 
ª¬ WDQ r    WDQ r   º¼  ¦ 
r 
7KHUHLVQRQHHGWRHYDOXDWH
 WDQDQGWDQ
> WDQ   WDQ  1RWHWKHYDOXHVDUH
WDQ HYDOXDWHGLQUDGLDQVLQVWHDG
 WDQ   WDQ  RIGHJUHHVR
 WDQ   WDQ   WDQ  

 WDQ 
 WDQ n    WDQ n   
$OWHUQDWLYHPHWKRG
 WDQ n    WDQ n   @  n   n

WDQ n    WDQ 
¦u
r 
r

 n
WDQ n
§ WDQ   WDQ  ·
¦u
r 
r ¨
© WDQ 
 ¸
¹


 ,WLVJLYHQWKDW I x OQ   H x :LWKRXWXVLQJDFDOFXODWRUILQGWKH0DFODXULQVHULHVIRU
I x XSWRDQGLQFOXGLQJWKHWHUPLQ x    >@
 I x OQ   H x
>@
 Hx 
I
x  
 H x
  Hx
Hx
I

x
  H x 

 
:KHQ x  I  OQ  I
  I

 
 

  
7KXVWKH0DFODXULQVHULHVIRU I x LV OQ   x   x    OQ   x  x    
   

Alternative:
/HWy OQ   H x
H y   Hx
Gy
Hy Hx 
Gx

G  y y § Gy ·
Hy H ¨ ¸ Hx
Gx  © Gx ¹
Gy  G y 
:KHQ x  y OQ   
Gx  Gx  

  
7KXVWKH0DFODXULQVHULHVIRU I x LV OQ   x   x    OQ   x  x   
   

Alternative:
8VLQJWKHVWDQGDUGVHULHVH[SDQVLRQVRI H x DQG OQ   x 
I x OQ   H x
§ § x ··
OQ ¨   ¨  x    ¸ ¸ 
© ©  ¹¹
§ § x x 
··
OQ ¨  ¨      ¸ ¸
© ©   ¹¹
§ x x 
·
OQ   OQ ¨     ¸
©   ¹

§ x x ·  § x x ·
OQ   ¨    ¸  ¨    ¸  
©  ¹ ©   ¹ 

x x  § x ·
OQ     ¨ ¸  
  ©¹
x x
OQ     
 

 1RWH:KHQXVLQJWKHVWDQGDUGVHULHVWKHIROORZLQJZLOOnotZRUN


 OQ   H
x
H x

H  H
x  x 
§ x · § x ·


  ¨  x    ¸  ¨   x    ¸   


   ©  ¹ ©  ¹

 
§ x
x · § x ·
 OQ   H OQ     x 
x
  ¨   x    ¸  ¨   x    ¸   
  ©  ¹ ©  ¹

 ,QHDFKH[SDQVLRQDOOWKHEUDFNHWVKDYHDFRQVWDQWWHUP x WHUP x  WHUPHWF
 VRZHZLOOQRWEHDEOHWRFROOHFWDOOWKHUHVSHFWLYHWHUPVWRWDOO\



 *LYHQD EDQG FDUHXQLWYHFWRUVVXFKWKDWD˜E D˜FZKDWLVWKHUHODWLRQVKLSEHWZHHQD
DQGEF" >@
 
 ,WLVIXUWKHUJLYHQWKDWELVSHUSHQGLFXODUWREF)LQGWKHDQJOHEHWZHHQEDQGF >@
 
 &RPPHQWRQWKHUHODWLRQVKLSEHWZHHQDuEDQGEF >@
 D ˜ E D˜ F 'RWSURGXFWLV
>@ GLVWULEXWLYHLH
D ˜ E  D˜ F  
D ˜ E  D˜ F D ˜ E  F
D ˜ E  F 
1RWH E z F DVEDQGF
7KHUHIRUHDLVSHUSHQGLFXODUWR E  F VLQFH D z  DQG E z F  DUHXQLWYHFWRUV
>@ /HW T EHWKHDQJOHEHWZHHQEDQGF 
*LYHQELVSHUSHQGLFXODUWR E  F  E ˜ E  F   

E ˜ E E ˜ F


E  E F FRV T 5HDGTXHVWLRQ
FDUHIXOO\WRPDNHXVH
 FRVT   VLQFHE DQGF DUHXQLWYHFWRUV  RI_E_ _F_ 

ʌ
T UDGLDQV

>@ 6LQFHDDQGEDUHSHUSHQGLFXODUWRE±FDQG D u E LV 
 SHUSHQGLFXODUWRDDQGE D u E LVSDUDOOHOWRE±F͘

6RPHWKLQJVWRQRWH

 x X A ZDQGY A ZGRHVnotLPSO\XDQGYDUHSDUDOOHO HJL A NDQGM A NEXW
 LDQGMDUHQRWSDUDOOHO 
 x :HGRQRWKDYHVXIILFLHQWLQIRUPDWLRQWRFRQFOXGHZKHWKHUWKHYHFWRUVDUH
 LQWKHVDPHGLUHFWLRQRULQWKHRSSRVLWHGLUHFWLRQRUDUHHTXDO:HFDQ
 RQO\FRQFOXGHWKDWWKHYHFWRUVDUHSDUDOOHO LHPD\EHLQWKHVDPHRU
 RSSRVLWHGLUHFWLRQDQGPD\QRWEHRIWKHVDPHPDJQLWXGH 



 $Q RSHQ WLQ ER[ RI QHJOLJLEOH WKLFNQHVV LV WR EH PDGH 7KH GHVLJQ RI WKH ER[ DQG LWV
KRUL]RQWDOEDVHDUHVKRZQEHORZ
x
x

y





 x
 x

7KH PLGGOHSRUWLRQRI
WKHKRUL]RQWDOEDVHRIWKHER[LVDUHFWDQJOHRIOHQJWK  x FP DQGZLGWK  x FP ZKLOHWKH
WZRHQGVDUHVHPLFLUFOHVRIUDGLXVxFP7KHER[KDVDGHSWKRI y FP DQGLWVYROXPHLV
FP

6KRZWKDWWKHWRWDOH[WHUQDOVXUIDFHDUHDALQFPRIWKHER[LVJLYHQE\
 S  
A S   x   
S   x

8VHGLIIHUHQWLDWLRQWRILQGWKHYDOXHRIxZKLFKPLQLPL]HVA >@
 6LQFHWKHYROXPHRIWKHER[LVFP 
>@  
ʌx    x  y  Ÿ y 
ʌ   x  
 
/HWWKHH[WHUQDOVXUIDFHDUHDEHAFP   
A  x   xy  ʌx  ʌxy
  

ʌ   x    ʌ   xy 
ª  º 
ʌ   x    ʌ   x « »
 
¬ ʌ   x ¼ 
 ʌ   
ʌ   x 

ʌ   x 
 
GA  ʌ   
 ʌ   x    
Gx ʌ   x 

 ʌ   
 ʌ   x 
ʌ   x  7RVKRZWKDWxPLQLPL]HVA
 ʌ   LWLVUHFRPPHQGHGWKDWWKH
x  6HFRQG'HULYDWLYH7HVWLV
ʌ   

 XVHG
ª  ʌ   º  ,WLVQRWHDV\WRMXVWLI\XVLQJ
x «  »
  FRUUHFWWRVI  WKH)LUVW'HULYDWLYH7HVW
¬ ʌ   ¼ EHFDXVHLWLVQRWHDV\WR
 VKRZZKHWKHUWKHVLJQRI
G A  ʌ    ʌ   GA  ʌ  
 ʌ    !  DW x    ʌ   x 
Gx  ʌ   x  ʌ    Gx ʌ   x 
 LVSRVLWLYHRUQHJDWLYHIRU
7KXVWKHH[WHUQDOVXUIDFHDUHDLVPLQLPXPDW x    RU  


 x   ax  b
$FXUYHCKDVHTXDWLRQ y ZKHUHabDQGcDUHFRQVWDQWV,WLVJLYHQWKDWC
xc
KDVDVWDWLRQDU\SRLQWDW   DQGWKHOLQH x  LVDQDV\PSWRWHWRC
 
 L  )LQGWKHYDOXHVRIabDQGc >@
  
 LL  6NHWFKWKHJUDSKRICVWDWLQJWKHHTXDWLRQVRIDQ\DV\PSWRWHVDQGWKHFRRUGLQDWHV
RIDQ\D[LDOLQWHUFHSWVDQGRUWXUQLQJSRLQWV >@

 L  6LQFH x  LVDQDV\PSWRWHWRWKHFXUYH c   
>@ 
$OVRCKDVDVWDWLRQDU\SRLQWDW   ͕   OLHVRQWKHFXUYHDQG G y  DWWKLVSRLQW
Gx
x   ax  b b
6XE   LQWR y ͕  Ÿ b  
x 
x   ax   Gy  x  a x    x   ax   x     a
y Ÿ 
x Gx x    x   
Gy a  
Ÿa 
1RWH
Gx  
LL  x %HDZDUHWKDWUDWLRQDOIXQFWLRQVVKRXOGWDNHXS

x  x  


>@ y x  µSRUWLRQV¶6R\RXZLOOQHHGWRDGMXVWWKHZLQGRZ
x x LQ\RXU*&ZKHQ\RXRQO\VHHRQHSRUWLRQ
2EOLTXHDV\PSWRWHLV y x    x /DEHODQ\DV\PSWRWHVDQGWKHFRRUGLQDWHVRIDQ\
y D[LDOLQWHUFHSWVDQGRUWXUQLQJSRLQWV
x  x 'UDZWKHJUDSKVPRRWKO\ZLWKWKHHQGWHQGLQJ
WRZDUGVWKHDV\PSWRWHV

  

   
 
x
 
   


 
x  x
 D  :LWKRXWXVLQJDFDOFXODWRUVROYHWKHLQHTXDOLW\ !   >@
x  x  


  
 
 E  L  a
2QWKHVDPHD[HVVNHWFKWKHJUDSKVRI y  DQG y    _ x _ ZKHUHa
x
LVDFRQVWDQWVXFKWKDW   a     >@
  

  LL  a
+HQFHRURWKHUZLVHVROYHWKHLQHTXDOLW\      _ x _   >@
x
 D  x  x 
>@ ! 
x   x  

 x   x  x    x  
! 
x   x   

x  x   
!
x   x   
x    
! 
x   x  

6LQFH x   ! x  \ ?^` 

7DNHQRWHRILQHTXDOLW\VLJQVR
 x   x   !   x z   DVWRH[FOXGH x  
? x   RU  x  RU x !  
E  y 7KHUHVWULFWLRQ   a   VKRXOG
L  EHWDNHQLQWRDFFRXQWZKHQ

>@ VNHWFKLQJWKHJUDSKRI
a
 y   6LQFHWKHxLQWHUFHSW
x
a
 LVDW x  DQG

 a 
  a   Ÿ      WKH
x  
 O  FXUYHVKRXOGFXWWKHxD[LVDWD

SRLQWWRWKHULJKWRI x  
RQHRIWKHxLQWHUFHSWVRIWKH
RWKHUJUDSK y   x 

E  $WSRLQWRILQWHUVHFWLRQ 1RWHWKDWWKHJUDSKVLQWHUVHFWDW
LL  a WKHSRLQWZKHUH x   DQGVR
  x
>@ x ZHKDYH x  x DWWKH
x a  LQWHUVHFWLRQ
x  a  ' x  
7KHVROXWLRQWRWKHLQHTXDOLW\LV  a  x   


 2Q-DQXDU\*DU\ORDQVAIURPDEDQNZKLFKFKDUJHVFRPSRXQGLQWHUHVWDWDUDWH
RIrSHUPRQWKDWWKHHQGRIHDFKPRQWK*DU\LQWHQGVWRIXOO\UHSD\WKHORDQLQnPRQWKV
ZLWKDIL[HGPRQWKO\LQVWDOPHQWRI Pn ZKLFKKHSD\VRQWKHILUVWGD\RIHDFKVXEVHTXHQW
PRQWK

A R   R n r
6KRZWKDW Pn ZKHUH R    >@
R 
n

 
 P Rn
L  6KRZWKDW n  >@
Pn Rn  
  
 LL  ,WLVJLYHQWKDW r  ILQGWKHOHDVWLQWHJUDOQXPEHURIPRQWKVnIRUZKLFKWKH
P 
UDWLR n LVJUHDWHUWKDQ  >@
Pn 
  
 Pn 
LLL  6KRZWKDWWKHUDWLR LVDOZD\VJUHDWHUWKDQ DQGH[SODLQZKDWWKLVVWDWHPHQW
Pn 
PHDQVLQWKHFRQWH[WRIWKHTXHVWLRQ >@

 0RQWK %HJLQQLQJRIPRQWK (QGRIPRQWK *LYHQVXFK
>@  -DQ  A AR ZRUG
 )HE  AR  Pn  AR  Pn R 
 SUREOHPVLWLV
EHVWWRFUHDWH
 0DU  AR   Pn R  Pn  AR   Pn R   Pn R  DWDEOHWR
 « « LGHQWLI\WKH
nPRQWKVODWHU AR  Pn R  Pn R    Pn  
n n  n  
SDWWHUQ
6LQFHORDQLVIXOO\UHSDLGLQnPRQWKV
AR n  Pn R n   Pn R n      Pn 
AR n  Pn R n   R n      
§    R n ·
Pn ¨ ¸ AR n 
¨  R ¸
© ¹
AR n   R
Pn
  Rn
AR n R  
Pn  VKRZQ
Rn 
L  P n AR  n R   Rn  
>@ u 
Pn R 
n
AR n R  
1HHGWRVKRZ
R n R n   H[SOLFLWO\KRZ
WRIDFWRULVHWKH
R n
 
 GHQRPLQDWRU
R  
R n n DQGFDQFHOWKH
WHUPVDVWKLVLV
R   R  
n n
D6+2:
TXHVWLRQ
Rn
VKRZQ
Rn  
LL   
>@ *LYHQ r  WRILQGnVXFKWKDWUDWLRLVODUJHUWKDQ  

r 
R    






Rn  
! 
R  
n

 R n !  R n   VLQFHR n   ! 


  
Rn !
 
 
OQ
n!   
OQ 
7KXVOHDVWnLV :KHQXVLQJ
 WKH*&WR
$OWHUQDWLYHO\ VROYHWKH
Rn  LQHTXDOLW\
!  WKHUHVKRXOG
R  
n

8VLQJJUDSKLQJFDOFXODWRU EHDQDWWHPSW
WRVKRZ
Rn 
:KHQ n   n    FOHDUO\ZK\
R   WKHOHDVWQLV
Rn  
:KHQ n   n  ! 
R  
/HDVW n  
LLL  r 
>@ 6LQFH R   !   VLQFH r !  

Rn ! 
Rn  Rn ! Rn  
Rn 
!
Rn  
Rn 
!
R  
n

P 
+HQFH n LVDOZD\VJUHDWHUWKDQ ͘
Pn 

,I*DU\LQWHQGVWRVWUHWFKKLVORDQWHUPWRWZLFHDVORQJWKHPRQWKO\
LQVWDOPHQWZRXOGEHPRUHWKDQKDOIRIZKDWKHZRXOGQHHGWRSD\LIKH
VWD\HGRQZLWKKLVLQLWLDOORDQWHUP


 7KHFRPSOH[QXPEHUVzDQGwDUHVXFKWKDW

z   La DQG w b  L 

ZKHUHaDQGbDUHUHDOQXPEHUV
 
 L  *LYHQ WKDW zz  w    L  ZKHUH z LV WKH FRPSOH[ FRQMXJDWH RI z ILQG WKH
H[DFWYDOXHVRIaDQGb >@
  

  z § z · ʌ
LL  *LYHQLQVWHDGWKDW a  DQG b  ILQG DQGVKRZWKDW DUJ ¨ ¸ 
 w © w ¹ 
 >@

:LWKRXWXVLQJDFDOFXODWRUILQGWKHVPDOOHVWSRVLWLYHZKROHQXPEHUnIRUZKLFK
n
§ z · LVSXUHO\LPDJLQDU\ >@
¨ ¸
©w¹

 zz  w    L  
L    La   La  b  L

  L
>@ 
  a  b  bL     L
 


&RPSDULQJUHDODQGLPDJLQDU\SDUWV
a   b   DQG b  Ÿ b  
 a   
 a r  
LL  
§ ʌ ·
 L ¨©   ¸¹ 
>@   L H 
z    
w  L §ʌ·
L¨ ¸

H ©  ¹
§ ʌ ʌ · 
 L ¨©     ¸¹ 
H
 
§ ʌ · 
 L ¨©    ʌ ¸¹
H  
 
§ ʌ ·
 L ¨©  ¸¹ 
H 


z  § z · ʌ 
7KXV DQG DUJ ¨ ¸  VKRZQ 
w  © w ¹  
$OWHUQDWLYH0HWKRG 
 
z _z_   
 
w _ w_  

§z·
DUJ ¨ ¸ DUJ z  DUJ w  ʌ 1RWHWKHSUHVHQWDWLRQIRU
© w¹ WKHDOWHUQDWLYHPHWKRGDV
ʌ ʌ LWLVPDWKHPDWLFDOO\
   ʌ  LQFRUUHFWWRZULWH
 

ʌ ʌ
  ʌ  VKRZQ  §z· ʌ S
  DUJ ¨ ¸  
©w¹  
>@ n n
)RU §¨ z ·¸ WREHSXUHO\LPDJLQDU\ZHUHTXLUH § z · SXUHO\LPDJLQDU\
¨ ¸
© w ¹ ©w¹
nʌ ʌ ʌ ʌ ʌ LPSOLHVWKDW
r  r  r  r 
     § nʌ ·
 FRV ¨ ¸ 
     ©  ¹
n       
    

7KHVPDOOHVWSRVLWLYHZKROHQXPEHUnLV
  

 7KHIXQFWLRQILVGHILQHGE\
m
   I  x 6 x   mx x  \ x t 

ZKHUHmLVDSRVLWLYHFRQVWDQW
 
 L  )LQG I  x DQGZULWHGRZQWKHGRPDLQRI I    >@
  
 LL  6NHWFK RQ WKH VDPH GLDJUDP WKH JUDSKV RI y I x DQG y I  x  VKRZLQJ
FOHDUO\WKHJUDSKLFDOUHODWLRQVKLSEHWZHHQWKHWZRJUDSKV >@
  
 LLL  )LQG WKH YDOXH RI m VXFK WKDW WKH FXUYHV LQ SDUW LL  LQWHUVHFW DW WKH SRLQW ZKHUH
x    >@
  
 ,QWKHUHVWRIWKHTXHVWLRQWKHYDOXHRImLVJLYHQWREH
 
 7KHIXQFWLRQJLVGHILQHGE\
   J  x 6 OQ x x  \ x t H 
  
 LY  )LQG DQ H[SUHVVLRQ IRU IJ x DQG KHQFH RU RWKHUZLVH ILQG WKH H[DFW YDOXH RI
IJ  

>@
  
 Y  6ROYHWKHLQHTXDOLW\ IJ x !    x  >@

 § m· m
  7RPDNHxWKHVXEMHFWZH
L  /HW y x  mx ¨ x  ¸ 

 FDQHLWKHUDSSO\µFRPSOHWH
>@ © ¹  WKHVTXDUH¶ DVVKRZQ RU

§ m· m FRQVLGHUWKHTXDGUDWLF
¨x ¸ y HTXDWLRQRIx
© ¹ 
 x  mx  y  DQGµVROYH¶
m m
x r y IRUxXVLQJWKHTXDGUDWLF
  IRUPXOD
m  m m 
6LQFH x t  I x  x  ([SODQDWLRQVKRXOGEHJLYHQ
   
 IRUWKHFKRLFHRI I x 
m
'RPDLQRI I   5DQJHRII  >  f 

LL  :KHQGUDZLQJJUDSKRII
>@ WKHGRPDLQ0867EH
FRQVLGHUHG

7KHHQGSRLQWVVKRXOGEH
LQGLFDWHGFOHDUO\


LLL  
6LQFHWKHJUDSKVRI y I x DQG y I x LQWHUVHFW 
>@ DORQJWKHOLQH y x WKHSRLQWRILQWHUVHFWLRQLV  
I  
   m  
m 
LY  OQ x  OQ x  1RWH

IJ x
>@
OQ x z OQ x 


/HW IJ  k Ÿ IJ k

IRUk t H 
IJ z I J 


OQ k  OQ k 



OQ k   OQ k   
OQ k RU OQ k 

k H RUk H 1$ ' k t H
? IJ 

H 
Y  OQ x 1RWH'IJ 'JVRFRQVLGHU

 OQ x !   x 
>@
y RQO\WKHJUDSKRI IJ IRU
xt H

7KH*&VKRXOGEHXVHGWR
VROYHWKHLQHTXDOLW\
$URXJKVNHWFKRIWKHJUDSK
VKRXOGEHSUHVHQWHGLQWKH
VROXWLRQWRVXSSRUWWKH
x DQVZHUIURPWKH*&
  :ULWLQJGRZQWKHDQVZHU

ZLWKRXWDQ\ZRUNLQJLVD
)URP*& x !  VI 
µKLJKULVN´VWUDWHJ\


 7KH,QVWUXPHQW/DQGLQJ6\VWHP ,/6 LVXVHGLQPDQ\DLUSRUWVDURXQGWKHZRUOG,WLVD


SUHFLVLRQ UXQZD\ DSSURDFK DLG EDVHG RQ WZR UDGLR EHDPV WUDQVPLWWHUV ZKLFK WRJHWKHU
SURYLGHSLORWVZLWKERWKWKHYHUWLFDODQGKRUL]RQWDOJXLGDQFHGXULQJDQDSSURDFKWRODQG
7KH ,/6 ZRUNV ZLWK D ORFDOL]HU /2&  WKDW SURYLGHV WKH JXLGDQFH IRUKRUL]RQWDO SODQDU
PRYHPHQWRIWKHDLUFUDIWZKLOHDVHFRQGWUDQVPLWWHUNQRZQDVWKHJOLGHVORSH *6 GHILQHV
WKHFRUUHFWYHUWLFDOSODQDUGHVFHQW7KHILQDODSSURDFKRIWKHDLUFUDIWRQWRWKHUXQZD\LV
JXLGHGE\WKHWUDMHFWRU\GHILQHGE\WKHLQWHUVHFWLRQEHWZHHQWKHEHDPVRI/2&DQG*6

7KHRULJLQO LVWDNHQWREHWKHEDVHRIWKH/2&DQGWKHJURXQGLVWKHxySODQH7KHEHDPV
§  · §·
RI /2& DQG *6 DUH GHILQHG E\ SODQHV ZLWK HTXDWLRQV  U ˜ ¨  ¸  DQG U ˜ ¨¨  ¸¸  
¨ ¸
¨  ¸ ¨¸
© ¹ © ¹
UHVSHFWLYHO\

 L  )LQGDYHFWRUHTXDWLRQRIWKHOLQHlZKLFK WKHILQDODSSURDFKWUDMHFWRU\OLHVRQ
  >@
  
 LL  )LQGWKHDQJOHWKDWlPDNHVZLWKWKHJURXQG >@
  
 §  ·
'XHWRDQHPHUJHQF\DLUFUDIWAORFDWHGDWDSRLQWP ZLWKFRRUGLQDWHV ¨    ¸ ZDV
©  ¹
UHGLUHFWHGEDFNWRWKHDLUSRUW7RGHWHUPLQHLIWKHDLUFUDIWLVDWDVDIHGLVWDQFHIURPWKH
ODQGLQJVWULSWKHSLORWQHHGVWRNQRZKRZIDUWKHDLUFUDIWLVIURPl
 
 LLL  6KRZWKDWWKHFRRUGLQDWHVRIWKHSRLQWFRQlVXFKWKDWFLVFORVHVWWRA LV   .
 >@
 
 $LUFUDIWBWRRNRIIIURPWKHVDPHDLUSRUWEXWIURPDGLIIHUHQWUXQZD\7KHIOLJKWSDWKRIB
§· §·
¨ ¸ ¨ ¸
OLHVRQWKHOLQH lB ZLWKHTXDWLRQ U ¨¸  O ¨  ¸  O  \ 
¨¸ ¨¸
© ¹ © ¹
 LY  :LWKRXW XVLQJ D FDOFXODWRU VKRZ WKDW WKH OLQHV PF DQG lB GR QRW LQWHUVHFW HDFK
RWKHU  >@
  
 Y  7KH VKRUWHVW GLVWDQFH EHWZHHQ  VNHZ OLQHV l DQG l LV JLYHQ E\ WKH OHQJWK RI

o
SURMHFWLRQRI XY RQWR Q ZKHUHX DQGY DUHSRLQWVRQ l DQG l UHVSHFWLYHO\DQG
Q LVDYHFWRUSHUSHQGLFXODUWRERWK l DQG l 8VLQJWKLVUHVXOWILQGWKHVKRUWHVW
GLVWDQFHEHWZHHQOLQHVPFDQG lB  >@

 §  · §· 
L  ¨ ¸ ¨ ¸ 
U ˜ ¨  ¸  DQG U ˜ ¨  ¸  
>@ ¨  ¸ ¨¸ 
© ¹ © ¹ 
x z    
 
y    
8VLQJWKH*& 1RWHWKHFRUUHFW
§  · § · SUHVHQWDWLRQIRU
¨ ¸ ¨ ¸ HTXDWLRQRIDOLQH,WLV
/LQHRILQWHUVHFWLRQLV l  r ¨  ¸  t ¨  ¸  t  \ 
 ¨ ¸ ¨ ¸ µ r ͛LQVWHDGRIµ l ͛͘
© ¹ ©  ¹ 
LL    


JJJG
§ · PF DQGYHFWRU PF DUH
§ · §· § · ¨ ¸
¨ ¸ ¨ ¸ ¨ ¸ 127WKHVDPHVR
¨  ¸
¨ ¸  O ¨  ¸ 
¨ ¸  P
¨ ¸
VLPSO\VROYLQJ
JJJG
¨ ¸ ¨¸ ¨¸ PF L  M  N  O L  N
© ¹ © ¹ © ¹ ¨  ¸
© ¹
 LVLQFRUUHFW

O  P    
 
P    
O  P    6LQFHWKLVLVD6+2:
TXHVWLRQWKH*&FDQQRW
)URP   P 
EHXVHGWRVROYHWKH
  HTXDWLRQV
    P  Ÿ P  
  
6LQFHWKHUHLVQRXQLTXHYDOXHRI P VDWLVI\LQJDOOHTXDWLRQV
WKHOLQHVPFDQG lB GRQRWLQWHUVHFWHDFKRWKHU
Y  § ·  
>@ ¨ ¸ § · 
YHFWRUSHUSHQGLFXODUWROLQHPFDQG lB  ¨  ¸ u ¨¨  ¸¸  
¨ ¸ ¨ ¸
¨  ¸ ©  ¹
© ¹
§  · §  ·
¨ ¸ ¨ ¸
       ¨    ¨  ¸ 
¸ ¨ ¸
¨ ¸
©  ¹ ©  ¹
/HW X    DQG Y    EHSRLQWVRQ lB ĂŶĚOLQHPF
UHVSHFWLYHO\
§    · §  ·

o
¨ ¸ ¨ ¸
XY ¨    ¸ ¨  ¸  
¨   ¸ ¨  ¸
© ¹ © ¹
§  · §  ·
¨ ¸¨ ¸
¨  ¸<¨  ¸
¨  ¸ ¨  ¸
© ¹© ¹
6KRUWHVWGLVWDQFH  
      



       FRUUHFWWRVI 



 7DNLQJOWREHWKHRULJLQRI&DUWHVLDQFRRUGLQDWHVWKHHQGARIDWKLQLQH[WHQVLEOHVWULQJ
 ABRIOHQJWK ʌXQLWVLVIL[HGWRDSRLQW    RQWKHFLUFXPIHUHQFHRIDFLUFOHRIXQLWUDGLXV
 DQGFHQWUH O,QLWLDOO\WKHHQG BLVDW   ʌ DQGWKHVWULQJLVVWUDLJKWDQGWDQJHQWWRWKH
 FLUFOH VHH)LJ 7KHVWULQJUHPDLQVWDXWDQGLVWKHQZUDSSHGURXQGWKHFLUFOHXQWLOWKH
 HQGBFRPHVLQWRFRQWDFWZLWKWKHFLUFOH
 
 %\FRQVLGHULQJWKHFRRUGLQDWHVRIWKHSRLQWFDQGWKHOHQJWKRIFB VHH)LJ RURWKHUZLVH
 VKRZWKDWWKHSDWKRIBFDQEHGHVFULEHGE\WKHFXUYHC ZLWKSDUDPHWULFHTXDWLRQV
 x FRV t  t VLQ t  y VLQ t  t FRV t IRU  d t d ʌ 

ZKHUHtLVWKHDQJOHLQUDGLDQVDVVKRZQLQ)LJ >@

 
 B  y  y
 
 
 
 
  B
 


 F


 t
 x x
 A O A O




)LJ )LJ


 L  )LQGWKHH[DFWYDOXHRIt IRUZKLFKx WDNHVLWVPD[LPXPYDOXHRQCDQGVNHWFKC
 >@
  
 LL  7KHSRLQWPRQWKHFXUYHC KDVSDUDPHWHUp ZKHUH   p  ʌ 7KHQRUPDOWRCDW
WKHSRLQWPPHHWVWKHyD[LVDWWKHSRLQWZLWKFRRUGLQDWHV  k )LQGkLQWHUPVRI
pDQGVKRZWKDWkLVQROHVVWKDQRQH >@

 
>@ y 


B 




F 




t 
x 
A O







&RRUGLQDWHVRISRLQWFDUH FRV t  VLQ t  
$OVRDQJOHAOF  ʌ  t DQGOHQJWKRIDUFAF  ʌ  t  +HUHPDNHXVHRI
7KXVOHQJWK FB t DQGWKHDQJOHWKDWFB PDNHVZLWKWKHKRUL]RQWDOLV x r FRV T DQG
ʌ y r VLQ T LQWKH
t 
 WULDQJOHEHORZ
+HQFHWKHxFRRUGLQDWHRIB 
§ ʌ· §ʌ ·  r
 FRV t  t FRV ¨ t  ¸ FRV t  t FRV ¨  t ¸ FRV t  t VLQ t  y
© ¹ © ¹
DQGWKHyFRRUGLQDWHRIB x
§ ʌ· §ʌ ·
 VLQ t  t VLQ ¨ t  ¸ VLQ t  t VLQ ¨  t ¸ VLQ t  t FRV t 
© ¹ © ¹
L  x FRV t  t VLQ t $SSO\SURGXFWUXOHWR 1HHGWRVROYH
>@ Gx  Gx
 VLQ t  VLQ t  t FRV t t FRV t GLIIHUHQWLDWH t VLQ t   LQVWHDGRI
Gt Gt
Gx Gy
)RUPD[LPXPx t FRV t  
Gt Gx
ʌ 
 t  RU FRV t  Ÿt 
 
G x 
FRV t  t VLQ t 
Gt  
G x 
:KHQ t   !  ͘ 
Gt 
 
ʌ G x ʌ 
:KHQ t       ͘
 Gt  
ʌ 
7KXVxLVPD[LPXPZKHQ t 
 
 
y 

/DEHOSRLQWV
 ʌ 
FOHDUO\

&RQVLGHUWKH
UHTXLUHGGRPDLQ
RIt ZKHQ
  GUDZLQJWKHFXUYH
VHWGRPDLQLQ
x *&XQGHU
   µZLQGRZ¶ 

LL  y VLQ t  t FRV t 
>@ Gy 
FRV t  FRV t  t VLQ t t VLQ t
Gt
Gy
Gy Gt t VLQ t
7KXV WDQ t ͘
Gx Gx t FRV t
Gt
$WSRLQW P FRV p  p VLQ p  VLQ p  p FRV p 
+0DWKHPDWLFV3URPRWLRQ([DPLQDWLRQ6ROXWLRQVIRU6WXGHQWV

 )LQG WKH HTXDWLRQ RI WKH WDQJHQW WR WKH FXUYH y    xy   x     DW WKH SRLQW   
>@
 y   xy   x   
  

>@ $SSO\SURGXFWUXOHDQGFKDLQ

'LIIHUHQWLDWLQJLPSOLFLWO\ZUWx
UXOHWRGLIIHUHQWLDWHxy
Gy § Gy ·
 y   ¨ y    xy ¸   x 
Gx © Gx ¹ $SSO\FKDLQUXOHWR
:KHQx  y  GLIIHUHQWLDWHy
Gy § Gy ·
 
  ¨      ¸    
Gx © Gx ¹ 5HDGTXHVWLRQ
Gy Gy FDUHIXOO\ILQGWKH
  Ÿ  HTXDWLRQRIWKH
Gx Gx
WDQJHQWOLQHQRW
7KHHTXDWLRQRIWKHWDQJHQWLV y    x    ILQGJUDGLHQWDW
LH y x   RQO\

 $Q DULWKPHWLF VHULHV KDV ILUVW WHUP a DQG FRPPRQ GLIIHUHQFH d ZKHUH a DQG d DUH
QRQ]HUR7KHVWDQGUGWHUPVRIWKHDULWKPHWLFVHULHVDUHDQGUHVSHFWLYHO\*LYHQ
DOVRWKDWWKHVXPRILWVILUVWnWHUPVLV)LQGWKHYDOXHVRIadDQGn >@
 n
Sn ª a  n   d º¼  
>@ ¬ un a  n   d
n d   an  nd       

n
Sn >a  n   d @ 
a  d    
a  d     

6ROYLQJ  DQG  XVLQJ*&a DQG d   1RQHHGWRVROYHWKH


6XEaDQGdLQWR   HTXDWLRQVE\KDQG
 n    n    n     8VH*&WRHYDOXDWHa
dDQGn
n    n   

8VLQJ*&n  ±QRWDFFHSWHGDV


WDF
DFF
DF n! 

 QFHH u  u  u   LVJLYHQE\


7KHVHTXHQFH
TXHQ FH LV JL
JLYH un WDQ n   WDQ n   IRU n t  

L WDQ A  WDQ B
%\FRQVLGHULQJ
%
%\
\ FRQ
R VLG WDQ A  B VKRZWKDW
  WDQ A WDQ B
WDQ n    WDQ n  
 u n    >@
WDQ

LL  n
+HQFHILQG ¦ u r LQWHUPVRIn >@
r 
 L  un WDQ n   WDQ n   7KLVLVD6+2:TXHVWLRQVR
>@ GHWDLOHGZRUNLQJVKRXOGEH
WDQ n    WDQ n  
WDQ n    n   VKRZQ6KRZH[SOLFLWO\
  WDQ n   WDQ n   WDQ A  B
WDQ n    WDQ n   WDQ n    n   
WDQ 
  un  WDQ   WDQ
  un WDQ  WDQ n    WDQ n   RU
WDQ A  B
WDQ n    WDQ n  
un   VKRZQ WDQ n    n   
WDQ
WDQ
GHSHQGLQJRQZDVFKRVHQIRU
ADQGB
LL  n n
ª WDQ r    WDQ r   º 6XIILFLHQWZRUNLQJQHHGVWR
>@ ¦ ur ¦« ¬ WDQ
 »  EHVKRZQ ILUVWDQGODVW
r  r ¼ URZV 
 n n

¦
WDQ r 
ª¬ WDQ r    WDQ r   º¼  ¦ 
r 
7KHUHLVQRQHHGWRHYDOXDWH
 WDQDQGWDQ
> WDQ   WDQ  1RWHWKHYDOXHVDUH
WDQ HYDOXDWHGLQUDGLDQVLQVWHDG
 WDQ   WDQ  RIGHJUHHVR
 WDQ   WDQ   WDQ  

 WDQ 
 WDQ n    WDQ n   
$OWHUQDWLYHPHWKRG
 WDQ n    WDQ n   @  n   n

WDQ n    WDQ 
¦u
r 
r

 n
WDQ n
§ WDQ   WDQ  ·
¦u
r 
r ¨
© WDQ 
 ¸
¹


 ,WLVJLYHQWKDW I x OQ   H x :LWKRXWXVLQJDFDOFXODWRUILQGWKH0DFODXULQVHULHVIRU
I x XSWRDQGLQFOXGLQJWKHWHUPLQ x    >@
 I x OQ   H x
>@
Hx 
I
x  
 H x
  Hx
Hx
I

x
  H x 

 
:KHQ x   I 
 
 OQ  I
  I

 
 

  
7KXVWKH0DFODXULQVHULHVIRU I x LV OQ   x   x    OQ   x  x    
   

Alternative:
/HWy OQ   H x
H y   Hx
Gy
Hy Hx 
Gx

G  y y § Gy ·
Hy H ¨ ¸ Hx
Gx  © Gx ¹
Gy  G y 
:KHQ x  y OQ   
Gx  Gx  

  
7KXVWKH0DFODXULQVHULHVIRU I x LV OQ   x   x    OQ   x  x   
   

Alternative:
8VLQJWKHVWDQGDUGVHULHVH[SDQVLRQVRI H x DQG OQ   x 
I x OQ   H x
§ § x ··
OQ ¨   ¨  x    ¸ ¸ 
© ©  ¹¹
§ § x x 
··
OQ ¨  ¨      ¸ ¸
© ©   ¹¹
§ x x 
·
OQ   OQ ¨     ¸
©   ¹

§ x x ·  § x x ·
OQ   ¨    ¸  ¨    ¸  
©  ¹ ©   ¹ 

x x  § x ·
OQ     ¨ ¸  
  ©¹
x x
OQ     
 

 1RWH:KHQXVLQJWKHVWDQGDUGVHULHVWKHIROORZLQJZLOO
VWDQ notZRUN


 OQ   H
x
H x

H  H
x  x 
§ x · § x ·


  ¨  x    ¸  ¨   x    ¸   


   ©  ¹ ©  ¹

 
§ x
x · § x ·
 OQ   H OOQ
Q  x
 x 
Q    ¨   x    ¸  ¨   x    ¸   
  ©  ¹ ©  ¹

 ,QHDFKH[SDQVLRQDOOWKHEUDFNHWVKDYHDFRQVWDQWWHUP x WHUP x  WHUPHWF
 VRZHZLOOQRWEHDEOHWRFROOHFWDOOWKHUHVSHFWLYHWHUPVWRWDOO\



 *LYHQD EDQG FDUHXQLWYHFWRUVVXFKWKDWD˜E D˜FZKDWLVWKHUHODWLRQVKLSEHWZHHQD
DQGEF" >@

,WLVIXUWKHUJLYHQWKDWELVSHUSHQGLFXODUWREF)LQGWKHDQJOHEHWZHHQEDQGF >@

&RPPHQWRQWKHUHODWLRQVKLSEHWZHHQDu EDQGEF >@


 D ˜ E D˜ F 'RWSURGXFWLV
>@ GLVWULEXWLYHLH
D ˜ E  D˜ F  
D ˜ E  D˜ F D ˜ E  F
D ˜ E  F 
1RWH E z F DVEDQGF
7KHUHIRUHDLVSHUSHQGLFXODUWR E  F VLQFH D z  DQG E z F  DUHXQLWYHFWRUV
>@ /HW T EHWKHDQJOHEHWZHHQEDQGF 
*LYHQELVSHUSHQGLFXODUWR E  F  E ˜ E  F   

E ˜ E E ˜ F


E  E F FRV T 5HDGTXHVWLRQ
FDUHIXOO\WRPDNHXVH
 FRVT   VLQFHE DQGF DUHXQLWYHFWRUV  RI_E_ _F_ 

ʌ
T UDGLDQV

>@ 6LQFHDDQGEDUHSHUSHQGLFXODUWRE±FDQG D u E LV 
SHUSHQGLFXODUWRDDQGE D u E LVSDUDOOHOWRE±F͘

6RPHWKLQJVWRQRWH

 x X A ZDQGY A ZGRHVnotLPSO\XDQGYDUHSDUDOOHO HJL A NDQGM A NEXW
 LDQGMDUHQRWSDUDOOHO 
 x :HGRQRWKDYHVXIILFLHQWLQIRUPDWLRQWRFRQFOXGHZKHWKHUWKHYHFWRUVDUH
 LQWKHVDPHGLUHFWLRQRULQWKHRSSRVLWHGLUHFWLRQRUDUHHTXDO:HFDQ
 RQO\FRQFOXGHWKDWWKHYHFWRUVDUHSDUDOOHO LHPD\EHLQWKHVDPHRU
 RSSRVLWHGLUHFWLRQDQGPD\QRWEHRIWKHVDPHPDJQLWXGH 



 $Q RSHQ WLQ ER[ RI QHJOLJLEOH


O WKLFNQHVV LV WR EH PDGH 7KH GHVLJQ RI WKH ER[ DQG LWV
QEHHOR
ORZ
KRUL]RQWDOEDVHDUHVKRZQEHORZ
x
x

y





 x
 x

7KH PLGGOHSRUWLRQRI
WKHKRUL]RQWDOEDVHRIWKHER[LVDUHFWDQJOHRIOHQJWK  x FP DQGZLGWK  x FP ZKLOHWKH
WZRHQGVDUHVHPLFLUFOHVRIUDGLXVxFP7KHER[KDVDGHSWKRI y FP DQGLWVYROXPHLV
FP

6KRZWKDWWKHWRWDOH[WHUQDOVXUIDFHDUHDALQFPRIWKHER[LVJLYHQE\
 S  
A S   x   
S   x

8VHGLIIHUHQWLDWLRQWRILQGWKHYDOXHRIxZKLFKPLQLPL]HVA >@
 6LQFHWKHYROXPHRIWKHER[LVFP 
>@  
ʌx    x  y  Ÿ y 
ʌ   x  
 
/HWWKHH[WHUQDOVXUIDFHDUHDEHAFP   
A  x   xy  ʌx  ʌxy
  

ʌ   x    ʌ   xy 
ª  º 
ʌ   x    ʌ   x « »
 
¬ ʌ   x ¼ 
 ʌ   
ʌ   x  

ʌ   x 
 
GA  ʌ
 ʌ  
 ʌ 
 ʌʌ    x 
   
Gx ʌ   x 
ʌ

 ʌ

 ʌ
 ʌ
 ʌ    
 ʌʌ  
 x
 x 
ʌ  
ʌ  x  7RVKRZWKDWxPLQLPL]HVA
 ʌ
  
 ʌ  
 LWLVUHFRPPHQGHGWKDWWKH
x  6HFRQG'HULYDWLYH7HVWLV
ʌ   

 XVHG
ª  ʌ   º  ,WLVQRWHDV\WRMXVWLI\XVLQJ
x «  »
  FRUUHFWWRVI  WKH)LUVW'HULYDWLYH7HVW
¬ ʌ   ¼ EHFDXVHLWLVQRWHDV\WR
 VKRZZKHWKHUWKHVLJQRI
G A  ʌ    ʌ   GA  ʌ  
 ʌ    !  DW x    ʌ   x 
Gx  ʌ   x  ʌ    Gx ʌ   x 
 LVSRVLWLYHRUQHJDWLYHIRU
7KXVWKHH[WHUQDOVXUIDFHDUHDLVPLQLPXPDW x    RU  


 x   ax  b
$FXUYHCKDVHTXDWLRQ y ZKHUHabDQGcDUHFRQVWDQWV,WLVJLYHQWKDWC
xc
KDVDVWDWLRQDU\SRLQWDW   DQGWKHOLQH x  LVDQDV\PSWRWHWRC
 
 L  )LQGWKHYDOXHVRIabDQGc >@
  
 LL  6NHWFKWKHJUDSKRICVWDWLQJWKHHTXDWLRQVRIDQ\DV\PSWRWHVDQGWKHFRRUGLQDWHV
RIDQ\D[LDOLQWHUFHSWVDQGRUWXUQLQJSRLQWV >@

 L  6LQFH x  LVDQDV\PSWRWHWRWKHFXUYH c   
>@ 
$OVRCKDVDVWDWLRQDU\SRLQWDW   ͕   OLHVRQWKHFXUYHDQG G y  DWWKLVSRLQW
Gx
x   ax  b b
6XE   LQWR y ͕  Ÿ b  
x 
x   ax   Gy  x  a x    x   ax   x     a
y Ÿ 
x Gx x    x   
Gy a  
Ÿa 
1RWH
Gx  
LL  x %HDZDUHWKDWUDWLRQDOIXQFWLRQVVKRXOGWDNHXS

x  x  


>@ y x  µSRUWLRQV¶6R\RXZLOOQHHGWRDGMXVWWKHZLQGRZ
x x LQ\RXU*&ZKHQ\RXRQO\VHHRQHSRUWLRQ
2EOLTXHDV\PSWRWHLV y x    x /DEHODQ\DV\PSWRWHVDQGWKHFRRUGLQDWHVRIDQ\
y D[LDOLQWHUFHSWVDQGRUWXUQLQJSRLQWV
x  x 'UDZWKHJUDSKVPRRWKO\ZLWKWKHHQGWHQGLQJ
WRZDUGVWKHDV\PSWRWHV

  

  
   


 

 
 
x
 

  




  
   


 
x  x
 D  :LWKRXWXVLQJDFDOFXODWRUVROYHWKHLQHTXDOLW\ !   >@
x  x  


  
 
 E  L  a
2QWKHVDPHD[HVVNHWFKWKHJUDSKVRI y  DQG y    _ x _ ZKHUHa
x
LVDFRQVWDQWVXFKWKDW   a     >@
  

  LL  a
+HQFHRURWKHUZLVHVROYHWKHLQHTXDOLW\      _ x _   >@
x
 D  x  x 
>@ ! 
x   x  

 x   x  x    x  
! 
x   x   

x  x   
!
x   x   
x    
! 
x   x  

6LQFH x   ! x  \ ?^` 

7DNHQRWHRILQHTXDOLW\VLJQVR
 x   x   !   x z   DVWRH[FOXGH x  
? x   RU  x  RU x !  
E  y 7KHUHVWULFWLRQ   a   VKRXOG
L  EHWDNHQLQWRDFFRXQWZKHQ

>@ VNHWFKLQJWKHJUDSKRI
a
 y   6LQFHWKHxLQWHUFHSW
x
a
 LVDW x  DQG

 a 
  a   Ÿ      WKH
x  
 O  FXUYHVKRXOGFXWWKHxD[LVDWD

SRLQWWRWKHULJKWRI x  
RQHRIWKHxLQWHUFHSWVRIWKH
RWKHUJUDSK y   x 

E  $WSRLQWRILQWHUVHFWLRQ 1RWHWKDWWKHJUDSKVLQWHUVHFWDW
LL  a WKHSRLQWZKHUH x   DQGVR
  x
>@ x ZHKDYH x  x DWWKH
x a  LQWHUVHFWLRQ
x  a  ' x  

ROXXWL
W RQQ WRRWK
WKH
H LQHT
LQHT
LQ OLW\\ LV  a  x   
HTX OL
HTXD
7KHVROXWLRQWRWKHLQHTXDOLW\LV


 -DQ
DQQXXDDU\\ 
2Q-DQXDU\*DU\ORDQVAIURPDEDQNZKLFKFKDUJHVFRPSRXQGLQWHUHVWDWDUDWH
SHHUPR
P
RIrSHUPRQWKDWWKHHQGRIHDFKPRQWK*DU\LQWHQGVWRIXOO\UHSD\WKHORDQLQnPRQWKV
ZLWKDIL[HGPRQWKO\LQVWDOPHQWRI Pn ZKLFKKHSD\VRQWKHILUVWGD\RIHDFKVXEVHTXHQW
PRQWK

A R   R n r
6KRZWKDW Pn ZKHUH R    >@
R 
n

 
P Rn
L  6KRZWKDW n  >@
Pn Rn  

 LL  ,WLVJLYHQWKDW r  ILQGWKHOHDVWLQWHJUDOQXPEHURIPRQWKVnIRUZKLFKWKH
P 
UDWLR n LVJUHDWHUWKDQ  >@
Pn 

Pn 
LLL  6KRZWKDWWKHUDWLR LVDOZD\VJUHDWHUWKDQ DQGH[SODLQZKDWWKLVVWDWHPHQW
Pn 
PHDQVLQWKHFRQWH[WRIWKHTXHVWLRQ >@

 0RQWK %HJLQQLQJRIPRQWK (QGRIPRQWK *LYHQVXFK
>@  -DQ  A AR ZRUG
 )HE  AR  Pn  AR  Pn R 
 SUREOHPVLWLV
EHVWWRFUHDWH
 0DU  AR   Pn R  Pn  AR   Pn R   Pn R  DWDEOHWR
 « « LGHQWLI\WKH
nPRQWKVODWHU AR  Pn R  Pn R    Pn  
n n  n  
SDWWHUQ
6LQFHORDQLVIXOO\UHSDLGLQnPRQWKV
AR n  Pn R n   Pn R n      Pn 
AR n  Pn R n   R n      
§    R n ·
Pn ¨ ¸ AR n 
¨  R ¸
© ¹
AR n   R
Pn
  Rn
AR n R  
Pn  VKRZQ
Rn 
L  P n AR  n R   Rn  
>@ u 
Pn R 
n
AR n R  
1HHGWRVKRZ
R n R n   H[SOLFLWO\KRZ
WRIDFWRULVHWKH
R n
 
 GHQRPLQDWRU
R  
R n n DQGFDQFHOWKH
WHUPVDVWKLVLV
R   R  
n n
D6+2:
TXHVWLRQ
Rn
VKRZQ
VKR
V
VKR
KRZQ
ZQ
Rn  
LL   
>@ *LYHQ r 
  WRILQGnVXFKWKDWUDWLRLVODUJHUWKDQ
  

r 
R    






Rn  
! 
R  
n

 R n !  R n   VLQFHR n   ! 


  
Rn !
 
 
OQ
n!   
OQ 
7KXVOHDVWnLV :KHQXVLQJ
 WKH*&WR
$OWHUQDWLYHO\ VROYHWKH
Rn  LQHTXDOLW\
!  WKHUHVKRXOG
R  
n

8VLQJJUDSKLQJFDOFXODWRU EHDQDWWHPSW
WRVKRZ
Rn 
:KHQ n   n    FOHDUO\ZK\
R   WKHOHDVWQLV
Rn  
:KHQ n   n  ! 
R  
/HDVW n  
LLL  r 
>@ 6LQFH R   !   VLQFH r !  

Rn ! 
Rn  Rn ! Rn  
Rn 
!
Rn  
Rn 
!
R  
n

P 
+HQFH n LVDOZD\VJUHDWHUWKDQ ͘
Pn 

,I*DU\LQWHQGVWRVWUHWFKKLVORDQWHUPWRWZLFHDVORQJWKHPRQWKO\
LQVWDOPHQWZRXOGEHPRUHWKDQKDOIRIZKDWKHZRXOGQHHGWRSD\LIKH
VWD\HGRQZLWKKLVLQLWLDOORDQWHUP
ORDDQ
QWH
W U


 7KHFRPSOH[QXPEHUVzDQGwDUHVXFKWKDW
H[QX
QXPE
QX PEHUUV z DQG
Q w DUHH VX
QG

z   La DQG w b  L 

ZKHUHaDQGbDUHUHDOQXPEHUV
H a DQG
QGb DUH

 L  *LYHQ WKDW zz  w    L  ZKHUH z LV WKH FRPSOH[ FRQMXJDWH RI z ILQG WKH
H[DFWYDOXHVRIaDQGb >@
 

 z § z · ʌ
LL  *LYHQLQVWHDGWKDW a  DQG b  ILQG DQGVKRZWKDW DUJ ¨ ¸ 
 w © w ¹ 
 >@

:LWKRXWXVLQJDFDOFXODWRUILQGWKHVPDOOHVWSRVLWLYHZKROHQXPEHUnIRUZKLFK
n
§ z · LVSXUHO\LPDJLQDU\ >@
¨ ¸
©w¹

 zz  w    L  
L    La   La  b  L

  L
>@ 
  a  b  bL     L
 


&RPSDULQJUHDODQGLPDJLQDU\SDUWV
a   b   DQG b  Ÿ b  
 a   
 a r  
LL  
§ ʌ ·
 L ¨©   ¸¹ 
>@   L H 
z    
w  L §ʌ·
L¨ ¸

H ©  ¹
§ ʌ ʌ · 
 L ¨©     ¸¹ 
H
 
§ ʌ · 
 L ¨©    ʌ ¸¹
H  
 
§ ʌ ·
 L ¨©  ¸¹ 
H 


z  § z · ʌ 
7KXV DQG DUJ ¨ ¸  VKRZQ 
w  © w ¹  
$OWHUQDWLYH0HWKRG 
 
z _z_   
 
w _ w_  

§z·
DUJ ¨ ¸ DUJ z  DUJ w  ʌ 1RWHWKHSUHVHQWDWLRQIRU
© w¹ WKHDOWHUQDWLYHPHWKRGDV
ʌ ʌ LWLVPDWKHPDWLFDOO\
   ʌ  LQFRUUHFWWRZULWH
 

ʌ
ʌ ʌ


  ʌ
ʌ  VKRZQ 
VKRZ
VKKR
RZZQ §z· ʌ S
  DUJ ¨ ¸  
©w¹  
>@ n n
)RU §¨ z ·¸ WREHSXUHO\LPDJLQDU\ZHUHTXLUH
WR EH
EH SXUHO  § z · SXUHO\LPDJLQDU\
¨ ¸
© w ¹ ©w¹
nʌ ʌ ʌ ʌ ʌ LPSOLHVWKDW
r  r  r  r 
     § nʌ ·
 FRV ¨ ¸ 
     ©  ¹
n       
    

7KHVPDOOHVWSRVLWLYHZKROHQXPEHUnLV

 7KHIXQFWLRQILVGHILQHGE\
m
   I  x 6 x   mx x  \ x t 

ZKHUHmLVDSRVLWLYHFRQVWDQW

 L  )LQG I  x DQGZULWHGRZQWKHGRPDLQRI I    >@


 
 LL  6NHWFK RQ WKH VDPH GLDJUDP WKH JUDSKV RI y I x DQG y I  x  VKRZLQJ
FOHDUO\WKHJUDSKLFDOUHODWLRQVKLSEHWZHHQWKHWZRJUDSKV >@

 LLL  )LQG WKH YDOXH RI m VXFK WKDW WKH FXUYHV LQ SDUW LL  LQWHUVHFW DW WKH SRLQW ZKHUH
x    >@

,QWKHUHVWRIWKHTXHVWLRQWKHYDOXHRImLVJLYHQWREH

7KHIXQFWLRQJLVGHILQHGE\
   J  x 6 OQ x x  \ x t H 

 LY  )LQG DQ H[SUHVVLRQ IRU IJ x DQG KHQFH RU RWKHUZLVH ILQG WKH H[DFW YDOXH RI
IJ  

>@

 Y  6ROYHWKHLQHTXDOLW\ IJ x !    x  >@

 § m· m
  7RPDNHxWKHVXEMHFWZH
L  /HW y x  mx ¨ x  ¸ 

 FDQHLWKHUDSSO\µFRPSOHWH
>@ © ¹  WKHVTXDUH¶ DVVKRZQ RU

§ m· m FRQVLGHUWKHTXDGUDWLF
¨x ¸ y HTXDWLRQRIx
© ¹ 
 x  mx  y  DQGµVROYH¶
m m
x r y IRUxXVLQJWKHTXDGUDWLF
  IRUPXOD
m  m m 
6LQFH x t  I x  x  ([SODQDWLRQVKRXOGEHJLYHQ
   
 IRUWKHFKRLFHRI I x 
m
5 QJ
5D RI I  >
'RPDLQRI I   5DQJHRII
QJH RI  f 

LL  :KHQGUDZLQJJUDSKRII
>@ WKHGRPDLQ0867EH
FRQVLGHUHG

7KHHQGSRLQWVVKRXOGEH
LQGLFDWHGFOHDUO\


LLL  
6LQFHWKHJUDSKVRI y I x DQG y I x LQWHUVHFW 
>@ DORQJWKHOLQH y x WKHSRLQWRILQWHUVHFWLRQLV  
I  
   m  
m 
LY  OQ x  OQ x  1RWH

IJ x
>@
OQ x z OQ x 


/HW IJ  k Ÿ IJ k

IRUk t H
IJ z I J 


OQ k  OQ k 



OQ k   OQ k   
OQ k RU OQ k 

k H RUk H 1$ ' k t H
? IJ 

H 
Y  OQ x 1RWH'IJ 'JVRFRQVLGHU

 OQ x !   x 
>@
y RQO\WKHJUDSKRI IJ IRU
xt H

7KH*&VKRXOGEHXVHGWR
VROYHWKHLQHTXDOLW\
$URXJKVNHWFKRIWKHJUDSK
VKRXOGEHSUHVHQWHGLQWKH
VROXWLRQWRVXSSRUWWKH
x DQVZHUIURPWKH*&
  :ULWLQJGRZQWKHDQVZHU

ZLWKRXWDQ\ZRUNLQJLVD
P *& x ! 
)URP*&  VI
   V 
µKLJKULVN´VWUDWHJ\


 7KH,QVWUXPHQW/DQGLQJ6\VWHP ,/6 LVXVHGLQPDQ\DLUSRUWVDURXQGWKHZRUOG,WLVD


SUHFLVLRQ UXQZD\ DSSURDFK DLG EDVHG RQ WZR UDGLR EHDPV WUDQVPLWWHUV ZKLFK WRJHWKHU
SURYLGHSLORWVZLWKERWKWKHYHUWLFDODQGKRUL]RQWDOJXLGDQFHGXULQJDQDSSURDFKWRODQG
7KH ,/6 ZRUNV ZLWK D ORFDOL]HU /2&  WKDW SURYLGHV WKH JXLGDQFH IRUKRUL]RQWDO SODQDU
PRYHPHQWRIWKHDLUFUDIWZKLOHDVHFRQGWUDQVPLWWHUNQRZQDVWKHJOLGHVORSH *6 GHILQHV
WKHFRUUHFWYHUWLFDOSODQDUGHVFHQW7KHILQDODSSURDFKRIWKHDLUFUDIWRQWRWKHUXQZD\LV
JXLGHGE\WKHWUDMHFWRU\GHILQHGE\WKHLQWHUVHFWLRQEHWZHHQWKHEHDPVRI/2&DQG*6

7KHRULJLQO LVWDNHQWREHWKHEDVHRIWKH/2&DQGWKHJURXQGLVWKHxySODQH7KHEHDPV
§  · §·
RI /2& DQG *6 DUH GHILQHG E\ SODQHV ZLWK HTXDWLRQV  U ˜ ¨  ¸  DQG U ˜ ¨¨  ¸¸  
¨ ¸
¨  ¸ ¨¸
© ¹ © ¹
UHVSHFWLYHO\

 L  )LQGDYHFWRUHTXDWLRQRIWKHOLQHlZKLFK WKHILQDODSSURDFKWUDMHFWRU\OLHVRQ
  >@
  
 LL  )LQGWKHDQJOHWKDWlPDNHVZLWKWKHJURXQG >@
  
 §  ·
'XHWRDQHPHUJHQF\DLUFUDIWAORFDWHGDWDSRLQWP ZLWKFRRUGLQDWHV ¨    ¸ ZDV
©  ¹
UHGLUHFWHGEDFNWRWKHDLUSRUW7RGHWHUPLQHLIWKHDLUFUDIWLVDWDVDIHGLVWDQFHIURPWKH
ODQGLQJVWULSWKHSLORWQHHGVWRNQRZKRZIDUWKHDLUFUDIWLVIURPl
 
 LLL  6KRZWKDWWKHFRRUGLQDWHVRIWKHSRLQWFRQlVXFKWKDWFLVFORVHVWWRA LV   .
 >@
 
 $LUFUDIWBWRRNRIIIURPWKHVDPHDLUSRUWEXWIURPDGLIIHUHQWUXQZD\7KHIOLJKWSDWKRIB
§· §·
¨ ¸ ¨ ¸
OLHVRQWKHOLQH lB ZLWKHTXDWLRQ U ¨¸  O ¨  ¸  O  \ 
¨¸ ¨¸
© ¹ © ¹
 LY  :LWKRXW XVLQJ D FDOFXODWRU VKRZ WKDW WKH OLQHV PF DQG lB GR QRW LQWHUVHFW HDFK
RWKHU  >@
  
 Y  7KH VKRUWHVW GLVWDQFH EHWZHHQ  VNHZ OLQHV l DQG l LV JLYHQ E\ WKH OHQJWK RI

o
SURMHFWLRQRI XY RQWR Q ZKHUHX DQGY DUHSRLQWVRQ l DQG l UHVSHFWLYHO\DQG
Q LVDYHFWRUSHUSHQGLFXODUWRERWK l DQG l 8VLQJWKLVUHVXOWILQGWKHVKRUWHVW
Q OLQHV
HHVVPF
GLVWDQFHEHWZHHQOLQHVPFDQG
P lB  >@

 §  · §· 
¨ ¸
L  QG U ˜ ¨¨  ¸¸  
U ˜ ¨  ¸  DQG
DQG
QG 
>@ ¨  ¸ ¨¸ 
©  ¹ © ¹ 
x z   
 
 
y    
8VLQJWKH*& 1RWHWKHFRUUHFW
§  · § · SUHVHQWDWLRQIRU
¨ ¸ ¨ ¸ HTXDWLRQRIDOLQH,WLV
/LQHRILQWHUVHFWLRQLV l  r ¨  ¸  t ¨  ¸  t  \
 ¨ ¸ ¨ ¸ µ r ͛LQVWHDGRIµ l ͛͘
© ¹ ©  ¹ 
LL    


>@  
 
 
 
§  · 
T WDQ  ¨ ¸   R  
©  ¹
$OWHUQDWLYH0HWKRG 

§  · §  ·
¨ ¸¨ ¸ $VWKHJURXQGLVWDNHQ
¨  ¸<¨  ¸ WREHWKHxySODQHD
¨  ¸ ¨¸ §  · YHFWRUSHUSHQGLFXODUWR
T VLQ  © ¹ © ¹  VLQ  ¨ ¸ R  WKHJURXQGLVN
  
 
©  ¹

LLL   
>@  P 


 
   F

§  · § ·
¨ ¸ ¨ ¸
l  r ¨  ¸  t ¨  ¸  t  \ 
 ¨ ¸ ¨ ¸
© ¹ ©  ¹
/HWFEHSRLQWRQOLQH l  
§   t   ·
 o  o  o ¨ ¸
PF OF  OP ¨  ¸ 
¨ ¸
¨ t  ¸
© ¹
§t   · 
¨ ¸ § ·
¸<¨  ¸  
 o
PF A l Ÿ ¨ 
¨ ¸ ¨¨ ¸¸
¨ t   ¸©¹
© ¹
t    t   
 
t 
7KHUHIRUHFLV    
LY  § · 
>@ 
o ¨ ¸ 
F
PF ¨  ¸  
¨ ¸ 
¨  ¸
© ¹ 
§ · 
§ · ¨ ¸ 
¨ ¸ ¨  ¸  P  \ 
(TXDWLRQRIOLQH3)LV r   P 7KH r LQWKHHTXDWLRQRI
 ¨¨ ¸¸ ¨ ¸ 
OLQH PF UHSUHVHQWVWKH
©¹ ¨  ¸
© ¹ SRVLWLRQYHFWRUVRIDOO
$WSRLQWRILQWHUVHFWLRQ WKHSRLQWVRQWKHOLQH
JJJG
ZKLOHYHFWRU PF LVMXVW
WKHYHFWRULWVHOI6ROLQH
JJJG
§ · PF DQGYHFWRU PF DUH
§ · §· § · ¨ ¸
¨ ¸ ¨ ¸ ¨ ¸ 127WKHVDPHVR
¨  ¸
¨ ¸  O ¨  ¸ 
¨ ¸  P
¨ ¸
VLPSO\VROYLQJ
JJJG
¨ ¸ ¨¸ ¨¸ PF L  M  N  O L  N
© ¹ © ¹ © ¹ ¨  ¸
© ¹
 LVLQFRUUHFW

O  P    
 
P    
O  P    6LQFHWKLVLVD6+2:
TXHVWLRQWKH*&FDQQRW
)URP   P 
EHXVHGWRVROYHWKH
  HTXDWLRQV
    P  Ÿ P  
  
6LQFHWKHUHLVQRXQLTXHYDOXHRI P VDWLVI\LQJDOOHTXDWLRQV
WKHOLQHVPFDQG lB GRQRWLQWHUVHFWHDFKRWKHU
Y  § ·  
>@ ¨ ¸ § · 
YHFWRUSHUSHQGLFXODUWROLQHPFDQG lB  ¨  ¸ u ¨¨  ¸¸ 
¨ ¸ ¨ ¸
¨  ¸ ©  ¹
© ¹
§  · §  ·
¨ ¸ ¨ ¸
       ¨    ¨  ¸ 
¸ ¨ ¸
¨ ¸
©  ¹ ©  ¹
/HW X    DQG Y    EHSRLQWVRQ lB ĂŶĚOLQHPF
UHVSHFWLYHO\
§    · §  ·

o
¨ ¸ ¨ ¸
XY ¨    ¸ ¨  ¸  
¨   ¸ ¨  ¸
© ¹ © ¹
§  · §  ·
¨ ¸¨ ¸
¨  ¸<¨  ¸
¨  ¸ ¨  ¸
© ¹© ¹
6KRUWHVWGLVWDQFH  
      




       FRUUHFWWRVI 




 7DNLQJ
DNLLQJ
QJOWREHWKHRULJLQRI&DUWHVLDQFRRUGLQDWHVWKHHQG
WREH
EH WKH
KH RULLJLJ QRI ARIDWKLQLQH[WHQVLEOHVWULQJ
ABRIOHQJWK
RIIOOHHQJ K ʌXQLWVLVIL[HGWRDSRLQW
JWWK XQLLWWVV LV I    RQWKHFLUFXPIHUHQFHRIDFLUFOHRIXQLWUDGLXV
DQGFHQWUH
HQWQWUH
QW
QWUH
UH O,QLWLDOO\WKHHQG
UH , LW
,Q BLVDW   ʌ DQGWKHVWULQJLVVWUDLJKWDQGWDQJHQWWRWKH
VHH )
FLUFOH VHH)LJ 7KHVWULQJUHPDLQVWDXWDQGLVWKHQZUDSSHGURXQGWKHFLUFOHXQWLOWKH
HQGBFRPHVLQWRFRQWDFWZLWKWKHFLUFOH

%\FRQVLGHULQJWKHFRRUGLQDWHVRIWKHSRLQWFDQGWKHOHQJWKRIFB VHH)LJ RURWKHUZLVH
VKRZWKDWWKHSDWKRIBFDQEHGHVFULEHGE\WKHFXUYHC ZLWKSDUDPHWULFHTXDWLRQV
x FRV t  t VLQ t  y VLQ t  t FRV t IRU  d t d ʌ 
ZKHUHtLVWKHDQJOHLQUDGLDQVDVVKRZQLQ)LJ >@
 
 B  y  y
 
 
 
 
  B
 


 F


 t
 x x
 A O A O




)LJ )LJ


 L  )LQGWKHH[DFWYDOXHRIt IRUZKLFKx WDNHVLWVPD[LPXPYDOXHRQCDQGVNHWFKC
 >@
  
 LL  7KHSRLQWPRQWKHFXUYHC KDVSDUDPHWHUp ZKHUH   p  ʌ 7KHQRUPDOWRCDW
WKHSRLQWPPHHWVWKHyD[LVDWWKHSRLQWZLWKFRRUGLQDWHV  k )LQGkLQWHUPVRI
pDQGVKRZWKDWkLVQROHVVWKDQRQH >@

 
>@ y 


B 




F 




t 
x 
A O







&RRUGLQDWHVRISRLQWFDUH FRV t  VLQ t  
$OVRDQJOHAOF  ʌ  t DQGOHQJWKRIDUFAF  ʌ  t  +HUHPDNHXVHRI
7KXVOHQJWK FB t DQGWKHDQJOHWKDWFB PDNHVZLWKWKHKRUL]RQWDOLV x r FRV T DQG
ʌ y r VLQ T LQWKH
t 
 WULDQJOHEHORZ
+HQFHWKHxFRRUGLQDWHRIB 
§ ʌ· §ʌ ·  r
 FRV t  t FRV ¨ t  ¸ FRV t  t FRV ¨  t ¸ FRV t  t VLQ t  y
© ¹ © ¹
DQGWKHyFRRUGLQDWHRIB x
§ ʌ· §ʌ ·
 VLQ t  t VLQ ¨ t  ¸ VLQ t  t VLQ ¨  t ¸ VLQ t  t FRV t 
© ¹ © ¹
L  x FRV t  t VLQ t $SSO\SURGXFWUXOHWR 1HHGWRVROYH
>@ Gx  Gx
 VLQ t  VLQ t  t FRV t t FRV t GLIIHUHQWLDWH t VLQ t   LQVWHDGRI
Gt Gt
Gx Gy
)RUPD[LPXPx t FRV t  
Gt Gx
ʌ 
 t  RU FRV t  Ÿt 
 
G x 
FRV t  t VLQ t 
Gt  
G x 
:KHQ t   !  ͘ 
Gt 
 
ʌ G x ʌ 
:KHQ t       ͘
 Gt  
ʌ 
7KXVxLVPD[LPXPZKHQ t 
 
 
y 

/DEHOSRLQWV
 ʌ 
FOHDUO\

&RQVLGHUWKH
UHTXLUHGGRPDLQ
RIt ZKHQ
  GUDZLQJWKHFXUYH
VHWGRPDLQLQ
x *&XQGHU
   µZLQGRZ¶ 

LL  y VLQQ t  t FFRV
RRVV t 
>@ Gy 
FRV t  FRV
t  t VLQ t t VLQ t
Gt
Gy
Gy Gt t VLQ t
7KXV WDQ t ͘
Gx Gx t FRV t
Gt
$WSRLQW P FRV p  p VLQ p  VLQ p  p FRV p 
JUDGLHQWRIQRUPDO   FRW p 

(TXDWLRQRIQRUPDODWPLV
y  VLQ p  p FRV p  FRW p x  FRV p  p VLQ p

:KHQ x   y k 
k FRW p FRV p  p FRV p  VLQ p  p FRV p
FRW p FRV p  VLQ p
FRV  p
 VLQ p 
VLQ p
FRV  p  VLQ  p
VLQ p

VLQ p

)RU   p  ʌ    VLQ p d  DQGWKXV k t  
VLQ p

RIVER VALLEY HIGH SCHOOL
2019 JC1 Promotional Examination
Higher 2

NAME

INDEX
CLASS
NUMBER

MATHEMATICS 9758/01
Paper 1 3 October 2019
3 hours
Candidates answer on the Question Paper
Additional Materials: List of Formulae (MF26)

For examiner’s
READ THESE INSTRUCTIONS FIRST use only
Question
Mark
Write your class, index number and name on all the work you hand number
in. 1
Write in dark blue or black pen.
You may use an HB pencil for any diagrams or graphs. 2
Do not use staples, paper clips, glue or correction fluid. 3
Answer all the questions. 4
Write your answers in the spaces provided in the question paper. 5
Give non-exact numerical answers correct to 3 significant figures, or
1 decimal place in the case of angles in degrees, unless a different 6
level of accuracy is specified in the question.
7
The use of an approved graphing calculator is expected, where
appropriate. 8
You are required to present the mathematical steps using
mathematical notations and not calculator commands. 9
You are reminded of the need for clear presentation in your answers. 10
Up to 2 marks may be deducted for poor presentation in your
answers. 11
The number of marks is given in brackets [ ] at the end of each 12
question or part question.
The total number of marks for this paper is 100.

Total

Calculator Model:
2

1 u n is a cubic polynomial in terms of n, where n is a positive integer. It is known that

u0 0 , u1 u2 8 and u 4 is four times of u1 . Find u n in terms of n. [4]

2 Show that sin(n  1)  sin(n  1) k cos n , where k is an exact constant to be


determined. [2]

Hence or otherwise, find cos(1)  cos(2)  cos(3)  ...  cos N in terms of N, expressing

your answer in the form a sin N  1  sin N  b , where a and b are constants to be

determined. [4]

3 (i) By means of the substitution x 12 sin T , show that


3 3
³0
12  x 2 dx π
2
3. [4]

(ii) The region R is bounded by the curve y 4


12  x 2 , the lines x 3 and

x  3 , and the x-axis. Using your result in part (i), find the exact volume of
the solid formed by rotating R through 2π radians about the x-axis. [3]

4 (a) Express x  3 A 2 x  2  B , where A and B are constants to be determined.

x3
Hence, find ´
µ 2 dx . [3]
¶ x  2x  2

(b) The Mean Value Theorem for Integrals states that if f is continuous on a closed
interval [a, b], then there is at least one point x = c in that interval such that
1 b
f c f x dx , where f(c) is known as the mean value of f on the
b  a ³a
interval [a, b].
Let f x xe x . Given that d ! 2 , determine the exact value of d such that the

mean value of f on the interval >1, d @ is f(2). [5]


3

5 (a) The diagram shows the graph of y = f(x). The curve passes through the origin, has
a minimum point at 3, 0 and its equations of asymptotes are x 2 and

y 2.

y=2

Sketch the following graphs on separate axes.


1
(i) y [3]
f x

(ii) y f  x [2]

Describe a series of transformations that transforms the graph of x 2  y  1


2
(b) 1

y2
onto the graph of x  1 
2
1. [3]
9
4

6 The “Nephroid” (also known as a two-cusped epicycloid) is a kidney-shaped curve often


seen on the surface of a cup of coffee in the sunshine – a crescent of light formed by
sunlight reflecting off the inside of the cup onto the surface of the drink. More generally,
it is the shape made by parallel rays of light reflecting from the inside of any semi-
circle1.

A particular Nephroid curve C has the following parametric equations:


x 3cos t  cos3t
y 3sin t  sin 3t , where 0 d t  2π .
(i) Sketch C for 0 d t  2π , indicating clearly the axial intercepts. [1]
π
(ii) Find the exact equation of the normal to C at the point P where t . [4]
3

(iii) Hence, find the area of the region in the first quadrant bounded by C, the normal
in part (ii) and the positive y-axis. [3]

7 Given that xy 1  y 2 , where y ! 0 , show that


2
d2 y dy § dy ·
x  2y 2
 2  2¨ ¸ 0. [3]
dx dx © dx ¹
Write down the Maclaurin series for y up to and including the term in x 2 . [2]

Hence, by using the Maclaurin series for y found above and the standard series found in
1
MF26, find the Maclaurin series for up to and including the term in x 2 . [3]
y

x 2  ax  6
8 The curve C has equation y , where a is a constant. It is given that the line
x2
y x  5 is an asymptote of C.
(i) Show that a = 3. [2]
(ii) Prove algebraically that y cannot lie between 1 and 15. [2]

1
http://www.daviddarling.info/encyclopedia/N/nephroid.html
5

(iii) Sketch the curve C, stating clearly the coordinates of any points of intersection
with the axes, the coordinates of any stationary points and the equations of
asymptotes. [3]
x 2  3x  6
(iv) Hence, solve the inequality t x  3 . [2]
x2

9 The function f is defined as follows

f :x x  x2  1,
1 x .

The graph of function g with domain f, 1 is given in the diagram below. It has

asymptotes x 1 and y 2 , and cuts the axes at 0, 3 and 3, 0 .

y
x=1

y=2

−3 O x

y = g(x)

−3

(a) (i) Show that f 1 exists. [2]

(ii) Show that gf exists and find the range of gf. [4]

(b) Sketch the graph of y g  x  1 . [3]


6

10 (a) Given that the sum of the first n terms of a sequence is n 2 , show that the
sequence is an arithmetic progression. [3]

2n

¦ nr  22 r , leaving your answer in terms


n
n
(b) Given ¦ r2
r 1 6
n  1 2n  1 , find
r 0
2

of n. [3]

f
(c) Cauchy’s Root Test states that a series of the form ¦a
r 1
r converges when

1 1 1
lim an n  1 and diverges when lim an n ! 1 . When lim an n
1 , the test is
n of n of n of

f
1
inconclusive. Using this test, explain why ¦ converges for all real
x2  2
r
r 1

values of x and find the sum to infinity of this series. [3]

11 A tank contains 20 kg of salt dissolved in 5000 l of water. Salt solution that contains
0.03 kg of salt per litre of water is pumped into the tank at a rate of 25 l per minute. The
solution in the tank is kept thoroughly mixed and is drained from the tank at a rate of 25 l
per minute. Let x be the amount of salt, in kg, in the tank at time t minutes.
dx 150  x
(i) Show that . [3]
dt 200

(ii) Find the amount of salt in the tank at time t. [5]

(iii) Find the amount of salt in the tank eventually. [1]


(iv) Sketch the graph for the amount of salt in the tank over time. Comment on the
suitability of the model. [3]
7

4 3
12 [It is given that a sphere of radius r has surface area 4πr 2 and volume πr .]
3
(a) An open metallic cup has a cylindrical body with height y cm and base radius
x cm. The base of the can is shaped as a hemisphere, which shares the same
diameter as the cylinder, as shown in the diagram.

y cm

x cm

The total area of metal sheet used to make the cup is 40π 2 cm2.
(i) Show that the volume V cm3 of the cup is given by
§ 5 ·
V πx ¨ 20π  x 2 ¸ . [3]
© 3 ¹

(ii) Find the exact value of x for which V has a maximum value. Hence,
find the maximum volume, giving your answer correct to 2 decimal
places. [4]

(iii) State an assumption made when carrying out your calculations. [1]
8

(b) Due to poor sales, the manufacturer decides to revamp the design of the
metallic cup. The newly designed cup consists of an inverted right cone of
radius 5 cm and height 15 cm, with its vertex fixed to a circular base. Before
mass production, the manufacturer produced a sample cup and tested it by
filling the cup completely with water. The sample cup was found to be
defective as water leaked out of a hole at the vertex of the cone at a rate of 2
cm3 per second. The diagram below shows the cup.

5 cm

15 cm

hole

Find the rate at which the water level is decreasing when the water is 3 cm
deep. [4]

- End of Paper -
2019 H2 Mathematics Promotional Examinations Solutions
1 Solution [4]

Let un an3  bn 2  cn  d

u0 0Ÿd 0

u1 abc 8 ………………….(1)

u2 8a  4b  2c 8 ………………….(2)

u4 4u1 Ÿ 64a  16b  4c 32 ……………..(3)

Using GC,
a 2, b 10, c 16
Hence, the cubic polynomial is:

un 2n3  10n 2  16n

2 Solution [6]
sin(n  1)  sin(n  1)
§ (n  1)  (n  1) · § (n  1)  (n  1) ·
2 cos ¨ ¸ sin ¨ ¸
© 2 ¹ © 2 ¹
2sin(1) cos n (shown, where k 2sin(1))

Alternatively,
sin(n  1)  sin(n  1)
sin n cos(1)  cos si 1)  sin
oss n sin(1)
o sin((1) (  cos n sin(1)
s n n cos(1)
si co
os(
os
s(1
(1)
1) cos
2sin(1) cos
os n (shown,
(ssh
how
own
n,, where
whe
here
re k 2sin(1))

1
cos(1)  cos(2)  cos(3)  ...  cos N
N

¦ cos n
n 1

1 N

¦ >sin(n  1)  sin(n  1)@


2sin(1) n 1
1
>sin(2)  sin(0)
2sin(1)
 sin(3)  sin(1)
 sin(4)  sin(2)
 sin(5)  sin(3)

 sin( N  1)  sin( N  3)
 sin( N )  sin( N  2)
 sin( N  1)  sin( N  1) @
1
>sin( N  1)  sin N  sin(1)@
2sin(1)
1 1
>sin( N  1)  sin N @ 
2sin(1) 2

1 1
?a (or 0.594 (to 3 s.f.)) , b 
2sin(1) 2

3 Solution [7]
(i) x 12 sin T
dx
12 cos T
dT
When x = 0,, T 0
1 π
When x 3, siin T Ÿ T
sin
2 6

2
3
³
0
12  x 2 dx
S

³ 12  12sin 2 T 12 cos T dT
6

0
S

12 ³ cos 2 T dT
6

0
S
cos 2T  1
12 ³ dT
6

0 2
S
ª 1 º6
6 «T  sin 2T »
¬ 2 ¼0
§S 3·
6 ¨¨  ¸¸
©6 4 ¹
3
S 3 (shown)
2
(ii) Volume
3
S³ y 2 dx
 3

3
S³ 12  x2 dx
 3

3
2S ³ 12  x 2 dx
0

§ 3 ·
2S ¨ S  3¸
© 2 ¹

S 2S  3 3 units3

3
4 Solution [8]
(a)
x3 A 2x  2  B

1
A , B 2
2

´ x3
µ 2 dx
¶ x  2x  2
1
´ 2x  2  2
µ2 2 dx
µ x  2x  2

1 ´ 2x  2 1
µ 2 dx  2 ³ 2 dx
2 ¶ x  2x  2 x  2x  2
1 ´ 1
ln x 2  2 x  2  2µ dx
¶ 1  x  1
2
2
1
ln x 2  2 x  2  2 tan 1 x  1  C
2
(b)
1 d x
f 2
d  1 ³1
xe dx

d d
³ ª¬ xe x º¼  ³ e x dx
d
xe x dx
1 1 1
d
ª¬ xe x  e x º¼
1

ed d  1  e  e
ed d  1

1
f 2 ª¬e d d  1 º¼ ed
d 1

2e 2 ed

ln ed n 2e2 lln
ln n22

d ln 2  2

4
5 Solution [8]
(a)(i)

y = 1/2
O

(a)(ii)

y=2

(b)
Translate
latte 1 un
unit
nit
it in
in the
tth
he positive
posi
po siti
tiive x-axis
Translatee 1 u
unnit
it in
unit in the
th positive y-axis
llell tto the y-axis by factor 3.
Scale parallel

OR

5
Translate 1 unit in the positive x-axis
Scale parallel to the y-axis by factor 3.
Translate 3 units in the positive y-axis
(Note: Transformation of x can be at any step)

6 Solution [8]
(i)

(ii)
dx
3sin t  3sin 3t
dt
dy
3cos t  3cos 3t
dt
dy 3cos t  3cos 3t cos t  cos 3t
dx 3sin t  3sin 3t  sin t  sin 3t
π
When t ,
3

π 3
3cos  3ccos
os π
3cos  3(1)
dy 3 2  3
dx π § 3·

3s
3ssiinn  33s
33sin sin
in π
3sin 3 ¨
3 ¸
© 2 ¹
1
Gradient of normal =
3

6
5 3 3
x , y
2 2
Thus equation of normal at P is

3 3 1 § 5·
y ¨x ¸
2 3© 2¹

1 5 3 3 3
y x 
3 6 2

1 2 3
?y x
3 3

(iii)

Area

1 § 5 ·§ 3 3 2 3 · 4
3 ¸¹ ³3 2 3
¨ ¸¨  ¸ x dy
2 © 2 ¹ ¨© 2

25 3 S

 ³S 3coss t  cooss 3t 3c 3cos 3t dt


o t 3
2
3cos
3 o ccos
o 3cos
cos
24 3

niitts2
= 4.30 units
un

Alternatively,
Area

7
5 5
1 2 3
³ y dx  ³ x
2 2
dx
0 0
3 3

S 5
1 2 3
³ 3sin t  sin 3t 3sin t  3sin 3t dx ³0 x
3 2
dx
S
2 3 3

= 4.30 units2

7 Solution [8]

xy 1  y 2
dy dy
yx 2 y
dx dx
dy
y  x  2 y 0 ---------------------- (1)
dx
dy d 2 y dy § dy ·
 x  2 y 2  ¨1  2 ¸ 0
dx dx dx © dx ¹
2
d2 y
x  2 y 2  2  2 §¨ ·¸
dy dy
0 (shown) ---------- (2)
dx dx © dx ¹

dy 1 d2 y 1
When x 0 , y 1,  and
dx 2 dx 2 4
1
1
y 1  x  4 x2 
2 2!
1 1
| 1  x  x2
2 8
1
1 § 1 1 ·
| ¨1  x  x 2 ¸
y © 2 8 ¹
1
§ § 1 1 2 ··
¨1  ¨  2 x  8 x ¸ ¸
© © ¹¹
2
§ 1 1 · § 1 1 ·
1  ¨  x  x 2 ¸  ¨  x  x 2 ¸  ...
© 2 8 ¹ © 2 8 ¹
1 1 1
1  x  x 2  x 2  ...
2 8 4
1 1 2
| 1 x  x
2 8

8
8 Solution [9]
(i)
x 2  ax  6 k
y x5 , for some k 
x2 x2
x 2  3x  (k  10)
x2
x 2  ax  6 = x2  3x  (k  10)

By comparing coefficients of x, a = 3 (shown)


Alternatively,

x 2  ax  6
y
x2
2a  10
x  (a  2)  (performing long division)
x2
Hence, a  2 5 Ÿ a 3 (shown)
(ii)

x 2  3x  6
y
x2
xy  2 y x 2  3 x  6
x 2  (3  y ) x  (2 y  6) 0

Want discriminant < 0 (so that we get ‘no real x’)


i.e. (3  y)2  4(2 y  6)  0

Ÿ y 2  14 y  15  0
Ÿ ( y  1)( y  15)  0
Ÿ 1  y  15 (shown)
(sh
show
own)
own)
n)

9
(iii) Graph of C:
y

A
x
(0 3)
(0,–3)

A (2, 1) , B (6,15)

(iv)

x 2  3x  6
t x  3
x2
Hence, sketch additionally the graph of y x  3 :
y

y x  3
A
x
–3
3

The two graphs intersect at A ( x 2 ) and at x 0.

10
?Answer: 2 d x d 0 or x ! 2
9 Solution [9]
(i)

y=k

Any horizontal line y k will intersect the graph of


y f x at most once. OR Any horizontal line y k ,
k < 0 will intersect the graph of y f x exactly once.

Thus, f is one-one. Therefore f 1 exist.


(ii)
Rf f, 0  f, 1 Dg

Therefore gf exists.

Using mapping method,


Df 
f
o Rf f, 0 
g
o 3, 2 Rgf

? Rgf 3, 2
(b)
y g x o y g x  1 o y g  x  1

11
10 Solution [9]
(a)
un S n  S n 1
n 2  n  1
2

n 2  n 2  2n  1
2n  1
un  un 1
2n  1  ª¬ 2 n  1  1º¼
2 (constant)
Therefore, the series is a AP.
(b)
2n

¦ nr
r 0
2
 22 r
2n 2n
n¦ r 2  ¦ 22
r

r 0 r 0

2n 2n
n¦ r 2  ¦ 4r  1
r 1 r 1

12
º 4 4  1
2n
ª 2n
n « 2n  1 4n  1 »  1
¬6 ¼ 4 1
1 2 1
n 2n  1 4n  1  42 n 1  1
3 3
1 2
3
n 2n  1 4n  1  42 n 1  1

(c)
1
n
1 1
lim lim
n of
x 2
 2
n n of
x  2
2

1
 1 since x 2  2 ! 1 for all x 
x 
2
2
f
1
?¦ converges.
x  2
2 r
r 1

1
f
1 x 2
2
¦
x  2 1
2 r
r 1
1 2
x 2
1 x2  1
y
x2  2 x2  2
1
x 1
2

13
11 Solution [12]
(i)
dx
rate in  rate out
dt

0.03 25  §¨
x ·
¸ 25
© 5000 ¹
x
0.75 
200
150  x
(shown)
200
(ii) -
dx 150  x
dt 200
1 dx 1
150  x dt 200
´ 1 dx 1
200 ³
µ 1 dt
¶ 150  x
1
 ln 150  x t C
200
1
 t C
150  x e 200

1
 t
150  x re C e 200

1
 t
150  x Ae 200

1
 t
x 150  Ae 200

When t 0 , x 20
1
 0
150  20 Ae 2 0
200

A 113
30
130
1
 t
?150  x 130e
130
30e 200
2

1
 t
x 150  130e 200

14
(iii)
t

As t o f , e 200
o 0 . ? x o 150 kg
(iv)

Model is not suitable as the concentration of salt in the tank


should be able to reach the same concentration as the salt
solution flowing into the tank in a finite time.

12 Solution [12]
(a) (i)
Total surface area of metal sheet

Total SA = 2S xy  2S x 2 40S 2
xy  x 2 20S
xy 20S  x 2

20S  x 2
y
x
20S
x
x

Total Volume
olu
lu
um
mee
2
S x 2 y  S x3
3
§ 20S · 2
S x2 ¨  x ¸  S x3
© x ¹ 3

15
2
S x(20S  x 2 )  S x3
3
ª 2 º
S x « 20S  x 2  x 2 »
¬ 3 ¼
ª 5 º
S x « 20S  x 2 » (shown)
¬ 3 ¼

(a)(ii)
dV ª 5 º ª 10 º
S « 20S  x 2 »  S x «  x » 0
dx ¬ 3 ¼ ¬ 3 ¼
5 10
20S  x 2  x 2 0
3 3
20S  5 x 0
2

5x2 20S
x 2
4S
x 4S cm or  4S cm (Rejected since x > 0)

To check if V is maximum:
x 3.535 4S | 3.54 3.545
dV 1.1018 > 0 0 −0.0103 < 0
dx

OR

d 2V ª 10 º 20S
S « x  x
dx 2 ¬ 3 »¼ 3

When x 4S ,

d 2V
11
 11.
11 37  0
.37
1.37
111.37
dx 2
Therefore, hee vvolume
re,, tthe
h olum
olu is maximum at x 4S

§ 5 ·
V S 4S ¨ 20S  4S ¸
© 3 ¹
466.49 cm3

16
(a)(iii)
We assume negligible thickness for the metal sheet used.
(b) The water forms a smaller cone. Let its radius be r and
height be h.
From the diagram, by similar triangles,
r 5
h 15
h
r
3
Let the volume of water in the cone be Vw .

1 2
Vw Sr h
3
2
1 §h·
S¨ ¸ h
3 ©3¹
1 § h2 ·
S¨ ¸h
3 © 9 ¹
S
h3
27
S
Vw h3
27
dVw S
h2
dh 9
dVw
Given: 2 cm 3 /s
dt
When h 3 ,
dVw dVw dh
u
dt dh dt
S 2 dh
2 (33)) u
(3)
9 dt
dh 2
 oorr  00.637 cm/s (to 3 sf)
dt S
The rate at which the water level is decreasing is 0.637
cm/s.

17
+0DWK3URPR6$-&

Questions from 2019 SAJC Promos

1 Solve the inequality 3x  2 t 7 1  x . [4]

1
2 Given that a curve has the equation x tan x y x 1 where x > 0 and y > 0, using a non-calculator
method, find the exact gradient of the curve at the point where x = 1. [5]

3 removed (not in syllabus)

4 Relative to the origin O, the points A and B have position vectors a and b respectively. It is
given that the magnitude of a is 4 and b is a unit vector perpendicular to a.
(i) Find the value of (2a  b) (3a  5b) . [4]

(ii) The point C is on AB such that AC : CB 3:1 . Write down the position vector of C, c, in
terms of a and b. [1]

(iii) State the geometrical meaning of b u c and find its exact value. [5]

5 Functions f and g are defined by


f :x  ln( x  a),
) x  , x ! a
g : x ( x  a )2 , x  , x ! 0
where a is a positive constant such that 1  a d 3 .
(i) Sketch the graph of y f ( x) and show that f has an inverse. [4]
(ii) On the same diagram in part (i), sketch the graph of y f 1 ( x) and y 1
f f ( x) . [2]

(iii) Find the range of the composite function fg. [2]


(iv) Without finding f 1 x , find x given that f 1g( x) e 3  a . [3]

x 2  ax  1
6 The curve C1 has equation y , where x  , x z b and a and b are constants.
xb
The lines x 4 and y x  6 are asymptotes to C1 .
(i) Write down the value of b. Hence, show that a 2 . [3]

With the values of a and b found in (i),


(ii) sketch C1 , stating the equations of any asymptotes, the coordinates of any turning points
and any points of intersection with the axes, [3]
(iii) A second curve C2 undergoes, in succession, the following transformations to get the
resulting curve C1:

A: Translation of 3 units in the negative x direction;


B: Scaling parallel to the x – axis by a scale factor of 2;
C: Reflection about the x – axis.

Find the equation of the curve C2, showing your workings clearly. [3]
7 removed (not in syllabus)

8 The curve C has parametric equations


t3
, y ª¬ln t º¼ , for 0  t d 3.
2
x
3
(i) Sketch the graph of C, giving the coordinates of its endpoint(s) and the point(s) where C
meets the axes. State also the equation of the vertical asymptote. [3]

§ p3 2·
(ii) Find the equation of the tangent to the curve C at the point ¨ , ª¬ln p º¼ ¸ , simplifying
© 3 ¹
your answer. [5]

(iii) Hence find the exact coordinates of the points Q and R where the tangent to the curve C
when t e meets the x-axis and y-axis respectively. [3]

(iv) Find the area of triangle OQR in exact form. [2]

9 removed (not in syllabus)

10 The plane –1 is defined by the equation r x i  2 j  2k 22 . A point P has coordinates


3 ,  2 , 1 .
(i) Find the position vector of the foot of perpendicular, F, from the point P on –1 . [3]

x 3
y  2 z 1
The line l has equation . The plane – 2 contains the line l and is
3 3 2
§5 ·
¨ ¸
perpendicular to a plane with normal ¨ 1 ¸ .
¨ 2 ¸
© ¹
(ii) Find sin T , where T is the acute angle between the plane – 2 and the line PF. [4]
(iii) A general point G has coordinates x , y , z . Find the position vector of N, the midpoint
of FG. [1]

(iv) Given that point N described in (iii) always lies in – 2 , find a cartesian equation that
describes the set of points which G may take. Hence, describe the relationship between
the set of points G and the plane – 2 . [4]
1 2
11 [The volume of a cone with base radius r and height h is S r h and the arc length of a sector
3
of radius r and angle T radians is r T .]

Figure 1 shows a sector AOB of T radians which is cut from a circular card of fixed radius a
metres with centre O. A cup in the shape of an inverted right circular cone with radius r and
height h is then formed by joining the two radii, OA and OB, of the sector together, without
overlap (as shown in Figure 2).

A r
a
O B
h

O
Figure 1 Figure 2

(i) Show that the volume of the cup in Figure 2, V cubic metres is given by
a3 2
V 2
T 4π 2  T 2 . [4]
24π

(ii) Use differentiation to find, in terms of a, the exact maximum volume of the cup as T
varies. You are not required to justify that the volume of the cup is a maximum. [5]

(iii) Hence, sketch the graph showing the volume of the cup, V as the angle of the sector AOB,
T varies. [3]
1 Equations & Inequalities 1 9
dxd
2 4
2 Differentiation & S
Applications 8
4 Vectors (i) 91
a  3b
(ii) c
4
(iii) 1
5 Functions (iii) R fg (f,  ln a]
(iv) x a 3
6 Graphs & Transformations 25
(i) y x  6 
x4
4 x  28 x  49
2
(iii) y
x  22
8 Differentiation & § 2 · 2
(ii) y ¨ 3 ln p ¸ x  >ln p @  ln p
2
Applications
©p ¹ 3
1
(iii) (0, )
3
3
e
(iv) units 2
36
10 Vectors §0·
¨
(i) 4
¸
¨ ¸
¨ ¸
©7¹
4 29
(ii)
299
§x ·

iiiii) ¨ 4  y ¸¸
(iii)
((i
2¨ ¸
©7  z ¹
v 2 x  4 y  3z
((iv)
(iiv) 35
11 ion &
Differentiation
attio
iat ion 2 3S a 3 3
onss
Applications (ii) m
27
TAMPINES MERIDIAN JUNIOR COLLEGE
JC1 YEAR-END EXAMINATION

CANDIDATE
NAME

CIVICS
GROUP
__________________________________________________________________________________

H2 MATHEMATICS 9758
2 October 2019
3 hours
Candidates answer on the question paper.
Additional material: List of Formulae (MF26)
________________________________________________________________________________
READ THESE INSTRUCTIONS FIRST
Write your name and civics group on all the work you hand in. For Examiners’ Use
Write in dark blue or black pen. 1
You may use an HB pencil for any diagrams or graphs.
Do not use staples, paper clips, glue or correction fluid. 2
3
Answer all the questions.
4
Write your answers in the spaces provided in the question paper.
Give non-exact numerical answers correct to 3 significant figures, or 5
1 decimal place in the case of angles in degrees, unless a different level 6
of accuracy is specified in the question.
The use of an approved graphing calculator is expected, where 7
appropriate. 8
Unsupported answers from a graphing calculator are allowed unless a
9
question specifically states otherwise.
Where unsupported answers from a graphing calculator are not allowed 10
in a question, you are required to present the mathematical steps using Total
mathematical notations and not calculator commands.
You are reminded of the need for clear presentation in your answers.

The number of marks is given in brackets [ ] at the end of each question or part question.
The total number of marks for this paper is 100.
______________________________________________________________________________

This document consists of 27 printed pages and 1 blank page.

Tampines Meridian Junior College 2019 JC1 Year-End Exam H2 Math


2

5 x2
1 Using the substitution x 5cos T , find the exact value of ³ 5
2
1
25
dx . [6]

2 Functions f and g are defined by


f :x ln x  a , x  , a  x  a 1 , where a is a positive constant,
1
g:x , x  , x d 2.
x 1
2

(i) Show that the composite function gf exists. [2]

(ii) Find gf in a similar form. [2]

(iii) Find the range of gf, showing your working clearly. [2]

x
3 The curve C has equation y 1
, 0 d x  4.
(16  x )2 4

(i) Sketch C. [2]

(ii) Find the exact volume of revolution when the region bounded by C, the line
x 2 3 and the x-axis, is rotated 2π radians about the x-axis. [4]

(iii) A horizontal line l intersects C at x 2 3 . Find the exact volume of revolution


when the region bounded by C, the horizontal line l and the line x 0 , is rotated
2π radians about the x-axis. [2]

[Turn Over
3

4 (a) State a sequence of transformations that will transform the curve with equation
1
y x 2 onto the curve with equation y 3  x 2 . [2]
4

(b) In the diagram, the graph of y f x has a maximum turning point at A(–5, 6) and

axial intercepts at B(–3, 0) and C(0, 4). The lines x 2 and y 1 are the
asymptotes of the graph. y

x
O

Sketch, on separate diagrams, the graphs of


(i) y f 3 x  5 , [3]

(ii) y f  x , [3]

stating clearly, in each case, the equations of any asymptotes and the coordinates of
the points corresponding to A, B and C.

5 Referred to the origin O, points A, B and C have position vectors a  2i  j  k ,


b 3j  3k and c i  4 j  4k respectively.
(i) Show that A, B and C are collinear. [2]

(ii) Point D lies on AB such that AD : DB = 1 : 2. Find the position vector of D. [2]

(iii) Find the area of triangle OAB. Hence write down the area of triangle OBD. [4]

(iv) Evaluate c ˜ aˆ and give a geometrical interpretation of c ˜ aˆ . [3]

[Turn Over
4

1
6 (a) Given that y ln 2  e2 x , where x  ln 2 , show that
2

ª d 2 y § dy · 2 º
ey « 2  ¨ ¸ » 4e 2 x .
«¬ dx © dx ¹ »¼

Hence, find the Maclaurin series of y, up to and including the term in x 3 . [5]

1 2x
(b) Find the expansion of in ascending powers of x, up to and including the
2  3x
term in x2 . State the range of values of x for which this expansion is valid. [5]

7 y

x
O P
The diagram shows the curve C with parametric equations

t  3
2
x , y t ln t , where t ! 0 .

(i) Find by differentiation, the value of t when the tangent to C is parallel to the
y-axis. [3]

(ii) The point P on C has coordinates 4, 0 . The normal to C at point P is denoted

by l. Find the equation of l. [3]


(iii) The line l intersects C again at point Q. Find the coordinates of point Q. [3]
(iv) Find the finite area bounded by C and the line l. [3]

[Turn Over
5

8 The function f is defined by


x 2  8 x  28
f :x for x  , x z 8.
4 x  32
(i) Find the exact x-coordinates of the turning points of y f x . [3]

(ii) Sketch the graph of y f x , labelling clearly the equations of the asymptotes and

coordinates of axial intercepts and turning points. [3]

For the rest of the question, the domain of f is restricted to 8  x d a, x  , where a is a

positive constant such that the function f 1 exists.


(iii) State the exact greatest value of a. [1]
(iv) Using the value of a found in part (iii), find f 1 x and write down the domain of

f 1 . [3]
(v) Sketch, on a single diagram, the graphs of y f x , y f 1 x and y x,
showing the relationship between the graphs. [3]

[Turn Over
6

9 (a) [It is given that the volume of a cylinder of radius r and height h is S r 2 h .]

The diagram above shows a cylinder of radius r cm and height h cm inscribed in a


spherical container with fixed radius 5 cm.

(i) Show that the volume, V cm3, of the cylinder is given by V 2πr 2 25  r 2 .
[2]
(ii) As r varies, use differentiation to find the exact value of r that gives a
maximum V of the cylinder. Hence find the exact maximum value of V,
showing that this value is a maximum. [6]

(b) y

x
O

The diagram shows the graph of y f x , which is strictly decreasing on the

1
interval 2,1 . Given that g x sin ª¬ 2f x º¼ , where 2  x  1 , use
2
differentiation to determine the number of stationary points on the graph of
y g x . [4]

[Turn Over
7

[Turn Over
8
G F
10

E
D

k
j C B

O i A

The diagram shows a structure with a horizontal square base OABC and a horizontal
square top DEFG, where OA 2 m and DE 6 m . The vertical height of the structure
is 8 m. Each sloping face of the structure is a trapezium, and the edges AE, BF, CG and
OD are of equal length.
The point O is taken as the origin and perpendicular unit vectors i, j, k are such that i and
j are parallel to OA and OC respectively. The coordinates of points D, E, F and G can be
expressed as  p,  p, 8 , 2  p,  p , 8 , 2  p, 2  p , 8 , and  p, 2  p, 8

respectively.
(i) Explain why p 2. [1]
(ii) Find the Cartesian equation of plane ABFE. [3]
(iii) Given that the Cartesian equation of plane OAED is 4 y  z 0 , determine the
obtuse angle between planes OAED and ABFE. [2]

A rod connects point M, the midpoint of edge FG, to a point N on the plane OAED such
that MN is made as small as possible.
(iv) Find the coordinates of N and the exact minimum length of the rod. [5]

The structure sits on the ground and is secured by a straight cable connecting a point on
the ground to the point D. This cable coincides with the line of reflection of the line
segment DM in the plane OAED.
(v) Using your answer in part (iv) or otherwise, find a vector equation representing the
straight cable. [3]
End of Paper

[Turn Over
2019 H2 MATH (9758/01) JC 1 YEAR-END EXAMINATION – SUGGESTED SOLUTIONS
Qn Solution
1 Integration by Substitution
dx
x 5 cos T Ÿ 5sin T
dT
When x 5 , 5 5cos T Ÿ cos T 1 Ÿ T 0
5 5 1 π
When x , 5cos T Ÿ cos T ŸT
2 2 2 4
5 x2 0
³ 5 1 dx ³π 1  cos 2 T 5sin T dT
2 25 4
0
³ π
4
5sin 2 T dT
π
1
5³ 4 1  cos 2T dT
0 2
π
5ª 1 º4
«T  sin 2T »
2¬ 2 ¼0
5§π 1 π · 5π 5
¨  sin  0  0 ¸ 
2© 4 2 2 ¹ 8 4

Page 1 of 15
Qn Solution
2 Functions
(i) y x=a

y = f (x)

x
O (a+1, 0)

Rf = (∞, 0), Dg = (∞, 2].


Since Rf Ž Dg Ÿ gf exists.
(ii)
gf(x) g(ln( x  a))
1
[ln( x  a)]2  1
1
gf : x , x  , a  x  a 1
[ln( x  a)]2  1
(iii) f
Df  o Rf g
(f, 0)  o(0,1) R gf
? R gf (0,1)

y
y = g(x) (0,1)

y=0 O x

Page 2 of 15
Qn Solution
3 Volume generated given a bounded region (Definite Integrals)
(i) x
y 1
(16  x )
2 4

y
x=4

(0,0)
x

(ii) Volume generated


2
ª º
2 3« x »
π³ « 1 » dx
« 16  x »
0
2 4
¬ ¼
π 2 3 ª 2 x º
 ³ « » dx
2 0 ¬ 16  x 2 ¼
2 3
ª º
π « 16  x 2 »
 « »
2« 1
»
¬ 2 ¼0
ª º

2
 π « 16  2 3  16 »
¬ ¼
π 2  4

(iii) Volume generated
2
ª º
« »
π«
«
2 3 » 2 3  2π
»


1


2
« 16  2 3 4
»
¬« ¼»
§2 3·
𠨨
¸¸ 2 3  2π
© 2 ¹

Page 3 of 15
Qn Solution
4 Transformation of Curves
(a) 1
(1) Stretch/Scaling of factor parallel to the y-axis. OR
4
Stretch/Scaling of factor 2 parallel to the x-axis.
(2) Translation of 3 units in the positive y-direction,

*or (2),(1)
(bi) y

x
O

(bii) y

O x

Page 4 of 15
Qn Solution
5 Vectors
(i) For A, B and C to be collinear, there exists O such that

o 
o
AC O AB
§ 1 · § 2 · § 3· §1·

o
¨ ¸ ¨ ¸ ¨ ¸ ¨ ¸
AC ¨ 4 ¸¨ 1 ¸ ¨ 3 ¸ 3¨ 1 ¸
¨ 4 ¸ ¨ 1 ¸ ¨ 3 ¸ ¨ 1 ¸
© ¹ © ¹ © ¹ © ¹
§ 0 · § 2 · § 2· §1·

o
¨ ¸ ¨ ¸ ¨ ¸ ¨ ¸
AB ¨ 3 ¸¨ 1 ¸ ¨ 2¸ 2¨ 1 ¸
¨ 3 ¸ ¨ 1 ¸ ¨ 2 ¸ ¨ 1¸
© ¹ © ¹ © ¹ © ¹

o 3 o
? AC AB
2
Hence A, B and C are collinear.
(ii) Using ratio theorem and 'OAB ,
ª § 2 · § 0 · º § 4 ·
1 « ¨ ¸ ¨ ¸» 1¨ ¸
OD 2 1  3 5
3 « ¨¨ ¸¸ ¨¨ ¸¸ » 3 ¨¨ ¸¸
«¬ © 1 ¹ © 3 ¹ »¼ © 5 ¹
(iii) 1
Area of triangle OAB aub
2
§ 2 · § 0 ·
1¨ ¸ ¨ ¸
1 u 3
2 ¨¨ ¸¸ ¨¨ ¸¸
© 1 ¹ © 3 ¹
§ 0·
1 ¨ ¸
2 ¨ 6 ¸
¨ 6 ¸
© ¹
§0·
¨ ¸
3 ¨ 1 ¸ 3 2
¨ 1 ¸
© ¹
Area of triangle OBD 2 2
(iv) § 1· § 2 ·
¨ ¸ 1 ¨ ¸
c ˜ aˆ ¨ 4 ¸ ¨ 1¸
¨ 4 ¸ 6 ¨ 1 ¸
© ¹ © ¹
1
= 2  4  4
6
= 6
c ˜ aˆ is the length of projection of c onto a.

Page 5 of 15
Qn Solution
6 Maclaurin Series and Binomial Theorem
(a) y ln 2  e2 x
Differentiate with respect to x:
dy 2e 2 x
dx 2  e2 x
2e 2 x
ey
dy
ey 2e2 x
dx
Differentiate with respect to x:
d 2 y dy § y dy ·
ey  ¨e ¸ 4e
2x

dx 2 dx © dx ¹
2 2
y d y y § dy ·
e 2
 e ¨ ¸ 4e 2 x
dx © dx ¹
ª d 2 y § dy · 2 º
ey « 2  ¨ ¸ » 4e 2 x
«¬ dx © dx ¹ »¼
Differentiate with respect to x:
dy d 2 y · ª d 2 y § dy · º § y dy ·
2
§ d3 y
ey ¨ 3  2 
¸ «  ¨ ¸ »¨e ¸ 8e 2 x
© dx dx dx 2 ¹ ¬« dx 2 © dx ¹ ¼» © dx ¹
ª d3 y dy d 2 y § dy · º
3

e « 3 3
y
¨ ¸ » 8e 2 x
¬« dx dx dx 2 © dx ¹ ¼»
dy d2 y d3 y
When x 0 , y = 0, 2 , 8 , 48
dx dx 2 dx3
Maclaurin series for y is
§ x2 · § x3 ·
y 2 x  8 ¨ ¸  48 ¨ ¸  ...
© 2! ¹ © 3! ¹
y 2x  4x  8x  ...
2 3

(b) 1 2x
2  3x
1
1  2 x 2 2  3x 1
1
1  2 x 2 §¨1  x ·¸
1 1 3
2 © 2 ¹
§ 1§ 1· ·
¨ ¨  ¸ ¸§ 3 2 ·
1 1
¨1  2 x  2 © 2 ¹ 2 x 2  ... ¸ §3 ·
... 1  x  ¨ x ¸  ... ¸
¨
2¨ 2 2!
2! ¸ ¨© 2 ©2 ¹ ¸
¹
¨ ¸
© ¹
1§ 1 2 ··§§ 3 9 ·
¨1  x  x  .... ¸¨1  x  x 2  ... ¸
2© 2 ¹© 2 4 ¹
1§ 3 9 2 3 2 1 2 ·
¨1  x  x  x  x  x  ... ¸
2© 2 4 2 2 ¹
1§ 5 13 2 ·
¨1  x  x  ... ¸
2© 2 4 ¹
1 5 13
 x  x 2  ...
2 4 8

Page 6 of 15
1 1
The expansion of 1  2x 2 is valid for 2 x  1 Ÿ x 
2
1 3x 2
The expansion of 2  3x is valid for 1Ÿ x 
2 3
1 1 2 2
 x  and   x 
2 2 3 3
1 1
Ÿ x
2 2
1 2x 1 1
Therefore, the expansion of is valid for   x  .
2  3x 2 2

Page 7 of 15
Qn Solution
7 Tangent & Normal, Integration
(i) dx
x t  3 Ÿ 2 t  3
2

dt
dy t
y t ln t Ÿ ln t  ln t  1
dt t
dy ln t  1
dx 2 t  3
Tangent parallel to the y-axis Ÿ 2 t  3 0
t=3
(ii) 1 2 t  3
Gradient of normal
§ dy · ln t  1
¨ ¸
© dx ¹
At (4, 0),
4 t  3
2
Ÿ t  3 r2, t 1 or t 5 (reject since y 5ln 5 z 0)
OR
t ln t 0 Ÿ t 0 (reject t > 0) or t 1
Equation of normal at point P
2 1  3
y  1 ln 1 ª x  1  3 2 º
ln 1  1 ¬ ¼
Ÿy 4 x  4 Ÿ y 4 x  16
(iii) y 4 x  16    (1)
t  3 , y
2
Substi. x t ln t into (1):
4 t  3  16
2
t ln t

Using G.C., t = 5.52164


When t = 5.52164,
5.52164  3 6.35867 6.36
2
x .3
y 5.52164 ln 5.52164 9
9.43
4346
43469
46
9.43469 9.43
Coordinates of Q is (6.
(6.36,
6.366, 9.43
6. 99.
9.43)
.4433) (3 s.f.)
(iv) y
Q

x
O P
Required Area

Page 8 of 15
§ y  16 ·
9.43469 9.43469
³0 ¨
© 4 ¹
¸ dy  ³
0
xC dy

9.43469 § y  16 · 5.52164
³0 ¨© 4 ¸¹ ³1 t  3 ln t  1 dt
2
dy 

31.3995 31.4 units 2 3 s.f.

Qn Solution
8 Functions
(i) x 2  8 x  28
y
4 x  32
dy 4 x  32 2 x  8  x  8 x  28 4
2

4 x  32
2
dx
At turning points,
dy 4 x  32 2 x  8  x  8 x  28 4
2

=0
4 x  32
2
dx
4 x 2  64 x  144 0
 64 r 64  4 4 144
2

x
2 4
x 8r 2 7
(ii)
y

x
(0, −0.875)
(iii) 8  x d 8 2 7
a 8 2 7 x=8
(iv) x 2  8 x  28
8
y
4 x  32
32
y 4 x  32 x 2  8 x  28
28
x 2  8  4 y x  28  32 y 0
8  4 y r 4 4  4 y  y2  7  8 y
x
2
4  2 y r 2 y2  4 y  3

Since 8  x d 8  2 7 ,
x 4  2 y  2 y2  4 y  3
Page 9 of 15
I.e. f 1 x 4  2x  2 x2  4 x  3
Domain of f 1 = Range of f = [4.65, f)
(v) y
‫ ݕ‬ൌ ˆሺ‫ݔ‬ሻ ‫ݕ‬ൌ‫ݔ‬
൫ͶǤ͸ͷǡ ͺ ൅ ʹξ͹൯

‫ ݕ‬ൌ ˆ ିଵ ሺ‫ݔ‬ሻ
‫ݕ‬ൌͺ

൫ͺ ൅ ʹξ͹ǡ ͶǤ͸ͷ൯

x
‫ݔ‬ൌͺ
O

Qn Solution
9 Application of Differentiation
(i)
r

2
§h· Alternatively,
r 2  ¨ ¸ 52 2
©2¹ §h·
r  ¨ ¸ 52
2

h2 ©2¹
r2  25
4 h
since h ! 0, 25  r 2
4r 2  h 2 100 2
? since h ! 0, h 100  4r 2 h 2 25  r 2

4 25  r 2

2 25  r 2

V πr 2 h


5  r2
πr 2 2 25

2πr 25  r ((shown)
2
25 2
shown)
(ii) dV
For maximum
um
m vvolume,
ol 0
dr

Page 10 of 15
dV ª § 1 ·§ 1 · º
2S « 2r 25  r 2  r 2 ¨ ¸ ¨ ¸ 2r »
dr «¬ © 2 ¹ © 25  r 2 ¹ »¼
r3
2r 25  r 2  0
25  r 2
r3
2r 25  r 2
25  r 2
2r 25  r 2 r3
50r 3r 3
50
r2 , rz0
3
50 5 6
since r ! 0, r
3 3

 
50 50 50
r
3 3 3
dV
dr
50
?r gives a maximum volume.
3

Method 2 (2nd derivative test)


d 2V
217.65  0
dr 2

50
?r gives a maximum volume.
3

2 2
§ 50 · § 50 ·
Maximum volume, V 2𠨨 ¸¸ 25  ¨¨ ¸¸
© 3 ¹ © 3 ¹
500
π
3 3
500 3
500
π
9
(b) 1
g x 2ff x º¼
in ª¬ 2
ssin
2
1
g' x 2 f ' x cos ª¬2f x º¼ 0
2

Since f is strictly decreasing on 2,1 , f ' x  0 and

Page 11 of 15
for cos ª¬2f x º¼ 0 ,
π 3π 5π
2f x ..., , , ,...
2 2 2
π 3π 5π
f x ..., , , ,...
4 4 4
5 3π
Since 1  f x  for 2  x  1 , ? f x
2 4
Hence there is one stationary point on the graph y g x .

Qn Solution
10 Vectors
(i) G F
From the top view,
2 p DE  OA
C B
62
p 2 (shown)
2
O A

D E
(ii) § 2· § 2· § 4·
¨ ¸ ¨ ¸ ¨ ¸
OA ¨ 0 ¸ , OB ¨ 2 ¸ , OE ¨ 2 ¸
¨0¸ ¨0¸ ¨ 8¸
© ¹ © ¹ © ¹
§ 2· § 2· § 0· § 4 · § 2· § 2·
¨ ¸ ¨ ¸ ¨ ¸ ¨ ¸ ¨ ¸ ¨ ¸
AB ¨ 2 ¸  ¨ 0 ¸ ¨ 2 ¸ , A
AE ¨ 2 ¸  ¨ 0 ¸ ¨ 2 ¸
¨0¸ ¨0¸ ¨0¸ ¨ 8 ¸ ¨0¸ ¨ 8¸
© ¹ © ¹ © ¹ © ¹ © ¹ © ¹
§ 0 · § 2 · § 16 ·
¨ ¸ ¨ ¸ ¨ ¸
AB u AE ¨ 2 ¸ u ¨ 2 ¸ ¨ 0 ¸
¨ 0 ¸ ¨ 8 ¸ ¨ 4 ¸
© ¹ © ¹ © ¹
§4·
A normal vector to the plane is ¨¨ 0 ¸¸ .
¨ 1¸
© ¹
§ 4 · § 2· § 4 ·
¨ ¸ ¨ ¸¨ ¸
r ¨ 0 ¸ ¨0¸ ¨ 0 ¸ 8
¨ 1¸ ¨ 0 ¸ ¨ 1¸
© ¹ © ¹© ¹
? The Cartesian
rtesi
sian
si
iaan
n eequation
quat
qu atio
ion of
of plane
plan
plane ABFE
an A is 4 x  z 8.

Page 12 of 15
(iii) §0·
¨ ¸
4y  z 0 Ÿ r ¨4¸ 0
¨1¸
© ¹
§ 4 · §0·
¨ ¸¨ ¸
¨ 0 ¸ ¨ 4¸
¨ ¸¨ ¸
1 © 1 ¹ © 1 ¹
Angle between planes OAED and ABFE cos
17 17
§ 1 ·
cos 1 ¨  ¸
© 17 ¹
93.4q (to 1 d.p.)
(iv) Coordinates of M = (1, 4, 8)
§1· §0·
Equation of line MN: r ¨ 4 ¸  O ¨¨ 4 ¸¸ , O 
¨ ¸ ---(1)
¨8¸ ¨1¸
© ¹ © ¹
§0·
Equation of plane OAED: r ¨ 4 ¸¸
¨
0 ---(2)
¨1¸
© ¹
Solving (1) and (2),
ª§ 1 · § 0 ·º § 0 ·
«¨ ¸ ¨ ¸» ¨ ¸
«¨ 4 ¸  O ¨ 4 ¸ » ¨ 4 ¸ 0
«¬¨© 8 ¸¹ ¨ 1 ¸» ¨ 1 ¸
© ¹¼ © ¹
Ÿ 24  17O 0
24
ŸO 
17
§ ·
¨ 1 ¸
§1· §0· ¨ ¸
¨ ¸ 24 ¨ ¸ ¨ 28 ¸
ON ¨ 4 ¸  ¨ 4 ¸ 
¨ 8 ¸ 17 ¨ 1 ¸ ¨¨ 17 ¸¸
© ¹ © ¹
¨ 112 ¸
© 17 ¹
§ 28 112 ·
?Coordinates of point N = ¨1,  , ¸
© 17
7 17 ¹

§ · § ·
¨ 1 ¸ ¨ 0 ¸
¨ ¸ §1· ¨ ¸
28
2 8 ¨  96 ¸ 24
ble = ¨  ¸  ¨ 4 ¸
Length of cabl
cable
ca
abl
17 ¸ ¨¨ ¸¸
¨ 17 ¨ 17 ¸ 17
¨ 112
1 2¸ © ¹
11
8 ¨ 24 ¸
¨ ¸ ¨ ¸
© 17 ¹ © 17 ¹

Page 13 of 15
Alternatively,
§ 1 · § 2 · § 3·
¨ ¸ ¨ ¸ ¨ ¸
DM ¨ 4 ¸  ¨ 2 ¸ ¨ 6¸
¨8¸ ¨ 8 ¸ ¨ 0¸
© ¹ © ¹ © ¹
§ § 3· § 0· · § 0·
¨¨ ¸ ¨ ¸¸ ¨ ¸
¨ ¨6¸ ¨ 4¸ ¸ ¨ 4¸
¨ ¨© 0 ¸¹ ¨© 1 ¸¹ ¸ ¨© 1 ¸¹
NM ¨ ¸
¨ 17 ¸ 17
¨ ¸
¨¨ ¸¸
© ¹
§0·
24 ¨ ¸
4
17 ¨¨ ¸¸
©1¹
§0·
24 ¨ ¸
OM  ON 4
17 ¨¨ ¸¸
©1¹
§ ·
¨ 1 ¸
§1· §0· ¨ ¸
¨ ¸ 24 ¨ ¸ ¨  28 ¸
ON ¨ 4 ¸  17 ¨ 4 ¸ ¨ 17 ¸
¨8¸ ¨1¸ ¨ 112 ¸
© ¹ © ¹
¨ ¸
© 17 ¹
§ 28 112 ·
?Coordinates of point N = ¨1,  , ¸
© 17 17 ¹
§ 3· § 0·
¨ ¸¨ ¸
¨ 6¸ ¨ 4¸
¨0¸ ¨1¸
© ¹© ¹ 24
Length of cable
17 17
(v) By ratio theorem,
OMM  OM '
ON
2
OM ' 2 ON  OM
2ON
§ · § ·
¨ 1 ¸ ¨ 1 ¸
¨ ¸ §1· ¨ ¸
¨ 288¸ ¨ ¸ 12
1 24 ¸
¨  124
2   4
17 ¸ ¨¨ ¸¸
¨ 17 ¨ 1717 ¸
¨ 112
12 ¸ © ¹
11
8 ¨ 88
8 ¸
¨ ¸ ¨ ¸
© 17
7 ¹ © 17 ¹

Page 14 of 15
DM ' OM '  OD
§ · § ·
¨ 1 ¸ ¨ 3 ¸
¨ ¸ § 2 · ¨ ¸ § 17 ·
¨  124 ¸  ¨ 2 ¸ ¨  90 ¸ 3¨
 ¨ 30 ¸
¸
¨ 17 ¸ ¨ ¸ ¨ 17 ¸ 17 ¨
¨ 88 ¸ ©¨ 8 ¹¸ ¨ 48 ¸ © 16 ¹
¸
¨ ¸ ¨ ¸
© 17 ¹ © 17 ¹

A vector equation of the reflection of line DM in the plane OAED is


§ 2 · § 17 ·
¨ ¸ ¨ ¸
r ¨ 2 ¸  P ¨ 30 ¸ , P  .
¨ 8¸ ¨ 16 ¸
© ¹ © ¹
§ ·
¨ 1 ¸
¨ ¸ § 17 ·
¨ 124 ¸  P ¨ 30 ¸ , P 
Also accept r ¨ ¸
¨ 17 ¸ ¨ ¸
¨ 88 ¸ © 16 ¹
¨ ¸
© 17 ¹

Page 15 of 15
2019 TJC Promotional Examination H2 Mathematics

Dx
1 (i) Sketch the curve with equation y , where D is a positive constant, stating
x 1
the equations of the asymptotes. On the same diagram, sketch the line with equation
y = Dx  2. [3]
Dx
(ii) Solve the inequality t D x  2 , giving your answers in term of D. [3]
x 1

2 Interpret geometrically the vector equation r = a + Pm where a and m are constant vectors
and P is a parameter. [2]

Referred to the origin O, the points A and B have position vectors a and b respectively,
such that a and b are non-parallel vectors. The point C lies on the line AB such that the
area of the triangle OBC is 6 units2. Given that a is a unit vector, |b| = 4 and the angle
between a and b is 30q, find the possible position vectors of C in terms of a and b. [6]

3 (a) The diagram shows the curve y f ( x) , where a is a positive constant. The curve
has a minimum point at A(a, 3a), a maximum point at B(a, 0) and cuts the x-axis
at the point C(3a, 0).
y
B(a, 0)
x
C(–3a, 0) O

A(–a, 3a)

Sketch, labelling each graph clearly and showing the coordinates of the points
corresponding to A, B and C whenever possible, the graphs of
(i) y 3f ( x  a) , [2]
§ x·
(ii) y f¨ ¸, [2]
©2¹
1
(iii) y . [2]
f ( x)

(b) The curve with equation y 2  e x is reflected in the line y = 5. Find the equation
of the reflected curve. [2]
2

4 It is given that

­ 4x  2 for 0 d x d 1,
°
f ( x) ® 12
° x x for 1  x d 4,
¯

and that f ( x  4) f ( x) for all real values of x.

(i) Sketch the graph of y f ( x) for 2  x d 6 . [3]


4
(ii) Use the substitution u x to find the exact value of ³0 f ( x) dx . [5]

5 (a) Find ³ sin x cos 3x dx . [2]


x 1
(b) Find ³ 1  2x  x2
dx . Find the greatest integer value of b such that

b x 1
³0
1  2x  x2
dx is defined. [5]

2S
(c) Find ³ x cos x dx . Hence find the exact value of ³
0
x cos x dx . [5]

6 (a) In an arithmetic progression, the 8th term is 20 and the 27th term is greater than the
15th term by 24. It is given that the sum of the first n terms is greater than the sum
of the 8th to the 40th term by more than 1218. Find the smallest value of n. [6]

(b) An infinite geometric progression is such that the sum of all the terms after the nth
term is equal to twice the nth term. Show that the sum to infinity of the progression
is three times the first term. [3]

© TJC 2019
3

7 A napkin-holder is formed by boring a cylindrical hole, of length 2h, through a wooden


sphere of radius a, where a is a fixed constant. The axis of the hole passes through the
centre O of the sphere. The diagram shows a cross-section through O, with x- and y-axes
taken parallel and perpendicular to the axis of the hole respectively.
y

x
a h O h a

(i) Let S denote the napkin holder’s total surface area, which is made up of its internal
(cylindrical) area and its external (spherical) area. It is given that the external surface
area is 4S ah.
(a) Show that S
4S h a  a 2  h 2 . [2]

(b) Use differentiation to find, in terms of a, the exact maximum value of S as h


varies. [You do not need to verify that this value of S is the maximum.] [4]

(ii) Let V denote the volume of the wood forming the napkin-holder. By considering the
napkin-holder as a solid of revolution about the x-axis, find V in terms of h, verifying
that it is independent of a. [4]

8 Functions f and g are defined by

x2  6 x  6
f :x , x  , x z 1 ,
x1 1
ax  1
g:x , x  , x z b ,
xb
where a and b are constants.
(i) Sketch the graph of y f ( x) , giving the coordinates of the turning points and the
equation of the asymptotes. Write down the range of f. [3]
(ii) Find the value of a and the range of values of b such that both composite functions
fg and gf exist. [4]
(iii) Find g1(x). Given that g1(x) = g(x) for all real x, x z b , find b in terms of a. [3]

© TJC 2019
4

A
9 Wall Wall

20
k

i O j 10
20 C

B Floor

A shoe store owner plans to install a triangular mirror ABC with negligible thickness at
one of the floor corner of his shop to allow his customers to view the fitting of their
selected shoes. Points (x, y, z) are defined relative to the corner point at O where the two
vertical walls, which are perpendicular to each other, and the horizontal floor meet. The
z-axis points vertically upwards. The x-axis and y-axis are the intersections of the floor
with the two walls. A , B and C lie on the z-axis, x-axis and y-axis and are 20 units, 20
units and 10 units from O respectively. The units of length are measured in inches.
(i) Find the cartesian equation of the face of the mirror ABC. Hence find the exact
shortest distance from O to the face of the mirror ABC. [4]
(ii) Find the coordinates of the point N on the face of the mirror ABC which is nearest to
O. [2]
(iii) Find the acute angle between the face of the mirror ABC and the floor. [2]
As a safety measure, a triangular plank OBR is installed to support the mirror, where R is
a point between A and C such that AR : RC P :1  P . The face of the mirror ABC meets
the plank OBR on l.
(iv) Given that N in (iii) lies on l, find the coordinates of R. [5]

10 A curve C has parametric equations


x 2t  sin 2t , y cos 2t , for 0 d t d S .
dy
(i) Show that  tan t. What can be said about the tangent to C at the point where
dx
S
t . [4]
2
(ii) Find the exact x-coordinates, x1 and x2 where x1  x2 , of the two points where C cuts
the x-axis. [2]

(iii) Sketch C, indicating the exact coordinates of the end-points. [2]


(iv) Find the exact area of the region bounded by C and the x-axis. [5]

(v) Find the value of t at which the tangent to C at the point x x1 intersects C
again. [3]
End of Paper
© TJC 2019
2019 TJC Promotional Examination H2 Mathematics [Solution]

1 [Solution]
(i) y
Dx D
y D
x 1 x 1

y Dx2
y D

x
O 2
D
2

x 1

(ii) Dx
tDx2
x 1
Dx
Solve Dx2
x 1
D x2  2 x  2 0
2 r 4  8D 1 r 2D  1
x
2D D

1  2D  1
x (Larger value of x)
D

1  2D  1
From graph, x d , x z 1
D

1  2D  1
(or equivalent form x  1 or 1  x d )
D
2

2 [Solution]
Vector equation r = a + Pm:
The equation gives the set of position vectors of the points on the line which passes
through the point with position vector a and is parallel to m.

Equation of line AB: r = a + P(b  a) = (1 P)a + Pb


Since C lies on line AB, c = (1 P)a + Pb for some P

OR using ratio theorem, c = (1 P)a + Pb for some P

Area of triangle OBC


1
cub
2
1
[(1  P )a  Pb] u b
2
1
(1  P )a u b  Pb u b
2
1
(1  P ) a u b since bub = 0
2
1
(1  P ) 1 4 sin 30q
2
(1  P )

(1  P ) 6
1  P = 6 or 1  P = 6
P =  5 or 7

? OC 6a  5b or OC 7b  6a

Alternatively
1 1
Area of triangle OAB aub a b sin 30q =1
2 2
Since A, B, C are collinear and Area triangle OBC = 6,

Area of 'OBC 6
Areaa of 'OAB
O 1
1
h BC BC
2 6 6
Ÿ
1
h AB 1 AB 1
2

© TJC 2019
3

By ratio Theorem, A
1
Case 1: B
6OA  OC
6OA 6
OB Ÿ OC 7b  6a
7
C

Case 2: C

5OB  OC
5OB 5
OA Ÿ OC 6a  5b
6
A1
B

© TJC 2019
4

3 [Solution]
(a)(i) y 3f ( x  a)

y
B1(2a, 0)
x
C1(–2a, 0) O

A1(0, 9a)

y
(ii) § x·
y= f¨ ¸
©2¹
(2a, 0) B2(2a, 0)
x
O

1
(iii) y
f ( x)
y

ଵ x
A3(–a, െ )
y=0 ଷ௔ O

x = –3a x=a
(b)
y = 2 + ex
o y = 2 + ex  5 = 3 + ex
o y = (3 + ex) = 3  ex
o y = 3  ex + 5 Ÿ y = 8  ex

© TJC 2019
5

Alternative method
y = 2 + ex
o y = (2 + ex)
o y = (2 + ex) +10 Ÿ y = 8  ex

4 [Solution]
(i) y

2
x
2 O 1 4 5 6

(ii) 4 1 4 12
³0 f ( x) dx 2  6 (1)  ³1 dx
2 x x

du 1 1
Let u x Ÿ
dx 2 x 2u
(giving dx = 2u du)

When x = 1, u = 1
When x = 4, u = 2

4 12 2 1
³1 x x
dx 12 ³
1 2
u u
2u du

2 1
24³ du
1 u 1

24 > ln(u  1)@1


2

§3·
24 ln ¨ ¸
©2¹
4 §3·
? ³0 f ( x ) dx 4  24ln ¨ ¸
©2¹

© TJC 2019
6

5 [Solution]
(a)
³ sin x cos 3x dx ³ cos 3x sin x dx
1

sin 4 x  sin 2 x dx

1 1
 cos 4 x  cos 2 x  c
8 4

(b) x 1 1 2(1  x)
³ 1 2x  x 2
dx  ³
2 1  2 x  x2
dx

1 1  2 x  x 2
2

 c
2 1
2

 1  2 x  x2  c

x 1
For to be defined,
1  2 x  x2
1  2x  x2 ! 0
( x  1) 2  2  0
x  1  2 x  1  2  0
Ÿ 1 2  x  1 2
Greatest integer value of b is 2

(c)
³ x cos x dx x sin x  ³ sin x dx
x sin x  cos x  c

2S S 3S 2S
³ ³ x cos x dx  ³S x cos x dx  ³3S x cos x dx
2 2
x cos x dx
0 0 2 2

S 3S
> x sin x  cos x @0  > x sin x  cos x @
2
S
2
2

2S
 > x sin x  cos x @3S (using above result)
2

§S · § 3S S · § 3S ·
¨  1¸  ¨   ¸  ¨1  ¸
©2 ¹ © 2 2¹ © 2 ¹
4S

© TJC 2019
7

6 [Solution]
(a) T8 a  7d 20 --- (1)

T27  T15 24
(a  26d )  (a  14d ) 24
12d24
Ÿ d 2

Subst into (1): a 20  7(2) 6


40
S40 >12  39(2)@ 1800
2
7
S7 >12  6(2)@ 84
2
Sum of T8 to T40 S 40  S7 1800  84 1716

n
>12  (n  1)(2)@  1716 ! 1218 (more than 1218)
2
n 2  5n  2934 ! 0
Method 1
n n2  5n  2934
51 78 < 0
52 30 > 0
Using GC, the smallest value of n is 52

Method 2
n2  5n  2934 ! 0

n  56.7 or n ! 51.7 n
56.7 51.7
? smallest value of n is 52

(b) Let the first term be a and the common ratio be r.


Given Sf  Sn = 2Tn
a


a 1 rn 2ar n 1
1 r 1 r
ar n r 2
2ar n 1 Ÿ 2Ÿr
1 r 1 r 3
a
Sf 3a
§2·
1 ¨ ¸
©3¹
Hence, sum to infinity is 3 times of the first term.

© TJC 2019
8

7 [Solution]
(i)(a) y

a
x
a h O h a

Radius of cylinder a 2  h2
Internal cylindrical area 2S a 2  h 2 2h 4S h a 2  h 2
?S 4S ah  4S h a 2  h 2


4S h a  a 2  h 2 (shown)


(b) dS
dh

4S a  a 2  h 2  4S h ˜ ¨
1 § 2h ·
¸
2 © a 2  h2 ¹
§ a 2  h2  h2 ·
4S ¨ a  ¸
© a 2  h2 ¹
§ a 2  2h 2 ·
4S ¨ a  ¸
© a 2  h2 ¹
dS
Let 0
dh
2h 2  a 2
a
a 2  h2
2h  a2 a 2 a 2  h2
2 2

4h 4  4h 2 a 2  a 4 a 4  a 2h2
4h 4  3h 2 a 2 0
h 2 (4h 2  3a 2 ) 0
3a 2
Since h z 0, h 2

4
3
h a ( h ! 0)
2

§ 3 ·§ 3 2·
Max value of S 4S ¨¨ a ¸¨
¸¨ a  a 2
 a ¸
© 2 ¹© 4 ¸¹
§ 1 ·
2 3S a ¨ a  a ¸
© 2 ¹
3 3S a 2

© TJC 2019
9

(ii) Equation of circle is x2  y 2 a2 Ÿ y 2 a2  x2

2 u S ³ a 2  x 2 dx  S (2h)
h 2
V a 2  h2
0
h
ª x3 º
2S « a 2 x  »  2S h(a 2  h 2 )
¬ 3 ¼0
§ h3 ·
2S ¨ a 2 h  ¸  2S ha 2  2S h3
© 3¹
4 3
S h which is independent of a
3

8 [Solution]
(i) 1
y f ( x) x7
x 1
x = 1
y

O
x

(0,6)

(2,10)

y = x 7

Rf = f, 10@ ‰ > 6, f

© TJC 2019
10

(ii) ax  1 1  ab
g(x) a y
xb xb
x = b

x
y=a

Rg = \ ^a`
Df = \ ^1`
For fg to exist, Rg Ž Df
i.e \ ^a` Ž \ ^1`
Hence, a = 1

Rf = f, 10@ ‰ > 6, f


Dg = \ ^b`
For gf to exist, Rf Ž Dg
i.e f, 10@ ‰ > 6, f Ž \ ^b`
Hence 10 < b < 6
Ÿ 6 < b < 10

(iii) ax  1
Let y
xb

xy  by ax  1
1  by
x
ya
1  bx
? g 1 ( x)
xa

Given g 1 ( x) g( x) for all real values of x,


1  bx ax  1
xa xb
By comparison, b = a

© TJC 2019
11

9 [Solution]
§0· § 20 · §0·
(i) ¨ ¸ ¨ ¸ ¨ ¸
OA ¨ 0 ¸ , O
OB ¨ 0 ¸, OC ¨10 ¸
¨ 20 ¸ ¨0¸ ¨0¸
© ¹ © ¹ © ¹
§ 220 ·
¨ ¸
AB OB  OA ¨ 0 ¸ ,
¨ 20 ¸
© ¹
§ 0 ·
¨ ¸
AC OC  OA ¨ 10 1 ¸
¨ 20 ¸
© ¹
§ 20
200 · §1·
¨ ¸ ¨ ¸
AB u AC ¨ 40
400 ¸ 200 ¨ 2 ¸
¨ 200 ¸ ¨1¸
© ¹ © ¹
§1· § 20 · §1·
¨ ¸ ¨ ¸ ¨ ¸
r ¨ 2¸ ¨0¸ ¨ 2¸ 20
¨1¸ ¨0¸ ¨1¸
© ¹ © ¹ © ¹

? Equation of the face ABC is x + 2y + z = 20

Shortest distance from O to ABC


20 20
=
1  2 1
2 2 2
6

(ii) Method 1 (Using Unit vector)


§1· § 10 ·
20 1 ¨ ¸ 1 ¨ ¸
Using (i), ON ˜ 2 20
6 6 ¨¨ ¸¸ 3 ¨¨ ¸¸
©1¹ © 10 ¹
§ 10 20 10 ·
? Coordinates of N is ¨ , , ¸
© 3 3 3¹

Alternative Solution
n
§1·
¨ ¸
N : r O ¨ 2¸, O 
Line ON
¨1¸
© ¹
§1· §1· §1·
¨ ¸ ¨ ¸¨ ¸
ON ¨ 2 ¸ 200 Ÿ O ¨ 2 ¸ ¨ 2 ¸ 20
¨1¸ ¨1¸ ¨1¸
© ¹ © ¹© ¹
10
Ÿ 6O 10 Ÿ O
3
§ 10 20 10 ·
Coordinates of N is ¨ , , ¸
© 3 3 3¹

© TJC 2019
12

(iii) Let T be the required acute angle.


§1· §0·
¨ ¸¨ ¸
¨ 2¸ ¨ 0¸ n1 ˜ n 2
¨1¸ ¨1¸ cos T
© ¹© ¹ 1
cos T n1 n 2
6 1 6
T 65.9q (1 d.p.)

A
Wall Wall

20
k

i O j 10
20 C

B Floor
(iv)
A
Wall Wall
1 P

R
20
k
P
i O j 10
20 C

B Floor

Method 1
(Use concept of point R is a p
point
oin on plane OBR)

§ 0 ·
¨ ¸
Using ratio theorem,
heo
eo
oreem,
m, OR
OR PO
OC (1  P )OA ¨ 10P ¸
C  (1
¨ 20(1  P ) ¸
© ¹

§1· §1· §0·


¨ ¸ ¨ ¸ ¨ ¸
Normal of plane
l OBR = ¨ 0 ¸ u ¨ 2 ¸ ¨ 1¸
¨0¸ ¨1¸ ¨2¸
© ¹ © ¹ © ¹

§0·
¨ ¸
Equation of plane OBR is r ˜ ¨ 1¸ 0
¨2¸
© ¹
© TJC 2019
13

Since point R is a point on plane OBR,


§ 0 · §0·
¨ ¸ ¨ ¸
¨ 10 P ¸ ˜ ¨ 1¸ 0 ----- (*)
¨ 20(1  P ) ¸ ¨ 2 ¸
© ¹ © ¹
4
10P  40  40P 0ŸP
5
§ 0 · §0·
¨ ¸ ¨ ¸
OR ¨ 110 54 ¸ ¨ 8 ¸
¨ 20(1  4 ) ¸ ¨ 4 ¸
© 5 ¹ © ¹
Therefore, coordinates of R is (0, 8, 4)

Method 2 (Use concept of point N is a point on line l)

§ 0 ·
¨ ¸
Using ratio theorem, OR P OC  (1  P )OA ¨ 10P ¸
¨ 20(1  P ) ¸
© ¹
§ 20 ·
¨ ¸
BR OR  OB ¨ 10P ¸
¨ 20(1  P ) ¸
© ¹

§ 20 · § 2 ·
¨ ¸ ¨ ¸
Equation of line l: r ¨ 0 ¸ D ¨ P ¸, P 
¨0¸ ¨ 2(1  P ) ¸
© ¹ © ¹

Given N lies on line l,


§ 10 ·
¨ 3 ¸
¨ ¸ § 20 · § 2 ·
¨ 20 ¸ ¨ 0 ¸  D ¨ P ¸
¨ 3 ¸ ¨ ¸ ¨ ¸
¨ ¸ ©¨ 0 ¹¸ ¨ 2(1  P ) ¸
© ¹
¨¨ 10 ¸¸
© 3 ¹

10 2
255
20  2D ŸD
3 3
20 4
DP P ŸP
3 5
10
2D (1  P )
3
§ 0 · §0·
¨ ¸ ¨ ¸
OR ¨ 110 5 ¸
4
¨8¸
¨ 20(1  4 ) ¸ ¨ 4¸
© 5 ¹ © ¹

Therefore, coordinates of R is (0, 8, 4)


© TJC 2019
14

10 [Solution]
(i) dx
x 2t  sin 2t Ÿ 2  2 cos 2t
dt
dy
y cos 2t Ÿ 2sin 2t
dt
dy 2sin 2t  sin 2t
dx 2  2 cos 2t 1  cos 2t
2sin t cos t

1  (2 cos 2 t  1)
sin t

cos t
 tan t (shown)
S dy S
As t ,  tan is undefined
2 dx 2
The tangent to the curve is parallel to the y-axis.

(ii)
When y = 0, cos 2t 0
S 3S
For 0 d t d S , 2t or
2 2
S 3S
t or
4 4
S 3S
Ÿ x1  1 or x2 1
2 2

(iii)
t 0, x 0 & y 1 y
t S , x 2S & y 1

(0, 1) (2S, 1)

O x
S
t ,x S & y 1 (S, 1)
2

(iv) Area =  x2 y dx
³ x1

 ³π cos 2t u ((2
2  2 cos 2t ) dt
4

4
3S
 ³S 2 cos 2t + 2 cos 2 2t dt
4

4
3S
 ³S 2 cos 2t + 1  cos 4t dt
4

© TJC 2019
15

3S
­° 3S
ª sin 4t º 4 ½°
 ®>sin 2t @ S  «t 
4
¾
°¯ 4
¬ 4 »¼ S4 °¿
­§ 3S S · § § 3S sin 3S · § S sin S · ·½
 ®¨ sin  sin ¸  ¨ ¨  ¸¨  ¸ ¸¾
¯© 2 2 ¹ ©© 4 4 ¹ ©4 4 ¹ ¹¿
§ S·
 ¨ 2  ¸
© 2¹
S
2
2

(v) S Sdy S
From (ii), x1  1, t  tan
, 1
2 4 dx 4
§ §S ··
Equation of tangent: y  ¨ x  ¨  1¸ ¸
© © 2 ¹¹
When the tangent intersects C again,
§ § S ··
cos 2t  ¨ 2t  sin 2t  ¨  1¸ ¸
© © 2 ¹¹
S
i.e. cos 2t  sin 2t 2t 1
2
Using GC, t = 1.99 (3 s.f) , t = 0.785 (i.e. S/4)

Thus the tangent to C intersects C again at t = 1.99 (3 s.f).

End of Paper

© TJC 2019
-&+0DWK3URPR([DP3DSHU9-&
2

1 (i) Sketch the curve with equation y 3ln x  3 , giving the equation of the asymptote and the
coordinates of any points of intersection with the axes. On the same diagram, sketch the curve
5
with equation y x3. [3]
5
(ii) Solve the inequality 3ln x  3 x3. [2]

d2 y
2
Given that 3x 2  y 2 dy
dx
2 xy , and that y 1 when x 0 , find the values of
dy
dx
and
dx 2
when x 0 .
Hence write down the first two non-zero terms in the Maclaurin series for y . [4]

1 2x
3 (i) Using integration, show that ³ e2 x sin x dx e 2sin x  cos x  C , where C is an arbitrary
5
constant. [4]

(ii) Hence find the gradient of the curve


y e 2 x  2 ª¬ 2sin x  1  cos x  1 º¼

S
at x  1 , leaving your answer in an exact form. [2]
2

4 The curve C has equation


y  2 2  x  3 2 1.
9 4
(i) Sketch C , giving the equations of its asymptotes and the coordinates of any turning points.
[4]
The curve D has equation 12 y 2  48 y  48  ax2  6ax  3a 0 , where a is a positive constant.
(ii) Find the set of values of a for which C and D do not intersect. [2]

1  x2
5 (i) Find the binomial expansion for 2
, up to and including the term in x 4 . Give the
2 x
coefficients as exact fractions in their simplest form. [3]
(ii) Find the set of values of x for which this expansion is valid. [2]


2
(iii) Using your answer in part (i), find the series expansion of 2  x 2 , up to and including the

term in x 2 . [3]
3

6 Functions f and g are defined by


f:x 1  O x2 , x  , x  1,
1 x
g:x 2e , x  , x 1,
where O is a positive constant.

(i) Find f  1 x and state the domain of f  1 . [3]

(ii) Show that gf exists and find the range of gf, giving your answer in terms of O . [4]

The function h is defined by


h:x 1  O x2 , x  , x  k,
where k is a constant.
Determine the set of values of k for which the range of gh is the same as the range of g. [1]

ax  b
7 The diagram below shows the graph of y g( x) , where g x .
2x  c
y

x
O

Determine the values of a, b and c. [3]


§1 ·
It is also given that g( x) f ¨ x  1¸ . State a sequence of 2 transformations that will map the graph
©2 ¹
of y g( x) to the graph of y f ( x) .
Find f ( x) . [5]

[Turn Over
4

8 (a) Find ³ sin 2 x cos6 2 x dx. [2]

3x
(b) (i) Find ´
µ 2 dx . [2]
¶ x 2

2 3x  1 Ax 2  Bx  1
(ii) Show that  , where A and B are constants to be found. [2]
x  3 x2  2 x  3 x 2  2
2
´ 10 x 2  16 x  2
(iii) Using your answers to parts (i) and (ii), find µ dx .

µ x  3 x 2  2
¶0
Give your answer in the form a tan  1 b  ln c , where a , b and c are constants to be
determined. [4]

9
y

x
O

The diagram above shows the graph of y f ( x) . The curve passes through (0,0) and (– 2,0), and

1
has a minimum point at (– 4, –2). The curve has asymptotes x 1 and y  .
2
(a) State the coordinates of the turning point of the curve y 1  2f ( x) . [1]

(b) On separate diagrams, sketch the graphs of


(i) y f ( x ), [2]

1
(ii) y , [3]
f ( x)

(iii) y f c x . [2]
5

D x2  x  1
10 A curve C has equation y , where D is a real, non-zero constant.
x2
Show that if C has 2 stationary points, then D  0 or D ! k , where k is a constant to be determined.
[4]
Sketch the curve C for D 1 , giving the equations of asymptotes, the coordinates of stationary
points and points of intersection with the axes. [4]

By considering C and an appropriate line, find the range of values of m such that the equation

 x 2  x  1 mx 2  2m  3 x  6

has exactly 1 positive root and 1 negative root. [3]

[Questions 11 & 12 are printed on the next page]

[Turn Over
6

11 During test drives, a sensor is used to record the number of revolutions per minute made by a
particular wheel of a vehicle.
(a) In a test drive, a car is initially travelling at constant speed but starts to slow down due to
engine malfunction. The total number of revolutions is recorded on every 1-minute interval
after malfunction. In the first n minutes, the total number of revolutions recorded, S n , is

given by Sn 54n 29  n .

(i) Show that the number of revolutions recorded in each minute after the malfunction
occurs, before the car comes to a complete stop, follows an arithmetic progression. [3]

(ii) The diameter of each wheel of the car is measured to be 61 cm. Show that the car travels
a distance of 21.7 km, correct to 3 significant figures, from the time the malfunction
occurs until it comes to a complete stop. [3]

(b) In another test drive, a truck was travelling at constant speed before it entered the roughterrain.
Before entering the rough terrain, the wheel was rotating at 486 revolutions per minute (rpm). After
entering the rough terrain, engine power increases the rate of rotation by 20 rpm almost immediately
at the beginning of each minute. However, at the end of each minute, friction slows the truck down
2
such that the rate of rotation is of that recorded at the beginning of that minute. The rate of
3
rotation of the wheel at the end of the the nth minute after entering the rough terrain is denoted by
vn rpm.
n
§2·
(i) Show that vn 446 ¨ ¸  40 . [4]
©3¹
(ii) Explain why the wheel always rotates at a rate of more than 40 rpm. [2]
(iii) Given that the rate of rotation of the wheel was less than 45 rpm at the end of m minutes, find
the least integer value of m. [2]
7

12 The diagram shows a string that is unwound from a circle while being held taut. The curve traced
by the end point P of the string is called the involute of the circle. One of the major applications of
involute of circle is in designing of gears for revolving parts where gear tooth follow the shape of
involute.

Involute of the circle

A circle has fixed radius a units and centre O and the initial position of P is at a, 0 .

§ S·
The parameter T , ¨ 0 T ¸ , is the angle measured from the positive x-axis to OT in the anti-
© 2¹
clockwise direction, where T is the point on the circle such that PT is tangential to the circle.
Show that the involute has parametric equations
S
x a cosT  T sin T , y a sin T  T cosT , for 0 T . [3]
2

S
The point W on the involute has parameter T .
3
(i) Show that the equation of the normal to the involute at W is
3y 2a  x . [5]
(ii) At W, x increases at a rate of 0.3 units per second. Given that z xy , determine, in terms of a,

the rate of change of z at W. [4]

[Turn Over
2019 H2 Mathematics Promo Solution

Qn Solution
1i

ii 5
Graphs of y 3ln x  3 and y x 3 intersect at x 0.395 and x 3.02 .
5
3ln x  3 t x 3
? 0.395 d x d 3.02
2
3x2  y 2 ddyx 2 xy    1
Differentiating w.r.t. x
2

3 x 2
 y dx2  §¨© 6x  2 y ddyx ·¸¹ ddyx 2 y  2x ddyx
2 d y

When x 0 ,
y 1
dy dy
0  1 2 0 1 Ÿ 0
dx dx
d2 y d2 y
0  1 2  0  2 0 0 2  2 0 0 Ÿ 2 2
dx dx
? the Maclaurin’s series for y is
2 2
y 1  0 x  x 
2!
1  x2 
2019 H2 Mathematics Promo Solution

Qn Solution

³
3i
e 2 x sin x dx

³
1 2x 1 2x
e sin x  e cos x dx
2 2

³
1 2x 1 ª§ 1 · 1 2x º
e sin x  «¨ e 2 x cos x ¸  e  sin x dx »
2 2 ¬© 2 ¹ 2 ¼

³
1 2x 1 1
e sin x  e 2 x cos x  e 2 x sin x dx
2 4 4

³
5 1 2x 1
e 2 x sin x dx e sin x  e 2 x cos x  D
4 2 4

³
2 2x 1
e 2 x sin x dx e sin x  e 2 x cos x  C
5 5
1 2x
e 2sin x  cos x  C
5
ii Let f ( x) e2 x 2sin x  cos x .
Observe that y e 2 x  2 2sin x  1  cos x  1 f ( x  1)
is a translation of y f ( x) by 1 unit in the negative x-direction.
S
Hence, the gradient of the curve y f ( x  1) at x  1 is the gradient of the curve y f ( x) at
2
S §S ·
x , which is given by y f c ¨ ¸ .
2 ©2¹

From part (i), f c( x) 5e2 x sin x .


§S ·
Hence, the required gradient is f c ¨ ¸ 5eS .
©2¹
4i Asymptotes:
y  2 x  3
2 2

9 4
y2 x3
r
3 2
3 x  3
y r 2
2
3 5 3 13
13
y x or y  x
2 2 2 2
2019 H2 Mathematics Promo Solution

Qn Solution

ii 12 y 2  48 y  48  ax 2  6ax  3a 0
12 y  4 y  a x  6 x  3a  48 0
2 2

12 y 2  4 y  4  48  a x 2  6 x  9  9a  3a  48 0

12 y  2  a x  3
2 2
12a
y  2 x  3
2 2

 1
a 12

which is an ellipse with centre 3,2 and vertices at 3, 2  a and 3, 2  a
For curve C and D to not intersect,
a  3Ÿ a  9
Since a is a positive constant,
?^a  : 0  a  9`
5i 1  x2
2  x2
1  x 2  x
2 2 1

1
§ 2
·
1  x 2 ¨1  x2 ¸
2 1

© ¹
ª § x 2 · 1
2 § x 2 · º
2
1
2
1  x «1  1 ¨  2 ¸  22!! ¨  2 ¸ 
2
»
«¬ © ¹ © ¹ »¼
1 § x2 x4 ·
2
1  x 2 ¨1   
2 4
¸
© ¹
1§ x2 x4 x4 ·
¨1  x    
2
¸
2© 2 2 4 ¹
1 3x 2 3x 4
  
2 4 8
2019 H2 Mathematics Promo Solution

Qn Solution
ii For expansion to be valid,
x2 x2
 1Ÿ 1
2 2
x2  2

x2  2 x 2
x 2 since x 2 t 0
x  2 x  2  0

^ x  :  2  x  2`
iii d § 1  x2 · d § 1 3x 2 3x 4 ·
¨ ¸ ¨    ¸
dx © 2  x 2 ¹ dx © 2 4 8 ¹
2  x 2 x  1  x 2 x
2 2
6 x 12 x3
 
2  x 2 2 4 8

4 x  2 x3  2 x  2 x3 6 x 12 x 3
 
2  x 2 2 4 8

6 x 12 x3
6 x 2  x2
2
 
4 8
2

2  x 2 14  x4 
2

6i Let y f x 1  O x 2 , O ! 0
1 y
x2
O
1 y
x r
O
1 y
 x  1
O
1 x
f 1
x 
O

Df  1 Rf
f,1  O

6ii Rf f,1  O
Since O ! 0 Ÿ1  O  1, heenncce R f Ž Dg
hhence f,1@ .
? gf exists

R gf f, 2  e O
2019 H2 Mathematics Promo Solution

Qn Solution
6 Since g is a one –one function,
then if R gh R g , we
will have R h Dg

Rh Dg f,1@

?^k  : k ! 0`

7 ax  b
y g( x)
2x  c
c 3
Vertical asym : x  Ÿ c 3
2 2
a
Horizontal asym : y 2 Ÿ a 4
2
b 4
y-intercept : y  Ÿb 4
c 3
4 x  4
? y g( x)
2x  3

§1 ·
y f ¨ x  1¸
©2 ¹

Replace x by x+2

§1 · §1 ·
y f ¨ x  2  1¸ f ¨ x ¸
©2 ¹ ©2 ¹

Replace x by 2x

§1 ·
y f ¨ 2x ¸ f x
©2 ¹
§1 ·
The graph of y f ¨ x  1¸ is translated te 2 units in the negative x-direction and then stretched
©2 ¹
1
parallel to the x-axis by factor
ctor
orr with
wi
with y-axis
y-axis invariant.
2
2019 H2 Mathematics Promo Solution

Qn Solution
§1 · 4 x  4
f ¨ x  1¸
©2 ¹ 2x  3
§1 · 4 x  2  4
f ¨ x¸
©2 ¹ 2 x  2  3
4 x  4
2x  1
4 2 x  4
f x
2 2x  1
8 x  4
4x  1

Alternatively,
§1 ·
y f ¨ x  1¸
© 2 ¹

Replace x by 2x

§1 ·
y f ¨ 2 x  1¸ f x  1
©2 ¹

Replace x by x+1

y f x  1  1 f x
§1 · 1
The graph of y f ¨ x  1¸ is stretched parallel to the x-axis by factor with y-axis invariant and
©2 ¹ 2
then translated 1 unit in the negative x-direction.
§1 · 4 x  4
f ¨ x  1¸
©2 ¹ 2x  3
4 2 x  4
f x  1
2 2x  3
8 x  4
4x  3
8 x  1  4
f x
4 x  1  3
8 x  4
4x  1

³ ³
8a 1
sin 2 x cos 6 2 x dx  2sin 2 x cos 6 2 x dx
2
1 § cos 7 2 x ·
 ¨ ¸C
2© 7 ¹
cos7 2 x
 C
14
2019 H2 Mathematics Promo Solution

Qn Solution
8bi
³ ³
3x 3 2x
dx dx
x 2
2
2 x 2 2

3
ln x 2  2  C
2
3
ln x 2  2  C ( x 2  2 ! 0)
2
8bii 3 x  1 2 x  2  3 x  1 x  3
2
2

x  3 x2  2 x  3 x 2  2
2 x 2  4  3x 2  8 x  3
x  3 x 2  2
5x2  8x  1
x  3 x 2  2

biii 2

³
10 x 2  16 x  2
dx
0 x  3 x 2  2
2

³
5x2  8x  1
2 dx
0 x  3 x 2  2
2

³
§ 2 3x  1 ·
2 ¨  2 ¸ dx
0 © x3 x  2¹
2

³
§ 2 3x 1 ·
2 ¨  2  2 ¸ dx
0 © x3 x  2 x  2¹
2
ª 3 1 § x ·º
2 « 2ln x  3  ln x 2  2  tan 1 ¨ ¸»
¬ 2 2 © 2 ¹¼0
ª 3 1 § 2 ·º
2 « 2ln1  ln 6  tan 1 ¨ ¸»
¬ 2 2 © 2 ¹¼
ª 3 1 º
 2 « 2ln 3  ln 2  tan 1 0 »
¬ 2 2 ¼
3ln 6  2 tan 1 2  4ln 3  3ln 2
2 tan 1 2  ln 3
9a Effects on turning pointnt
nt
4, 2 on y f x
Reflection
R
Reefl
fleeccti
tion
on iinn x – aaxis, and stretch //
o yy-axis,
to -ax
axisis,, fa
ffactor
ct 2, x - axis invariant

y
4,4 on  f x ª¬i.e. y 2f x º¼
2
Translate 1 unit in the positive y - direction
2019 H2 Mathematics Promo Solution

Qn Solution
4,5 on y 1  2f x
4,5 is the turning point of the curve y 1  2f x

Alternatively,

Sub x 4
y 1  2f 4 1  2 2 5
4,5 is the turning point of the curve y 1  2f x
bi
y

bii
y

x
O
2019 H2 Mathematics Promo Solution

Qn Solution
b iii
y

x
O

10 D x2  x  1
y
x2

dy x  2 2D x  1  D x2  x  1 D x2  4D x  1
x  2 x  2
2 2
dx
dy
For C to have 2 stationary points, 0 has 2 real roots.
dx
For D x 2  4D x  1 0 to have 2 real roots,
Discrimant > 0
4D  4D ! 0
2

4D 4D  1 ! 0

+ – +
α
0
1
D  0 or D !
4
1
?k
4

Alternatively,

D x2  x  1 4D  1
y D x  1  2D 
x2 x2
dy 4D  1
D
x  2
2
dx
2019 H2 Mathematics Promo Solution

Qn Solution
dy
For C to have 2 stationary points, 0 has 2 real roots.
dx
dy 4D  1
0ŸD  0
x  2
2
dx
4D  1
Ÿ x  2
2

for equation to have 2 real roots,


4D  1
!0
D
+ – + α
0
1
D  0 or D !
4
1
?k
4
y

x
O

 x 2  x  1 mx 2  2m  3 x  6
mx x  2  3 x  2
mx
 x2  x  1
mxx  3
x2

Add line y mx  3 such that there are exactly 2 points of intersection with C, one with x  0 and
one with x ! 0 .
For the equation to have 1 positive root and 1 negative root,
m  1
2019 H2 Mathematics Promo Solution

Qn Solution
11 No. of revolutions in nth minute,
ai un S n  S n 1
54n 29  n  54 n  1 29  n  1
1620  108n

un  un 1 1620  108n  ª¬1620  108 n  1 º¼


108

Since un  un 1 108 is a constant (independent of n), the number of revolutions made in each
minute follows an arithmetic progression.
11 For un 1620  108n d 0,
aii n t 15.
Total number of revolutions,
S15 54 15 29  15 11340
Distance travelled
11340 u S u 61 cm
21732165 cm
21.7 km (to 3 s.f.) (shown)
bi
486  20 §¨

v1 ¸
©3¹
ª §2· º§ 2 ·
v2 « 486  20 ¨ 3 ¸  20 » ¨ 3 ¸
¬ © ¹ ¼© ¹
2 2
§2· §2· §2·
486 ¨ ¸  20 ¨ ¸  20 ¨ ¸
©3¹ ©3¹ ©3¹

n n n 1
§2· §2· §2· §2·
vn 486 ¨ ¸  20 ¨ ¸  20 ¨ ¸   20 ¨ ¸
©3¹ ©3¹ ©3¹ ©3¹
ª 2 § § 2 ·n · º
« ¨¨1  ¨ ¸ ¸¸ »
«3 © 3 ¹ ¹»
n
§2·
486 ¨ ¸  20 « © »
« 1  §¨ ·¸ »
©3¹ 2
« ©3¹ »
¬ ¼
§2·
n
§ §2· n
·
486 ¨ ¸  40 ¨1  ¨ ¸ ¸¸
©3¹ ¨ ©3¹
© ¹
n
§2·
446 ¨ ¸  40
40 (shown)
((sshhoown
wn)
©3¹
bii §2·
n
§2·
n

Since ¨ ¸ ! 0 for all n ! 0 , vn 446 ¨ ¸  40 ! 40 .


©3¹ ©3¹
Thus, the wheel always rotates at a rate of more than 40 rpm.
2019 H2 Mathematics Promo Solution

Qn Solution
biii §2·
m

446 ¨ ¸  40  45
©3¹
m
§2· 5
¨ ¸ 
©3¹ 446
5 2
m ! ln y ln
446 3
m ! 11.1 (to 3 s.f.)
Least m 12
Alternative method
m
§2·
446 ¨ ¸  40  45
©3¹
m §2·
m

446 ¨ ¸  40
©3¹
11 45.156 > 45
12 43.437 < 45
13 42.292 < 45
From the GC,
Least m 12
12 OQ a cosT ; TQ a sin T
TP aT (arc length of unit circle)
SP aT sin T ; TS aT cosT

x OQ  SP
a cos T  aT sin T a
(shown)

y TQ  TS
a sin T  aT cos T (shown)

i dx
a sin T  a sin T  aT cosT aT cosT
dT
dy
a cosT  a cosT  aT sin T aT ssin
in T
dT
dy aT sin T
tan T
dx aT cosT
S
When T
3
§ S S S· §1 S 3·
x a ¨ cos  sin
sin
i ¸ a ¨¨  ¸
© 3 3 3¹ ©2 6 ¸¹

§ S S S· § 3 S·
y a ¨ sin  cos ¸ a ¨¨  ¸¸
© 3 3 3¹ © 2 6¹
2019 H2 Mathematics Promo Solution

Qn Solution
1 1
Gradient of normal  
S 3
tan
3
Equation of normal at W is
§ 3 S· 1 § a Sa 3 ·
y  a ¨¨  ¸¸  ¨¨ x   ¸
© 2 6¹ 3© 2 6 ¸¹
§ 3 · S 3a a S 3a
3 y  3a ¨¨ ¸¸  x  
© 2 ¹ 6 2 6
a 3a
3y  x
2 2
3 y 2a  x (shown)
12ii S dx
At T , 0.3
3 dt
dy dy dx § S· 3 3
u ¨ tan ¸ 0.3
dt dx dt © 3¹ 10
z xy -----(1)
Differentiate (1) w.r.t. t
dz dy dx
x y
dt dt dt
§ 1 S 3 ·§ 3 3 · § 3 S·
a ¨¨  ¸¨
¸¨ ¸¸  a ¨¨  ¸¸ 0.3
©2 6 ¹© 10 ¹ © 2 6¹
0.834a (3sf)

Alternatively,
dz dy
yx
dx dx
§ 3 S· §1 S 3·
a ¨¨
 ¸¸  3 a ¨¨  ¸
6 ¸¹
© 2 6¹ ©2
2.7792a
dz dz dx
u 2.7792a 0.3 0.834a (3sf)
dt dx dt
+0DWK3URPR<,-&

Questions from 2019 YIJC Promos

1 Without using a calculator, solve the inequality


x2  4 x  7
0. [4]
2 x2  9 x  5

2 Removed (not in syllabus)


ecos3 x 1  3 x sin 3 x
³
3 Differentiate 2xe cos3 x
with respect to x. Hence find dx . [3]
1+2xecos3 x

2
4 (a) Find ³ x(ln x) dx . [4]
(b) The region R is bounded by the curves y tan x , y cos 2 x and the y-axis. Find
the numerical value of the volume of the solid obtained when R is rotated through
2S radians about the y-axis. [3]

5 A curve y f ( x) undergoes, in succession, the following transformations.


A: A translation of 1 unit in the negative x-direction.
B: A reflection about the y-axis.
C: A scaling parallel to the y-axis with scale factor of 2.
4x 1
The equation of the resulting curve is y g( x) , where g( x) .
x2  3
(i) Find f ( x) . [3]
(ii) Find the exact area of the region bounded by the graph of y g( x) , the x-axis and
the lines x 1 and x 3 . [4]

6 The function f is defined as follows.


3
f :x x 2  3x  2,
2, x , x t .
2
(i) Find f 1( x) and write down the domain and range of f 1 . [4]

The function g is defined as follows.


g : x ln x, x , x ! 0.
(ii) Explain why the composite function gf 1 exists. [1]
(iii) Find gf 1( x) , stating the domain and range of gf 1 . [3]
7 The curve C has equation
2 x  y 2 x  2y .
It is given that C has only one turning point.
dy 5
(i) Show that 2 . [3]
dx 4x  2 y  2
3
d2 y
§ dy ·
(ii) Hence, or otherwise, show that 2
k ¨  2 ¸ , where k is a constant to be
dx © dx ¹
determined. [3]
(iii) Hence state, with a reason, whether the turning point is a maximum or a minimum.
[2]

8 Removed (not in syllabus)

9 (a) The function f is given by


­ x  1, 0 d x 1
°
f x ®  x  3, 1d x  3
°0, 3d x  4
¯
and f x f x  4 for all real values of x.
(i) Sketch the graph of y f x for 2 d x d 7 . [3]
7
(ii) Hence find ³2
f ( x) dx . [2]
(b) The diagram shows the curve y g x with asymptotes y 2 , x 10 and
x 0 . The maximum point and the minimum point of the curve are 5, 2 and
7,1 respectively.
y

y=2
x

x x

x = –10 x=0
On separate diagrams, sketch the graphs of
1
(i) y , [3]
g x
(ii) y gc x , [3]
stating clearly the equations of any asymptotes, the coordinates of any turning
points and any points of intersection with the x- and y-axes.

10 A curve C has parametric equations


S
x 2sin 3 t , y 5cos t for 0  t d
.
2
(i) Sketch the curve C, stating clearly the coordinates of the points of intersection
with the axes. [2]
S
(ii) The tangent to the curve at the point P where t is denoted by l. Show that the
4
equation of l is 3 y  5x 10 2 . [4]
(iii) The tangent l meets the x-axis at Q. Given that the point R has coordinates
§ 1 ·
¨ , 0 ¸ , find the exact area of triangle PQR. [2]
© 2 ¹
1
(iv) Show that the area of the region bounded by the curve C, the line x and the
2
S

³
15 2
x-axis is given by sin 2 2t dt . Hence find the exact area of the region
2 S
4
bounded by the curve C, the tangent l and the x-axis. [6]
1 2
11 [It is given that a cone with base radius r and height h has volume S r h and external
3
curved surface area, also known as lateral area, S r r 2  h2 .]
A conical paper cup is to hold a fixed volume, k cm3 of water. It is given that the cup has
height h cm, base radius r cm and lateral area S cm2.
9k 2
(i) Show that S 2 S 2r 4  . [2]
r2
(ii) Given that the lateral area is a minimum, use differentiation to find the values of
r and h in terms of k. Simplify your answers. [5]

It is now given that the volume of the cup is 125 cm3.


(iii) Sketch the graph of S against r, indicating clearly the coordinates of the minimum
point. [3]
2
(iv) It is also given that the lateral area is 120 cm and the base radius is less than
4 cm. Find the values of r and h. [2]
1 Equations & Inequalities 1
5  x 
2
3 Integration & Applications 1

1  3x sin 3x ; 1+2xe 2  C
cos3 x cos3 x
2e
4 Integration & Applications x2 x2 x2
ln x  ln x   C
2

2 2 4
(a)
§ x2 ª ·
2 ln x  2 ln x  1º  C ¸
2
¨ or
© 4 ¬ ¼ ¹
(b) 0.324 (to 3 s.f.)

5 Graphs & Transformations 3  4x 3  4x


(i) f ( x) or
2( x  1)2  6 2x  4x  8
2

S
(ii) 2 ln 3 
6 3
6 Functions 3 1 ª 1 · ª3 ·
(i) f 1 x  x  , Df 1 «¬  4 , f ¸ , R f 1 «¬ 2 , f ¸
2 4 ¹ ¹
§3 1· ª 1 ·
(iii) gf 1 ( x) ln ¨¨  x  ¸¸ , Dgf 1 «¬  4 , f ¸¹ ,
©2 4¹
ª §3· ·
R gf 1 «ln ¨ ¸ , f ¸ or > 0.405, f
¬ ©2¹ ¹

7 Differentiation & 2
(ii) k 
Applications 5
9 Graphs & Transformations 15
(a)(ii) 2
10 Integration & Applications 15
(iii)
4
15 15
(iv)  S
4 16
11 Differentiation
fferen ntitiat on &
atiio 1 1 1 1
Applications
App
Ap icaattio
pllic ns
ions § 9k 2 · 6 § 9 ·6 § k ·3 § 6k · 3
(ii)
( i) r ¨¨ 2 ¸¸ or
(i ¨ ¸ ¨ ¸ , h ¨ ¸
© 2S ¹ © 2¹ ©S ¹ ©S ¹
(iv) r 3.25 (3 s.f.), h 11.3 (3 s.f.)

You might also like